You are on page 1of 241

Phm Th Long Ch bin

Nguyn c Hiu, Nguyn Thin Lun


Nguyn Xun Vin, Nguyn Vn Xut

Ton ri rc

H Ni 2003

Li ni u
Ton ri rc l mt trong nhng kin thc c s -c ging dy tt c cc khoa
Cng ngh Thng tin hin nay. Tuy nhin, tu theo yu cu kin thc v cu trc ca
ch-ng trnh o to m kt cu mn hc mi ni t nhiu c th khc bit. Nhm p ng
nhng yu cu a dng v kin thc, trong cun sch ny cc tc gi c gng gii thiu
mt cch c ng hu ht nhng ni dung c bn ca Ton hc ri rc, bao gm cc kin
thc c s v logic, tp hp v i s quan h (Ch-ng I); mt s bi ton trong l
thuyt t hp (Ch-ng II); th v cc bi ton trn th (Ch-ng III); i s Boole v
ng dng trong phn tch mch in t (Ch-ng IV); ngn ng hnh thc v tmat (Ch-ng
V). Trong cch trnh by cun sch, cc tc gi quan tm nhiu hn n k thut gii
quyt vn , khng qu cu n vo nhng i hi cht ch v mt ton hc theo kiu
nh l-chng minh, khng t khi nim v kt qu ch yu -c trnh by thng qua cc
v d v bi tp.

Ngoi kh nng t- duy lgic nht nh, gio trnh khng i hi t pha bn c mt
s chun b c bit no v ton hc ni chung, v vy c th b tr ging dy theo cun
sch ny ngay t hc k 1 nm th nht cc tr-ng i hc v cao ng. Nu c mt s
hiu bit c bn v lgic v tp hp (trong phm vi cc mc 1-3 ca ch-ng I), bn c
c th tm hiu bt k ch-ng sau no m khng cn tun th trnh t cc ch-ng nu
trong cun sch. Trong s cc ti liu tham kho -c nu cui sch, cc tc gi mun
bn c c bit l-u ti liu Ton ri rc ng dng trong Tin hc ca Kenneth H. Rosen.
S phong ph v a dng ca cc v d v bi tp trong cun sch s ht sc hu ch
cho bn c. Nhng ai quan tm n vic ch-ng trnh ho mt s thut ton nu trong
cun sch ny c th tham kho thm ti liu Ton ri rc ca Nguyn c Ngha v
Nguyn T Thnh.
Phn cng cng vic gia cc tc gi nh- sau:

Ch bin v hiu nh ton b ni dung bn tho: Phm Th Long.


Ch-ng 1: Nguyn Xun Vin.

Ch-ng 2: Nguyn Thin Lun, Phm Th Long.


Ch-ng 3: Nguyn c Hiu, Phm Th Long.
Ch-ng 4: Phm Th Long.

Ch-ng 5: Nguyn Vn Xut.

Cc tc gi xin by t lng cm n chn thnh PGS.TSKH Nguyn Xun Huy (Vin


Cng ngh Thng Tin), PGS.TS ng Huy Run (HQG H Ni) c k bn tho v
cho nhiu kin ng gp xc ng.
Chc chn khng th trnh khi nhng thiu st trong cun sch ny. Cc tc gi
rt mong nhn -c s ch bo v ng gp ca tt c bn c c th hon chnh ni
dung cho nhng ln xut bn sau.

Cc tc gi

Ch-ng 1

Mc lc

Nhng khi nim c bn v logic, tp hp v suy lun ton hc

1. Mnh , mnh c iu kin v s t-ng -ng logic . . . . . . . . . . . . . . . . . . . . .

1.2 Cc php ton trn mnh . . . . . . . . . . . . . . . . . . . . . . . . . . . . . . . . . . . . . . . . . . . . .

1.1 Mnh . . . . . . . . . . . . . . . . . . . . . . . . . . . . . . . . . . . . . . . . . . . . . . . . . . . . . . . . . . . .
1.3 Mnh c iu kin v s t-ng -ng logic . . . . . . . . . . . . . . . . . . . . . . . . . . . . . .

2. Tp hp v cc php ton trn tp hp . . . . . . . . . . . . . . . . . . . . . . . . . . . . . . . . . . .


2.1 Tp hp, tp con v tch Decac . . . . . . . . . . . . . . . . . . . . . . . . . . . . . . . . . . . . . . . .

5
7
9
9

2.2 Cc php ton trn tp hp . . . . . . . . . . . . . . . . . . . . . . . . . . . . . . . . . . . . . . . . . . . . . .

10

3.1 Hm mnh . . . . . . . . . . . . . . . . . . . . . . . . . . . . . . . . . . . . . . . . . . . . . . . . . . . . . . . .

13

3. L-ng t v v t . . . . . . . . . . . . . . . . . . . . . . . . . . . . . . . . . . . . . . . . . . . . . . . . . . . . .

3.2 V t . . . . . . . . . . . . . . . . . . . . . . . . . . . . . . . . . . . . . . . . . . . . . . . . . . . . . . . . . . . . . . .
3.3 Ph nh ca v t . . . . . . . . . . . . . . . . . . . . . . . . . . . . . . . . . . . . . . . . . . . . . . . . . . . .

4. Quan h . . . . . . . . . . . . . . . . . . . . . . . . . . . . . . . . . . . . . . . . . . . . . . . . . . . . . . . . . . . .

4.1 Khi nim v tnh cht . . . . . . . . . . . . . . . . . . . . . . . . . . . . . . . . . . . . . . . . . . . . . . . .


4.2 Ma trn quan h . . . . . . . . . . . . . . . . . . . . . . . . . . . . . . . . . . . . . . . . . . . . . . . . . . . . . .
4.3 Quan h t-ng -ng, lp t-ng -ng . . . . . . . . . . . . . . . . . . . . . . . . . . . . . . . . . . .
4.4 Quan h n - ngi. C s d liu quan h . . . . . . . . . . . . . . . . . . . . . . . . . . . . . . . . . . .

5. Suy lun ton hc . . . . . . . . . . . . . . . . . . . . . . . . . . . . . . . . . . . . . . . . . . . . . . . . . . . .

5.1 Cc ph-ng php chng minh . . . . . . . . . . . . . . . . . . . . . . . . . . . . . . . . . . . . . . . . . . .


5.2 Quy np ton hc . . . . . . . . . . . . . . . . . . . . . . . . . . . . . . . . . . . . . . . . . . . . . . . . . . . . .

5.3 quy v ng dng . . . . . . . . . . . . . . . . . . . . . . . . . . . . . . . . . . . . . . . . . . . . . . . . . .


Bi tp . . . . . . . . . . . . . . . . . . . . . . . . . . . . . . . . . . . . . . . . . . . . . . . . . . . . . . . . . . . . . . . .
Ch-ng 2

Cc ph-ng php m v nguyn l Dirichlet

13
14
15
16
16
19
20
22
26
26
28
29
31
39

1. Cc nguyn l m c bn . . . . . . . . . . . . . . . . . . . . . . . . . . . . . . . . . . . . . . . . . . . . . .

39

1.2 Nguyn l nhn . . . . . . . . . . . . . . . . . . . . . . . . . . . . . . . . . . . . . . . . . . . . . . . . . . . . . .

40

1.1. Nguyn l cng . . . . . . . . . . . . . . . . . . . . . . . . . . . . . . . . . . . . . . . . . . . . . . . . . . . . . .

2. Mt s bi ton m c bn: Hon v, t hp, chnh hp . . . . . . . . . . . . . . . . . . . . .

2.1. Chnh hp lp . . . . . . . . . . . . . . . . . . . . . . . . . . . . . . . . . . . . . . . . . . . . . . . . . . . . . . .
2.2. Chnh hp khng lp . . . . . . . . . . . . . . . . . . . . . . . . . . . . . . . . . . . . . . . . . . . . . . . . . .

39
42
42
42

2.3. Hon v . . . . . . . . . . . . . . . . . . . . . . . . . . . . . . . . . . . . . . . . . . . . . . . . . . . . . . . . . . . .

43

2.5. T hp lp . . . . . . . . . . . . . . . . . . . . . . . . . . . . . . . . . . . . . . . . . . . . . . . . . . . . . . . . . .

45

2.4. T hp . . . . . . . . . . . . . . . . . . . . . . . . . . . . . . . . . . . . . . . . . . . . . . . . . . . . . . . . . . . .
2.6. Hon v ca tp hp c cc phn t ging nhau . . . . . . . . . . . . . . . . . . . . . . . . . . . . .

2.7. Phn b cc vt vo trong hp . . . . . . . . . . . . . . . . . . . . . . . . . . . . . . . . . . . . . . . .


2.8. So snh cc cu hnh t hp . . . . . . . . . . . . . . . . . . . . . . . . . . . . . . . . . . . . . . . . . . . .

3. Sinh cc cu hnh t hp . . . . . . . . . . . . . . . . . . . . . . . . . . . . . . . . . . . . . . . . . . . . . . .

3.1. Sinh cc hon v . . . . . . . . . . . . . . . . . . . . . . . . . . . . . . . . . . . . . . . . . . . . . . . . . . . . .


3.2. Sinh cc t hp . . . . . . . . . . . . . . . . . . . . . . . . . . . . . . . . . . . . . . . . . . . . . . . . . . . . . .
3.3. Nh thc Newton . . . . . . . . . . . . . . . . . . . . . . . . . . . . . . . . . . . . . . . . . . . . . . . . . . . . .

4. Nguyn l Dirichlet . . . . . . . . . . . . . . . . . . . . . . . . . . . . . . . . . . . . . . . . . . . . . . . . . . .

4.1. M u . . . . . . . . . . . . . . . . . . . . . . . . . . . . . . . . . . . . . . . . . . . . . . . . . . . . . . . . . . . .
4.2. Nguyn l Dirichlet tng qut . . . . . . . . . . . . . . . . . . . . . . . . . . . . . . . . . . . . . . . . . .
4.3. Mt vi ng dng th v ca nguyn l Dirichlet . . . . . . . . . . . . . . . . . . . . . . . . . . . .

5. H thc truy hi . . . . . . . . . . . . . . . . . . . . . . . . . . . . . . . . . . . . . . . . . . . . . . . . . . . . . .

5.1. Khi nim v cc v d . . . . . . . . . . . . . . . . . . . . . . . . . . . . . . . . . . . . . . . . . . . . . . . .


5.2. Gii cc h thc truy hi . . . . . . . . . . . . . . . . . . . . . . . . . . . . . . . . . . . . . . . . . . . . . . .

5.3. Quan h chia tr . . . . . . . . . . . . . . . . . . . . . . . . . . . . . . . . . . . . . . . . . . . . . . . . . . .

6. Nguyn l b tr . . . . . . . . . . . . . . . . . . . . . . . . . . . . . . . . . . . . . . . . . . . . . . . . . . . . .

6.1. M u . . . . . . . . . . . . . . . . . . . . . . . . . . . . . . . . . . . . . . . . . . . . . . . . . . . . . . . . . . . .
6.2. Nguyn l b tr . . . . . . . . . . . . . . . . . . . . . . . . . . . . . . . . . . . . . . . . . . . . . . . . . . . . .

Bi tp . . . . . . . . . . . . . . . . . . . . . . . . . . . . . . . . . . . . . . . . . . . . . . . . . . . . . . . . . . . . . . .
Ch-ng 3

44
47
48
49
49
50
51
52
56
56
57
58
60
60
65
70
75
75
77
79
85

th v ng dng

1. Cc khi nim c bn . . . . . . . . . . . . . . . . . . . . . . . . . . . . . . . . . . . . . . . . . . . . . . . . .
1.1. Khi nim v thut ng . . . . . . . . . . . . . . . . . . . . . . . . . . . . . . . . . . . . . . . . . . . . . . . .

85
85

1.3. Mt s dng th n c bit . . . . . . . . . . . . . . . . . . . . . . . . . . . . . . . . . . . . . . . . .

90

1.2. -ng i. Chu trnh. th lin thng . . . . . . . . . . . . . . . . . . . . . . . . . . . . . . . . . . . .

2. Biu din th . . . . . . . . . . . . . . . . . . . . . . . . . . . . . . . . . . . . . . . . . . . . . . . . . . . . . .

2.1. Ma trn k, ma trn trng s. . . . . . . . . . . . . . . . . . . . . . . . . . . . . . . . . . . . . . . . . . . .


2.2. Ma trn lin thuc.

.................................................

88
93
93
94

...........................................

95

3. Cc thut ton tm kim trn th . . . . . . . . . . . . . . . . . . . . . . . . . . . . . . . . . . . . .

97

3.2. Tm kim theo chiu rng . . . . . . . . . . . . . . . . . . . . . . . . . . . . . . . . . . . . . . . . . . . . . .

98

2.3. S ng cu ca cc th.

3.1. Tm kim theo chiu su . . . . . . . . . . . . . . . . . . . . . . . . . . . . . . . . . . . . . . . . . . . . . . .

97

4. th Euler v th Hamilton . . . . . . . . . . . . . . . . . . . . . . . . . . . . . . . . . . . . . . . . .

99

4.2. -ng i v chu trnh Hamilton . . . . . . . . . . . . . . . . . . . . . . . . . . . . . . . . . . . . . . . . .

103

5.1. th c trng s . . . . . . . . . . . . . . . . . . . . . . . . . . . . . . . . . . . . . . . . . . . . . . . . . . . .

106

4.1 -ng i Euler v chu trnh Euler . . . . . . . . . . . . . . . . . . . . . . . . . . . . . . . . . . . . . . . .

5. Bi ton tm -ng i ngn nht . . . . . . . . . . . . . . . . . . . . . . . . . . . . . . . . . . . . . . . .

5.2. Thut ton tm -ng i ngn nht Dijkstra . . . . . . . . . . . . . . . . . . . . . . . . . . . . . . . .

6. th phng . . . . . . . . . . . . . . . . . . . . . . . . . . . . . . . . . . . . . . . . . . . . . . . . . . . . . . . . .

6.1 Khi nim . . . . . . . . . . . . . . . . . . . . . . . . . . . . . . . . . . . . . . . . . . . . . . . . . . . . . . . . . . .


6.2. Cng thc Euler . . . . . . . . . . . . . . . . . . . . . . . . . . . . . . . . . . . . . . . . . . . . . . . . . . . . .
6.3. nh l Kuratowski . . . . . . . . . . . . . . . . . . . . . . . . . . . . . . . . . . . . . . . . . . . . . . . . . .

7. T mu th . . . . . . . . . . . . . . . . . . . . . . . . . . . . . . . . . . . . . . . . . . . . . . . . . . . . . . . .
7.1. M u . . . . . . . . . . . . . . . . . . . . . . . . . . . . . . . . . . . . . . . . . . . . . . . . . . . . . . . . . . . .

99

106
108
110
110
112
113
115
115

7.2. Mt s ng dng ca bi ton t mu th . . . . . . . . . . . . . . . . . . . . . . . . . . . . . .

119

8.1. M u . . . . . . . . . . . . . . . . . . . . . . . . . . . . . . . . . . . . . . . . . . . . . . . . . . . . . . . . . . . .

120

8. Cy v ng dng . . . . . . . . . . . . . . . . . . . . . . . . . . . . . . . . . . . . . . . . . . . . . . . . . . . . .

8.2. Cc ph-ng php duyt cy . . . . . . . . . . . . . . . . . . . . . . . . . . . . . . . . . . . . . . . . . . . .


8.3. Cy v bi ton sp xp . . . . . . . . . . . . . . . . . . . . . . . . . . . . . . . . . . . . . . . . . . . . . . . .
8.4. Cy khung . . . . . . . . . . . . . . . . . . . . . . . . . . . . . . . . . . . . . . . . . . . . . . . . . . . . . . . . . .
8.5. Cy khung nh nht . . . . . . . . . . . . . . . . . . . . . . . . . . . . . . . . . . . . . . . . . . . . . . . . . .

9. Mng. Lung trn mng . . . . . . . . . . . . . . . . . . . . . . . . . . . . . . . . . . . . . . . . . . . . . . .

9.1. Cc khi nim . . . . . . . . . . . . . . . . . . . . . . . . . . . . . . . . . . . . . . . . . . . . . . . . . . . . . . .


9.2 Thut ton tm lung cc i trong mng . . . . . . . . . . . . . . . . . . . . . . . . . . . . . . . . . .

Bi tp . . . . . . . . . . . . . . . . . . . . . . . . . . . . . . . . . . . . . . . . . . . . . . . . . . . . . . . . . . . . . . .
Ch-ng 4
i s Boole v mch t hp

120
124
130
136
140
143
143
145
151
163

1. Khi nim v mch t hp . . . . . . . . . . . . . . . . . . . . . . . . . . . . . . . . . . . . . . . . . . . . . .

163

1.2 Biu thc Boole . . . . . . . . . . . . . . . . . . . . . . . . . . . . . . . . . . . . . . . . . . . . . . . . . . . . . .

165

1.1 Khi nim . . . . . . . . . . . . . . . . . . . . . . . . . . . . . . . . . . . . . . . . . . . . . . . . . . . . . . . . . . .

2. Cc tnh cht ca mch t hp . . . . . . . . . . . . . . . . . . . . . . . . . . . . . . . . . . . . . . . . . .

2.1 Cc tnh cht . . . . . . . . . . . . . . . . . . . . . . . . . . . . . . . . . . . . . . . . . . . . . . . . . . . . . . . .


2.2 Mch t hp t-ng -ng . . . . . . . . . . . . . . . . . . . . . . . . . . . . . . . . . . . . . . . . . . . . . .

3. Hm Boole v vn t hp mch . . . . . . . . . . . . . . . . . . . . . . . . . . . . . . . . . . . . . . .

3.1. i s Boole . . . . . . . . . . . . . . . . . . . . . . . . . . . . . . . . . . . . . . . . . . . . . . . . . . . . . . . .
3.2. Hm Boole v vn tng hp mch . . . . . . . . . . . . . . . . . . . . . . . . . . . . . . . . . . . . .

4. Mt vi ng dng . . . . . . . . . . . . . . . . . . . . . . . . . . . . . . . . . . . . . . . . . . . . . . . . . . . . .

163
167
167
168
170
170
171
174

4.1 B cng . . . . . . . . . . . . . . . . . . . . . . . . . . . . . . . . . . . . . . . . . . . . . . . . . . . . . . . . . . . .

174

Bi tp . . . . . . . . . . . . . . . . . . . . . . . . . . . . . . . . . . . . . . . . . . . . . . . . . .

180

4.2 Cc tiu ho cc mch. Ph-ng php Quine-McCluskey . . . . . . . . . . . . . . . . . . . . . .

Ch-ng 5
Automat, vn phm v ngn ng hnh thc

176

185

1. Mch tun t v my hu hn trng thi . . . . . . . . . . . . . . . . . . . . . . . . . . . . . . . . .

185

1.2. My hu hn trng thi . . . . . . . . . . . . . . . . . . . . . . . . . . . . . . . . . . . . . . . . . . . . . . . .

186

1.1. Mch tun t . . . . . . . . . . . . . . . . . . . . . . . . . . . . . . . . . . . . . . . . . . . . . . . . . . . . . . . .

2. Automat hu hn . . . . . . . . . . . . . . . . . . . . . . . . . . . . . . . . . . . . . . . . . . . . . . . . . . . . .

2.1. Khi nim v nh ngha . . . . . . . . . . . . . . . . . . . . . . . . . . . . . . . . . . . . . . . . . . . . . . .


2.2. Biu din automat hu hn . . . . . . . . . . . . . . . . . . . . . . . . . . . . . . . . . . . . . . . . . . . . .
2.3. Ngn ng on nhn bi automat . . . . . . . . . . . . . . . . . . . . . . . . . . . . . . . . . . . . . . . .
2.4. Automat khng tt nh (nondeterministic automat) . . . . . . . . . . . . . . . . . . . . . . . . .
2.5. Quan h gia automat tt nh v khng tt nh . . . . . . . . . . . . . . . . . . . . . . . . . . . .

3. Vn phm v ngn ng . . . . . . . . . . . . . . . . . . . . . . . . . . . . . . . . . . . . . . . . . . . . . . . .

3.1. Cc khi nim v nh ngha . . . . . . . . . . . . . . . . . . . . . . . . . . . . . . . . . . . . . . . . . . . .


3.2. Vn phm v ngn ng . . . . . . . . . . . . . . . . . . . . . . . . . . . . . . . . . . . . . . . . . . . . . . .
3.3. Phn loi vn phm v ngn ng . . . . . . . . . . . . . . . . . . . . . . . . . . . . . . . . . . . . . . . .
3.4. Mt s tnh cht ca ngn ng . . . . . . . . . . . . . . . . . . . . . . . . . . . . . . . . . . . . . . . . . .
3.5. Tnh qui ngn ng. . . . . . . . . . . . . . . . . . . . . . . . . . . . . . . . . . . . . . . . . . . . . . . . . .

4. Automat hu hn v ngn ng chnh qui . . . . . . . . . . . . . . . . . . . . . . . . . . . . . . . . .

4.1. Quan h gia automat hu hn v ngn ng chnh qui . . . . . . . . . . . . . . . . . . . . . . . .


4.2. Mt s tnh cht ca ngn ng loi 3 . . . . . . . . . . . . . . . . . . . . . . . . . . . . . . . . . . . . .
4.3. Mt s tnh cht ca vn phm phi ng cnh . . . . . . . . . . . . . . . . . . . . . . . . . . . . . . .
4.4. Cc dng chun ca vn phm phi ng cnh . . . . . . . . . . . . . . . . . . . . . . . . . . . . . . .
4.5. Lc l-ng ca vn phm phi ng cnh . . . . . . . . . . . . . . . . . . . . . . . . . . . . . . . . . . . .

5. My Turing (Turing machine) . . . . . . . . . . . . . . . . . . . . . . . . . . . . . . . . . . . . . . . . . .

5.1. M u . . . . . . . . . . . . . . . . . . . . . . . . . . . . . . . . . . . . . . . . . . . . . . . . . . . . . . . . . . . .
5.2. Khi nim v nh ngha . . . . . . . . . . . . . . . . . . . . . . . . . . . . . . . . . . . . . . . . . . . . . . .
5.3. Hm Turing thc hin -c . . . . . . . . . . . . . . . . . . . . . . . . . . . . . . . . . . . . . . . . . . . .
5.4. phc tp ca thut ton . . . . . . . . . . . . . . . . . . . . . . . . . . . . . . . . . . . . . . . . . . . . .

Bi tp. . . . . . . . . . . . . . . . . . . . . . . . . . . . . . . . . . . . . . . . . . . . . . . . . . . . . . . . . . . . . . . .

Ti liu tham kho . . . . . . . . . . . . . . . . . . . . . . . . . . . . . . . . . . . . . . . . . . . . . . . . . . . .

185
188
188
189
189
191
192
197
197
199
201
203
205
207
207
210
212
218
220
221
221
222
224
226
228
234

Ch-ng I.
Nhng khi nim c bn v logic, tp hp v suy lun ton hc
Trong ch-ng ny chng ta nghin cu mt s vn mang tnh cht c s khng ch ca ton
hc ri rc ni ring, m ca c ton hc ni chung. l nhng khi nim c bn v logic
(khi nim mnh , cc php ton trn cc mnh ), tp hp (khi nim v tp hp v cc php
ton trn tp hp) v suy lun ton hc (cc lp lun ton hc c s v cc php chng minh
th-ng dng trong ton hc). Khi nim quan h nh- l mt tp con ca tp tch Decac cng s
-c cp n trong ch-ng.
1. Mnh , mnh c iu kin v s t-ng -ng logic
1.1 Mnh
Logic ton l mn hc nghin cu cc quy lut gia nguyn nhn v h qu, gia gi
thit v kt lun, t rt ra -c nhng quy tc quan trng nht nhn -c nhng nguyn l
ng n -c p dng cho hu ht cc ngnh khoa hc t nhin cng nh- x hi. Mt trong
nhng khi nim quan trng nht ca logic ton l logic mnh , logic ton -c t nn
mng trn i s mnh .
Khi ta ni Hu l mt thnh ph ca Vit Nam th chng ta -a mt khng nh m mi
ng-i u thy ng. Nh-ng khi ta ni 1 2 th ng-i ta li thy ngay ta ni sai.
nh ngha 1.1.1. Mnh l mt khng nh m ta c th bit -c n ng hoc sai. Khng
c mnh va ng va sai. Cc mnh -c k hiu bng cc ch Latinh in A, B, C... Khi
mnh ng th ta ni mnh nhn gi tr ng v vit A : T hay A T , nu mnh B sai
th ta ni B nhn gi tr sai v vit B : F hay B F .
V d 1.1.1. Tt c cc khng nh sau u l cc mnh
1. H Ni l Th ca Vit Nam
2. 2>3
3. 1+3=4
Cc mnh 1, 3 l cc mnh ng cn mnh 2 l mnh sai.
1.2 Cc php ton trn mnh
T cc mnh ban u A, B, C... ng-i ta c th xy dng cc mnh mi vi s
gip ca cc php ton logic tuyn, hi v ph nh sau y.

nh ngha 1.2.1. Gi s A, B l cc mnh . Hi ca A, B l mt mnh -c k hiu l


A B v c l A v B. Mnh A B ng khi c A v B u ng, v sai trong tt c cc
tr-ng hp cn li.
C th biu din hi ca A v B d-i dng bng gi tr chn l sau
A

A B

nh ngha 1.2.2. Gi s A, B l cc mnh . Tuyn ca A, B l mt mnh -c k hiu l


A B v c l A hoc B. Mnh A B sai ch khi c A v B u sai, v ng trong cc
tr-ng hp cn li.
Ta c bng gi tr chn l ca mnh

A B

sau

A B

V d 1.2.1. Nu k hiu A, B, C t-ng ng l cc mnh 1, 2, 3 trong v d 1.1.1

A B l mnh sai v B sai


A B l mnh ng v A ng

C A, C A u l cc mnh ng.
nh ngha 1.2.3. Gi s A l mt mnh , ph nh ca A, k hiu A , l mt mnh nhn
gi tr ng khi A sai v nhn gi tr sai khi A ng.

1.3 Mnh c iu kin v s t-ng -ng logic


nh ngha 1.3.1. Gi s A, B l cc mnh . Mnh c iu kin (cn gi l php suy din
hay php ko theo) A B l mt mnh sai ch khi A ng v B sai, v l mnh ng
trong mi tr-ng hp cn li.
Trong mnh A B ng-i ta gi A l gi thuyt (hay A l nguyn nhn) B l kt lun (hay B
l kt qu). Nh- vy, theo nh ngha, php suy din A B ch b coi l sai nu t gi thuyt
ng suy ra kt lun sai. Ta c bng gi tr chn l
A

A B

Mnh c iu kin A B cn -c c l nu A th B hay B ch nu A. Kt lun B biu


th iu kin cn ca A, cn gi thit A biu th iu kin ca B.
V d 1.3.1. K hiu A l mnh Hm nay l th t-
B l mnh Tam gic vung l tam gic c mt gc bng 90o
C l mnh 1 1 3
Khi , theo nh ngha, A B l mnh ng: Nu hm nay l th t-, th tam gic
vung l tam gic c mt gc bng 90o l mnh ng cho d hm nay c l th t- hay
khng. Cn mnh A C : Hm nay l th t- th 1 1 3 l mnh nhn gi tr ng ch
khi hm nay khng phi l th t-.
nh ngha 1.3.2. Gi s A, B l cc mnh . Khi mnh A t-ng -ng vi B, k hiu
l A B , l mt mnh nhn gi tr ng khi v ch khi A v B c gi tr chn l ging nhau.
Ta c bng gi tr chn l sau ca mnh A B .
A

A B

Ng-i ta cn s dng cc cch gi khc nhau ca mnh A B nh- c A khi v ch khi B,


A l cn v i vi B hay nu A th B v ng-c li.

V d 1.3.2. Nu gi A l mnh Hm nay tri nng


B l mnh Nhit ngoi tri cao hn 300C
th A B s nhn gi tr ng nu khng nh sau l ng: Tri nng th nhit ngoi tri
cao hn 300C v nhit ngoi tri cao hn 300C th tri nng. Vit Nam iu ny r rng
khng ng, v vo ma h n-c ta nhit c th cao hn 300C m tri vn khng nng.
nh ngha 1.3.3. T cc mnh ban u ng-i ta xy dng nn cc mnh mi vi s gip
ca cc php ton logic: hi, tuyn, ph nh, suy din v t-ng -ng. Cc mnh ban u
-c gi l cc mnh s cp, cc mnh mi nhn -c gi l cc cng thc. Cng thc c
gi tr ng vi mi gi tr khc nhau ca cc mnh s cp -c gi l cng thc hng ng
hay nh l (i khi cn gi l lut).
V d 1.3.3. Xt cng thc A B A B
Ta thnh lp bng gi tr chn l ca mnh ny
A

A B

A B

A B A B

Nhn trong bng ny ta thy mnh trn lun nhn gi tr ng vi mi gi tr khc nhau ca
cc mnh s cp A, B cho nn n chnh l mt nh l.
Ghi ch 1.3.1. n gin hn trong cch vit cc cng thc ng-i ta quy -c trt t cc php
ton nh- sau: cc php ton trong ngoc thc hin tr-c, cng thc no c ph nh phi thc
hin n tr-c sau theo th t -u tin nu khng c du ngoc th php hi thc hin tr-c
php tuyn thc hin sau, cui cng mi n cc php ton suy din v t-ng -ng. V d cng
thc

A B C A vi cng thc A B C A ch l mt.

Sau y chng ta dn ra mt s tnh cht quan trng nht ca logic mnh . Cc tnh cht
u c th chng minh -c bng ph-ng php lp bng gi tr chn l t-ng t nh- xt
trong v d 1.3.3.
nh l 1.3.1. Cho A, B, C l cc mnh bt k. Khi ta c:
a) Lut giao hon A B B A ; A B B A
b) Lut kt hp
c) Lut phn phi

A B C A B C ; A B C A B C

A B C A B A C ; A B C A B A C

d) Lut lu ng
e) Lut hp th

A A A; A A A

A A B A ; A A B A

f) Cc lut De Morgan A B A B ; A B A B

g) Lut hai ln ph nh A A
h) Lut chng minh phn chng th nht A B B A
i) Lut chng minh phn chng th hai A B A B
2. Tp hp v cc php ton trn tp hp
2.1 Tp hp, tp con v tch Decac
Tp hp l mt khi nim ton hc khng nh ngha -c. Ng-i ta ch c th m t
tp hp thng qua cc phn t ca n. Tp hp th-ng -c k hiu bng cc ch Latin hoa nhA, B, C... , cn cc phn t ca tp hp -c k hiu bng cc ch Latin th-ng a, b, c,...

a A . Nh- vy gia phn t v tp hp c quan h


ph thuc. Nu a khng phi l phn t ca tp A th ta vit a A hay a A .

Khi a l phn t ca tp A th chng ta vit

k hiu tp A gm c cc phn t no th ta vit cc phn t y gia 2 ngoc nhn


d

A a, b

trong v d ny

l mt tp hp c hai phn t a v b; ta vit

... , v

a A , b A . Cng d dng thy

0 A : 0 khng phi l phn t thuc tp A.

Tp hp c th c hu hn phn t, v d A 1,2,..., n , nh-ng cng c th c v hn phn t,

v d B 1, 2,... x x l s t nhi n . S cc phn t ca mt tp hu hn A k hiu l A .


Nh- vy 1,2,3 3 .

Tp khng c phn t -c gi l tp rng. Tp rng -c k hiu l

nh ngha 2.1.1. Hai tp hp A v B gi l bng nhau nu chng c cng cc phn t.


V d 2.1.1. Xt tp A 1,2 cn B l tp cc nghim ca ph-ng trnh bc hai x 2 3 x 2 0 .
R rng l A = B v ph-ng trnh x 2 3 x 2 0 c hai nghim x1 1, x2 2 .

nh ngha 2.1.2. Nu mi phn t ca tp A u l phn t ca tp B th ta ni tp A l tp con


ca B v vit A B . R rng l vi mi A ta u c A A; A .
Theo nh ngha 2.1.1, hai tp A B khi v ch khi A B v B A .

V d 2.1.2. Tp A 1, 2 , B 1, 2,3, a . y A c 2 phn t 1 v 2 th c hai phn t ny u


c trong B cho nn A l tp con ca B.

p ( A) l tp tt c cc tp con ca A.
V d 2.1.3. Nu A 1,2,3 th p ( A) s gm cc phn t l cc tp con ca A sau:
,a ,b ,c ,a, b ,b, c ,a, c ,a, b, c cho nn p A 8 .
K hiu

p A 2 . iu ny chng ta s chng minh phn sau bng ph-ng php


quy np ton hc. V l do p A 2 khi A n nn ng-i ta cn k hiu p A l 2 v
Nu A n th

gi tp tt c cc tp con ca A l tp lu tha ca A.

nh ngha 2.1.3. Gi s A,B l hai tp hp. Ta xy dng tp mi k hiu l A B , trong


A B a , b a A, b B sao cho a, b c , d khi v ch khi a c, b d .
Tp hp A B -c xc nh theo nh ngha 2.1.3 gi l tch Decac ca A, B. Cn b hai
a, b trong nh ngha l b hai c th t: phn t th nht (to th nht) a A ; phn t th
hai (to th hai) b B .
V

2.1.4.

A 1, 2,3 , B a, b

1, a , 1, b , 2, a , 2, b , 3, a , 3, b .
A A

th

A B

gm

phn

gm c 9 phn t l 1,1 , 1,2 , 1,3 , 2,1 , 2,2 , 2,3 , 3,1 , 3,2 , 3,3 . D dng

nhn thy rng nu A, B l cc tp hu hn th A B A B .

nh ngha 2.1.4. (Tch Decac n - ngi) Gi s A1 , A2 ,..., An l n tp hp no . Tch Decac (n ngi)


ca
A1 , A2 ,..., An
-c
k
hiu
l
A1 A2 ... An ,
vi

A1 A2 ... An a1 , a2 ,..., an a1 A1 , a2 A2 ,..., an An ,

v ch khi ai ai vi mi i 1, 2,..., n .
Cng nh- trn, c th d
A1 A2 ... An A1 A2 ... An .

dng

thy

rng,

a1 , a2 ,..., an a1, a2 ,..., an

nu

cc

tp

Ai

hu

hn

khi

th

2.2 Cc php ton trn tp hp


T mt s tp hp ban u ng-i ta xy dng cc tp mi vi s gip ca cc php
ton trn tp hp. Cc quy tc xy dng nn cc tp mi v thc hin cc php ton trn tp hp
th-ng -c gi l i s tp hp.

nh ngha 2.2.1. Gi s A, B l hai tp hp. A hp B, k hiu A B , l mt tp hp cha tt c


cc phn t ca A, tt c cc phn t ca B. Nh- vy

A B x x A x B hay x A B x A x B
V d 2.2.1. Nu A a, b, c, d l tp cc hc sinh trong mt t tp bng chuyn cn

B a, c, e, f l tp cc hc sinh ca t tp bi th tp cc hc sinh ca t tham gia tp th

thao (bng chuyn hoc bi) s l A B a , b, c, d , e, f .

Ng-i ta th-ng minh ho cc php ton trn tp hp bng gin Venn: Cc tp A,B,... u l
tp con ca mt tp U gi l tp v tr U. Tp v tr -c biu din l mt hnh ch nht, cc
tp A,B,... l cc hnh trn trong U. Gin Venn ca tp A B c dng sau

A B
nh ngha 2.2.2. Gi s A, B l hai tp hp. Giao ca A, B, k hiu A B , l tp gm cc phn
t va l ca A, va l ca B. Nh- vy

A B x x A x B hay x A B x A x B
V d 2.2.2. Cng ly v d A,B t v d 2.2.1 ta c
t tp c hai mn bng chuyn v bi.
Ta c gin Venn ca tp

A B a , c

A B nh- sau:

AB

l tp cc hc sinh trong

nh ngha 2.2.3. Gi s A, B l hai tp hp. Hiu ca A v B, k hiu


gm cc phn t ca A nh-ng khng phi ca B. Nh- vy

A\ B

l mt tp hp

A \ B x x A x B hay x A \ B x A x B
Ta c gin Venn ca

A\ B

l:

A\ B
Khi B A th A \ B th-ng -c vit
tp v tr, ta c nh ngha sau:
nh ngha 2.2.4.

A B . c bit trong tr-ng hp A U , vi U l

U A -c gi l phn b ca A (i vi U) , k hiu l A . Nh- vy


x A x A

Ta c gin Venn ca

A
V d 2.2.3. Ly cc tp A, B t v d 2.2.1 th

A \ B b, d

cn

B \ A e, f .

Sau y chng ta nu ra mt s tnh cht ca cc php ton trn tp hp m chng minh chng
-c -a v cc lut t-ng ng ca logic mnh .
nh l 2.2.1. Gi s U l tp v tr, A, B, C l cc tp con ca U.

A B C A B C ;

a) Lut kt hp

A B C A B C
b) Lut giao hon

A B B A; A B B A

c) Lut phn phi

A B C A B AC ;

A B C A B A C

A A; A U A

d)

Lut ng nht

e)

Lut nut

A U U; A

f)

Lut lm y

A AU; A A

g)

Lut lu ng

A A A; A A A

h)

Lut hp th

A A B A ; A A B A

i)

Lut b

A A

j)

Lut 0, 1

U; U

k)

Lut De Morgan

A B A B; A B A B

Chng minh. Nh- ta ni trn tt c cc lut trong nh l 2.2.1 u -c chng minh bng
ph-ng php -a v cc lut t-ng ng ca logic mnh . V d ta chng minh lut hp th

A A B A

nh- sau.

Theo nh ngha 2.2.1, 2.2.2 th

x A A B x A x A B

x A x A x B

Theo lut hp th th hai trong logic mnh , iu sau cng t-ng -ng vi
Nh- vy ta chng minh -c
ny c ngha l

x A.

x A A B x A , theo nh ngha 2.1.1 iu

A A B A .

C th nh ngha t-ng t cho hp v giao ca mt h bt k cc tp hp.


3. L-ng t v v t
3.1 Hm mnh
Cc mnh -c xt trong phn u ca ch-ng l nhng mnh m chng ta c th xc
nh ngay gi tr ca chng ng hoc sai. Trong mc ny chng ta s xem xt mt loi mnh
khc, mnh m gi tr ca n ph thuc vo cc gi tr khc nhau ly t mt tp no . V d
khng nh x l mt s nguyn ln hn 5 l mt loi khng nh m khi thay x bng mt gi
tr nguyn c th no ta s -c mt mnh , chng hn vi x 2 ta c mnh 2 ln
hn 5 vit bng k hiu ton hc l 2 5 , y l mnh sai, cn vi x bng 6 ta s -c
mnh 6 5 l mnh ng. Loi mnh ph thuc vo cc tham s ly gi tr t mt
tp no -c gi l hm mnh . Ni mt cch chnh xc ta c nh ngha sau:
nh ngha 3.1.1. Hm P x1 , x2 ,..., xn xc nh trn tp A -c gi l hm mnh n - ngi
nu khi thay x1 a1 , x2 a2 ,..., xn an vi a1 , a2 ,..., an A , ta nhn -c mt mnh .

Khi n 1 , hm 1- ngi P x th-ng gi n gin l hm mnh .

V d 3.1.1. x y l mt hm mnh 2- ngi xc nh trn tp s nguyn Z . Tht vy, khi ta


gn x m , y n l cc s nguyn c th no ta s -c mnh m n .
3.2 V t
Xt khng nh vi mi s t nhin n ta u c n 5 . y l mt khng nh sai v ta d dng
tm thy mt s t nhin, chng hn 1 5 . Tuy nhin, d dng thy rng, khng nh tn ti s
t nhin n sao cho n 5 li l mt mnh ng. Cc mnh nh- nu y -c xy dng
t hm mnh n 5 xc nh trn tp s t nhin vi s gip ca cc ton t -c gi l
ton t chung (l-ng t chung) n , c l vi mi n v ton t ring (l-ng t ring) n c l
tn ti n.

nh ngha 3.2.1. Gi s P x l mt hm mnh xc nh trn tp A. x P x (c l vi

mi x P x ) l mt mnh , n nhn gi tr ng khi v ch khi vi phn t bt k a A ta c

P a T .

x P x (c l tn ti x P x ) l mt mnh , n nhn gi tr ng khi v ch khi tn ti


a A P a T . Cc ton t x , x gi l cc l-ng t.

x -c gi l l-ng t chung, x -c gi l l-ng t ring. Mnh c cha cc l-ng t


-c gi l v t.
Hon ton t-ng t nh- trong nh ngha 3.2.1 ta c th nhn -c cc v t xy dng t cc
hm mnh n - ngi ( n 1 ). Chng hn xyP x, y hay xyP x, y ... Khi hm mnh

P x1 , x2 ,..., xn xc nh trn tp A th ta ni v t t-ng ng xc nh trn tp A.

V d 3.2.2. xy x y l mt v t (mnh ) xc nh trn tp s nguyn. Mnh ny


khng nh vi mi s nguyn x m tn ti s nguyn y n m n . R rng y l mt
mnh ng.
3.3 Ph nh ca v t
Ta xt ph nh ca v t x x 5 trn tp s nguyn Z , tc l x x 5 . y l mt mnh

nhn gi tr ng khi v ch khi x x 5 sai. Theo nh ngha 3.2.1 th iu ny c ngha l

x x 5 hay x x 5 vy l x x 5 x x 5 x x 5 (D dng thy mnh

vi mi x nguyn x 5 l sai) cho nn mnh x x 5 l mnh sai. Ta c nh l sau:

nh l 3.3.1. Nu P x l hm mnh xc nh trn tp A th cc khng nh sau l hng


ng:
a) xP x xP x
b) xP x xP x
Chng minh. Ta chng minh khng nh a). Mnh xP x nhn gi tr ng khi v ch khi

xP x nhn gi tr sai. Theo nh ngha 3.2.1 iu ny c ngha l tn ti x a thuc A

P a sai hay P a ng, tc l xP x ng. Do vy xP x xP x l mt mnh


hng ng hay l mt nh l. Vic chng minh khng nh b) l hon ton t-ng t.
V d 3.3.1. V t ca nh ngha dy s an c gii hn a l

0 K n n K : an a .

Hm mnh n K : an a

xc nh trn tp s t nhin n K , K N . Theo ch

thch 3.3.2 th v t ca nh ngha dy an khng c gii hn a s l:


0 K n n K : an a 0

T nh l 1.3.1 ta c h qu sau:
H qu 3.3.1. Cc cng thc sau l cc nh l

a) x P x Q x x P x Q x

b) x P x Q x x P x Q x

Ch thch 3.3.1. Cc l-ng t cng loi trong v t ca hm mnh n - ngi ( n 1 ) c tnh


giao hon cn cc l-ng t khc loi khng giao hon -c cho nhau. C th thy iu qua v
d sau:
V d 3.3.2. Gi s P x , y l hm mnh xc nh trn tp s nguyn Z . V t xy x y

l mt mnh ng (v d 3.2.2), nh-ng mnh yx x y l mnh sai v n khng

nh tn ti mt s nguyn y m0 m0 nh hn hoc bng tt c cc s nguyn x n khc,


tc l m0 l s nguyn nh nht. iu ny l v l v tp s nguyn khng c s nguyn nh
nht.
Ch thch 3.3..2. C th tng qut ho nh l 3.3.1 cho hm mnh P x1 , x2 ,..., xn n - ngi

vi n 1 : Ph nh ca v t nhn -c bng cch thay l-ng t thnh l-ng t khc loi v

hm mnh P x1 , x2 ,..., xn thnh ph nh ca n P x1 , x2 ,..., xn .


4. Quan h
4.1 Khi nim v tnh cht

Quan h gia hai tp theo ngn ngha trc gic -c hiu hiu l mi quan h gia mt s phn

t ca tp ny vi mt s phn t ca tp khc. Chng hn, nu ly A tp cc sinh vin no v


B l tp cc mn hc m h ang theo hcth ta c th ni sinh vin a c quan h vi mn hc a

nu sinh vin a theo hc mn a. Hon ton t-ng t nh- vy nu ly A l tp mt s ng-i no

v quan h m ta quan tm l quan h h hng chng hn th ta ni anh a v c b thuc quan


h ny nu a v b c h vi nhau.

nh ngha 4.1.1. Gi s X, Y l hai tp hp. Quan h hai ngi R t X n Y -c nh ngha l


tp con ca tch Decac X Y , tc l R X Y . Trong tr-ng hp X Y th ng-i ta ni
R X X l quan h hai ngi trong X.

x X y Y x, y R -c gi l tp xc nh ca quan h R. Cn tp
y Y x X x, y R -c gi l tp gi tr ca quan h R.

Tp

x, y R ng-i ta th-ng hay vit


thc: , khi v ch khi .

xc nh

xRy

bt ch-c theo quan h trong tp s

Nu quan h hai ngi biu din d-i dng mt bng gm 2 ct th ct th nht gm cc phn t
ca min xc nh, ct th hai - min gi tr.
V d 4.1.1. Gi s X 2,3, 4,5 , Y 1,3, 4,5, 6, 7,9 . Ta xc nh quan h hai ngi

x, y R x y :

x l -c s ca y (hay y chia ht cho x). D dng thy khi

R 2,4, 2,6, 3,3, 3,6 , 3,9, 4,4, 5,5. Hay biu din d-i dng bng ta c
A

Trong cc phn t trn cng mt hng nm trong quan h R. Nhn vo bng ny cc phn t
nm trn ct th nht 2,3,4,5l tp xc nh ca quan h R cn cc phn t trn ct th hai

3,4,6,5,9 l tp gi tr ca R.

a, b R nu a b .
R 2, 2 , 2,3 , 2, 4 , 2,5 , 3,3 , 3, 4 , 3,5 , 4, 4 , 4,5 , 5,5 .

X 2,3,4,5 .

4.1.2.

Ta

ni

nh ngha 4.1.2. Quan h R trong tp X -c gi l c tnh phn x nu

x X

Ta

x, x R vi mi

Quan h trong v d 4.1.2 l quan h c tnh phn x.


V d 4.1.3. Gi

X 2,3,4,5 . Ta ni a, b R

R 2, 2 , 2, 4 , 3,3 , 4, 4 , 5,5

v ch khi

a b . R rng l

v R l quan h c tnh phn x.

V d 4.1.4. Gi s X R tp cc s thc.
Ta ni

x, y R

nu

y x2 .

Tp R chnh l th ca parabol
v d nh-

y x 2 . R rng y l quan h khng c tnh phn thn, v

3,3 R l do x 2 32 x 3 .

nh ngha 4.1.6. Quan h R trong tp X -c gi l c tnh bc cu nu vi mi


t

x, y R v y, z R suy ra x, z R .

x, y, z X

Cc quan h trong cc v d 4.1.2, 4.1.3 u c tnh bc cu, trong khi quan h trong v d
4.1.4 th khng c tnh bc cu v

2, 4 R, 4,16 R

nh-ng

2,16 R v 22 16 .

nh ngha 4.1.7. Quan h R trong tp X -c gi l c tnh i xng nu vi mi

x, y R suy ra y, x R .

x, y X

Quan h trong cc v d 4.1.2, 4.1.3 u khng phi l cc quan h c tnh i xng v trong v
d 4.1.2 ta c

2 3 nh-ng 3 2 ; Trong v d 4.1.3 c 2,4 R

nh-ng

4,2 R .

nh ngha 4.1.8. Quan h R trong tp X -c gi l c tnh phn i xng nu vi mi

x, y X t x, y R v y, x R
x, y R v x y th y, x R .

suy ra y = x hay ni mt cch t-ng -ng l t

Quan h trong cc v d 4.1.2, 4.1.3 u l cc quan h c tnh phn i xng l do trong tp s


thc x y v y x th x = y.

T-ng t nh- vy trong tp s t nhin nu m chia ht cho n v n chia ht cho m th m = n. R


rng l quan h trong v d 4.1.4 khng c tnh phn i xng.
nh ngha 4.1.9. Quan h R trong tp X -c gi l quan h th t tng phn trong X nu n l
quan h c tnh phn x, phn i xng v bc cu.
T cc nhn xt trn ta thy cc quan h trong cc v d 4.1.2, 4.1.3 l cc quan h th t tng
phn trn cc tp X t-ng ng. Nu R l mt quan h th t tng phn trong tp X th khi

x, y R

ng-i ta th-ng ni x, y so snh -c vi nhau. Khi

th ni x, y khng so snh -c vi nhau.

x, y R

y, x R

V d v quan h chia ht trn tp s t nhin l quan h th t tng phn v khng phi hai s t
nhin no cng so snh -c vi nhau: 2 khng phi l -c ca 3 v 3 cng khng l -c ca 2.
Tp X vi quan h th t tng phn R m vi hai phn t bt k x, y X th hoc xRy hoc

yRx

-c gi l tp c quan h th t tuyn tnh (hay th t ton b).

nh ngha 4.1.10. Gi s R l quan h t X vo Y. Quan h ng-c ca R, k hiu l


h t Y vo X -c xc nh nh- sau

yR 1 x xRy
R 1

R 1 l quan

hay t-ng -ng

y, x x, y R

4.2 Ma trn quan h


Gi s R l mt quan h t tp X vo tp Y. Ta c th biu din quan h R d-i dng mt ma trn
khng mt M R gi l ma trn quan h R nh- sau:
Cc phn t ca tp X -c sp xp theo mt trt t no
phn t ca Y theo mt trt t ca n
Ma trn

j l

MR

y1 , y2 ,..., yn

x1 , x2 ,..., xn

trn mt hng. Gi s

trn mt ct, cn cc

X n, Y m.

c m hng n ct (c (m,n)) vi cc phn t mij sao cho phn t nm trn hng i, ct

mij 1 nu xi , y j R , mij 0

V d 4.2.1. Gi s

nu

x , y R .
i

X 2,3,4 , Y 5,6,7,8 ,

x, y R x y s c ma trn M R

nh- sau

quan h

R X Y

xc nh

5 6 7 8
2 0 1 0 1

3 0 1 0 0
4 0 0 0 1
hay khi bit trt t sp xp cc phn t ca tp X (theo ct) tp Y theo hng (v d y theo
th t tng dn) th ng-i ta ch vit

0 1 0 1

M R 0 1 0 0
0 0 0 1

Nh- vy l vi mi quan h R X Y t tp hu hn X vo tp hu hn Y ta c th xy dng


-c mt ma trn quan h (chnh xc n s sp xp th t ca tp X v tp Y). Ng-c li vi
mi ma trn khng mt M ta c mt quan h R t X vo Y.
Tht vy nu

mij 1 th ta t xi , y j R , nu mij 0

th

x , y R . y cng nhi

tr-c ta phi coi cc trt t sp xp phn t ca X v Y l c nh chn tr-c. Nu


l quan h trong tp X c

X n

th

MR

l ma trn vung cp n.

D dng nhn thy quan h R trong tp X c tnh i xng khi v ch khi


xng. R c tnh phn i xng khi v ch khi

MR

R X X

l ma trn i

mij 1 m ji 0

mij 0 m ji 1 m ji 0
Quan h R trong tp X c tnh phn x khi v ch khi tt c cc phn t trn -ng cho

mij 1.

4.3 Quan h t-ng -ng, lp t-ng -ng


nh ngha 4.3.1. Quan h R trong tp X -c gi l quan h t-ng -ng nu n c tnh phn
x, i xng v bc cu. Nu hai phn t x, y nm trong quan h t-ng -ng R th ta ni x, y
t-ng -ng vi nhau.
V d 4.3.1. Ta c tp X gm 10 qu cu trong c 3 qu cu trng

t1 , t 2 , t3 , 3 qu cu xanh:
nh nh- sau a, b X ,

x1 , x2 , x3 v 4 qu cu d1 , d 2 , d 3 . Quan h R -c xc
a, b R khi v ch khi a, b cng mu. D dng nhn thy R l mt quan h t-ng -ng:

a, a R , qu cu ch c mt mu

a, b R th b, a R : qu cu a cng mu vi b th b cng mu

vi a.

a, b R , b, c R th a, c R : nu a, b cng mu v b, c cng mu th r rng a, c

cng mu.

V d 4.3.2. Gi s X l tp cc t ting Vit, gi


ch ci trong xu).

Gi R l quan h trong tp X m

a, b R

l a

l chiu di ca xu

khi v ch khi

cng d dng nhn thy R l quan h t-ng -ng trong tp X.

a X

(s cc

l a l b . Nh- trn, y

nh ngha 4.3.2. H S cc tp con khc rng ca X -c gi l phn hoch ca X, nu mi


phn t x ca X thuc mt v ch mt phn t S. Cc phn t S th-ng -c gi l cc lp ca
phn hoch S.
Nh- vy h S
v

X
iI

X i iI , X i X , X i

l mt phn hoch ca X nu

Xi X j

V d 4.3.1 c th tng qut thnh nh l sau:


nh l 4.3.1. Nu S
theo quy tc

x, y X i

X i iI

x, y R

l mt phn hoch ca tp X. Xc nh quan h

khi v ch khi tn ti

iI

R X X

c hai phn t x, y u thuc Xi tc l

th R l mt quan h t-ng -ng trong tp X.

Chng minh. R rng l vi mi

x X

th

x, x R v

X X i , do tn ti i I
iI

x X i : quan h R c tnh phn x. Ngoi ra nu xRy tc l x, y X i ; yRz tc l


y , z X i th r rng x, z X i : nh- vy quan h R c tnh bc cu. Cui cng xRy tc l
x, y X i th hin nhin yRx cng vi ngha nh- th y, x X i vy l quan h R li c

tnh i xng. nh l -c chng minh xong.

n 0, n N * , m Z
n

V d 4.3.3. Xt tp

r1

thuc cng mt lp nu

m1
n1

r2

m2
n2

m1n2 m2n1

cc s hu t. Hai s hu t
v vit

r1 r2 : nh- vy mi lp

ca
gm tt c cc s hu t bng nhau. Ta c th chng minh -c cc lp ny to thnh
cc lp ca mt phn hoch s hu t.

Tht vy, r rng l hp cc lp ny cho ta

r Q

v mi

thuc vo 1 lp no . Hai

lp c chung 1 phn t th s trng nhau, tht vy nu lp A v B m c chung phn t

r1 A c r1 r2 B nn r1 B
r3 r2 A cho nn B A vy l A B .

th ly

A B.

tc l

Ng-c li ly

r2

r3 B

m2
n2

th

Nh- vy l cc lp khc nhau th ri nhau, ta nhn -c phn hoch ca cc s hu t gm cc


lp l cc s hu t bng nhau. Ng-c li vi nh l 4.3.1 ta c:
nh l 4.3.2. Gi s

R X X

l mt quan h t-ng -ng trong X. Xc nh cc lp

cc tp con ca X nh- sau: Vi mi

x x X l mt phn hoch ca X.

Chng minh. Th nht, tp

x X

x x X

nhau th ri nhau. Tht vy nu a v


c

x a X

cho ta mi

x x

b c chung phn t

xRa .

Khi d h S

S . Th hai, hai lp khc

x X th vi mi y a ta
yRb c ngha l y b . Nh- vy a b .

yRa , aRx , xRb nn theo tnh bc cu


i vai tr a , b cho nhau ta -c b a . T cc l lun trn ta c a b . Nh-

vy hai lp c chung mt phn t th chng phi trng nhau; iu sau cng c ngha l hai lp
khc nhau th ri nhau. nh l -c chng minh.
T hai nh l 4.3.1 v 4.3.2 ta thy khi nim quan h t-ng -ng trong tp X v phn hoch
ca X l t-ng -ng vi nhau.
V d 4.3.4. Cho quan h t-ng -ng

1,1 , 3, 3 , 5,5 , 1,3 , 3,1 , 1, 5 ,

5,1 , 3,5 , 5,3 , 2, 2 , 4, 4 , 2, 4 , 4, 2

X 1,2,3,4,5 , X 1,3,5 2,4


hay cho cc lp t-ng -ng 1 3 5 1,3,5 ; 2 4 2, 4 .

cng c ngha l cho mt phn hoch ca tp

V d 4.3.5. Ly

X 1,2,...,7 . Quan h t-ng -ng R -c xc nh nh- sau: xRy

v ch khi x v y ng d- theo modun 3 ( x

chia ht cho 3 hay cn vit

3 x y ). Khi chia

1, 2,3
l 3 6 ,

cc s nguyn cho 3 c th c cc s d- 0, 1, 2, v th c 3 lp t-ng -ng l

3 3,6 , 1 1,4,7 , 2 2,5


1 4 7 , 2 5 .

trong

nh-

khi

vy

4.4 Quan h n - ngi. C s d liu quan h*


nh ngha 4.4.1. Gi s

A1 , A2 ,..., An

A1 , A2 ,..., An l mt tp
R A1 A2 ... An .

l n tp hp. Quan h n ngi xc nh trn cc tp

con R ca tch Decac

Cc tp

A1 , A2 ,..., An gi l min xc nh ca quan


a1, a2 ,..., an R l b n - to (n - thnh phn).

4.4.1.

Nu

ly

A1 A2 ... An .

h, s n gi l bc ca quan h; gi

A A1 A2 A3 1,2,3,4

a, b, c R A A A l quan h a b c th

R 1, 2,3 , 1,3, 4 , 2,3, 4 v 3,1,2 R

Quan h n ngi c th biu din d-i dng bng.

Nh- vy l

Phn ny c th b qua m khng nh h-ng n vic tip thu cc ch-ng sau.

quan

V d 4.4.2.
gio vin
S in thoi

Tn

B mn

Tui

1010001

Vui

Ton

40

1230401

37

2123456

Khoi

Ho

49

6106333

Ch

Sinh

27

6665555

Khoi

Ton

40

Hng th nht Gio vin ch tn ca quan h lit k d-i bng. Bng ny gm c 4 ct n biu
din quan h 4 - ngi vi cc thnh phn: s in thoi, tn gio vin, b mn v tui.
Mt c s d liu gm cc bn ghi l cc b n thnh phn n thnh phn ny ng vi n to
ca quan h n - ngi.
Trong v d 4.4.2 quan h 4 ngi cho trn bng ng vi 5 bn ghi mi bn ghi c 4 thnh phn
l
(1010001, Vui, Ton, 40)
(1230401, V, L, 37)
(2123456, Khoi, Ho, 49)
(6106333, Ch, Sinh, 27)
(6665555, Khoi, Ton, 40)
Mi ct ca bng quan h n - ngi ng vi mt tiu ch (ca quan h hay ca c s d liu). Mt
tiu ch ca quan h n - ngi gi l kho c bn nu cc gi tr ca n xc nh hon ton cc b
ca quan h ny. Ni mt cch khc trong min ny mi phn t ch ng vi mt b duy nht
ca quan h R.
vi d 4.4.2 th tiu ch s in thoi l kho c bn ca quan h. Nhng t hp cc tiu ch
m cc gi tr ca chng xc nh duy nht cc b ca quan h R gi l kho phc hp. Trong v
d 4.4.2 th (Tn, b mn) chnh l mt kho phc hp v trong quan h ny khng c 2 gio
vin no trng c tn ln b mn.

Mt c s d liu phi p ng -c cc i hi ca ng-i s dng khi ng-i ta cn ly nhng


thng tin no ra.
nh ngha 4.4.2. Ton t chn - Select chn t cc hng ca quan h ra cc hng m tiu ch
cn quan tm nhn mt gi tr nht nh.
V d 4.4.3. Ton t chn Gio vin [Tn = Khoi] chn ra cc hng no trn gio vin l Khoi
l
(2123456, Khoi, Ho, 49)
(6665555, Khoi, Ton, 40)
nh ngha 4.4.3. Ton t chiu

Pi1i2 ...im

ch quan tm n cc ct mang s th t

qua cc ct khc. Ngoi ra nu c nhiu b cng

i1i2...im th ch gi li mt.

i1i2 ...im , b

V d 4.4.4. Ton t chiu P(Tn, b mn) cho ta


(Vui, Ton)
(V, L)
(Khoi, Ho)
(Ch, Sinh)
(Khoi, Ton)
th ton t chiu P(Tn) ch cho ta (Vui), (V), (Khoi), (Ch).
nh ngha 4.4.4. Ton t lin kt - join thc hin trn 2 quan h
tr s ca cc tiu ch ca hai quan h

quan h

R1 , cc hng ca quan h R2

R1 , R2 . Khi thc hin ton t ny ta xem cc hng ca


xem c hng no c hai tiu ch chn trng nhau th

vit sang mt bng mi vi b sung thm nhng ct mi vo


V d 4.4.5. Gi

R1

l quan h 4 ngi trong v d 4.4.2 cn

t ct ca

R1
R2

i tuyn

1010001

Bng chuyn

6106333

Bng

R2 .

l quan h 2 ngi

Th thao
S in thoi

R1 , R2 . Khi chng c chung

th ton t lin kt

R1 , R2

s cho ta quan h mi 5 ngi l

S in thoi

Tn

B mn

Tui

i tuyn

1010001

Vui

Ton

40

Bng chuyn

6106333

Ch

Sinh

27

Bng

nh ngha 4.4.5. Ton t t hp

R1 , R2

thc hin t hai quan h

mi c m n p tiu ch (ct) trong m l bc ca


(thnh phn) trng nhau.
V d nu

R1

l quan h trong v d 4.4.2, cn

R2

R1 , R2 thnh lp quan h
R1 , n l bc ca R2 , p l s tiu ch

l quan h

M khoa
B mn

th kt qu ca ton t t hp
S in thoi

M khoa

Ton

K12

K11

Ho

K13

Sinh

K14

R1, R2 s l quan h cho trong bng sau


Tn

B mn

Tui

M khoa

1010001

Vui

Ton

40

K12

1230401

37

K11

2123456

Khoi

Ho

49

K13

6106333

Ch

Sinh

27

K14

6665555

Khoi

Ton

40

K12

5. Suy lun ton hc


5.1 Cc ph-ng php chng minh
Theo nh ngha 1.3.1, trong php suy din A B (mnh c iu kin) A -c gi l gi
thit, B -c gi l kt lun. Php suy din A B b coi l sai ch trong tr-ng hp khi m A
ng cn B sai. Qu trnh thc hin php suy din -c gi l lp lun hay lun chng. Lun
chng -c coi l ng nu chng ta nhn -c cng thc hng ng hay nh l .V d nh-

A A B B l mt nh l v vi mi gi tr ca A, B

cng thc l cng thc

hng ng. Trong lp lun th cng thc hng ng y -c vit nh- sau:

A, A B
B
Khi dng k hiu ny ta mun nhn mnh ti cc kha cnh ca lp lun: Gi thit A v
A B -c vit trn gch ngang; d-i du gch ngang vit kt lun B, k hiu thay cho
vy th trong lp lun. Quy tc suy din theo lp lun trn -c gi l lut tch ri.
V d 5.1.1: Gi s B l mnh nu n chia ht cho 3 th n chia ht cho 9, cn A l mnh
2

n = 6. Khi , theo lut tch ri n 36 s chia ht cho 9.


2

Sau y l mt s quy tc suy din quan trng chng ta s t t-ng ng cc quy tc ny vi cc


nh lut t-ng ng trong logic mnh (hng ng)

A
t-ng ng vi hng ng A A B
A B

i)

Lut cng

ii)

Lut rt gn

iii)

Lut tch ri

A, A B
B
iv)

AB
A

t-ng ng vi hng ng

t-ng ng vi hng ng

A A B B

Tam on lun gi nh

A B

BC
B C
v)

A B A

t-ng ng hng ng

Tam on lun tuyn

A B B C A C

A B
A
B

t-ng ng vi hng ng

A B A B

V d 5.1.2. Lp lun Nu hm nay tri nng ti s i do cng vin c c s l lut tch ri


iii)
V d 5.1.3. Lp lun Nu hm nay tri nng ti s i chi cng vin. Nu hm nay ti i chi
ngoi cng vin th ngy mai ti phi nh hc bi. Vy th, nu hm nay tri nng ngy mai
ti s nh hc bi c c s l lut tam on gi nh nu ta k hiu A l mnh Hm nay
tri nng B l mnh Ti s i chi cn C l mnh Ngy mai ti s nh hc bi.
C mt s ph-ng php chng minh c bn l: Chng minh trc tip, chng minh gin tip,
chng minh bng phn chng, chng minh ring r, chng minh bng ph-ng php quay vng.
chng minh mnh A B ng ta c th ch ra rng nu A ng th B phi ng.
Ph-ng php chng minh ny -c gi l chng minh trc tip.
V d 5.1.4. chng minh nu n chia ht cho 3 th n2 chia ht cho 9 ta c th p dng ph-ng
php chng minh trc tip. Gi s n chia ht cho 3 tc l n 3k , k z Khi r rng l

n 2 9k 2 chia ht cho 9.

chng minh

A B ng

ta c

th chng minh

B A

mnh

A B B A l hng ng. Cch chng minh ny gi l chng minh gin tip.

V d 5.1.5. chng minh 3n 1 l s chn nu n l ta c th chng minh bng ph-ng php


gin tip: gi s n l s chn khi n 2k v 3n 1 3.2 k 1 6k 1 l s l.
Chng minh bng phn chng da trn mnh hng ng sau

A B A B C C

Nh- vy chng minh A suy ra B ta c th chng minh bng phn chng rng gi s A ng

nh-ng B li sai, khi ta s nhn -c mu thun ( C C vi C l mnh tu ). y l


ph-ng php th-ng hay gp trong cc l lun khoa hc, nht l trong ton hc.

V d 5.1.6. chng minh Mt dy s nu c gii hn th ch c mt ta c th p dng


ph-ng php chng minh bng phn chng.
Nu dy s c gii hn m li c 2 gii hn khc nhau th ta chn -c 2 ln cn ca 2 im ny
ri nhau trong khi vi mi s th t ln th tt c cc phn t ca dy u phi ri vo mi
ln cn ny mu thun vi giao ca cc ln cn ny bng trng. Mu thun chnh ny l do
chng ta gi thit dy c hai gii hn khc nhau. Vy dy c gii hn th ch c mt -c
chng minh.

chng minh cc mnh

A1 , A2 ,..., An t-ng -ng vi nhau ta c th chng minh bng


A1 A2 ... An1 An A1 . V c s ca ph-ng php

ph-ng php quay vng


quay vng ny chnh l mnh hng ng

A1 A2 ... An A1 A2 ... An A1
y cng l mt ph-ng php th-ng xuyn -c p dng trong cc chng minh t-ng -ng
ca nhiu mnh ton hc.
5.2 Quy np ton hc
chng minh mt mnh no

ph-ng php quy np ton hc.


nh l 5.2.1. Khng nh
i)

P 1

ii) T

P n

P n

ph thuc vo s t nhin n ng-i ta c th p dng

ng cho mi s t nhin

n 1 nu

ng.

P n

ng suy ra

P n 1 ng vi mi s t nhin n.

Chng minh. Ta chng minh bng ph-ng php phn chng, gi s rng cho d c i), ii) trong
nh l 5.2.1 nh-ng vn m
* P m sai.

Gi

M m N P m F

Khi tp M c phn t nh nht

.
m0

v v

P 1 T

m0 1 m0 ,v m0 2 nn m0 1 N
P m0 F nn P m0 1 T v theo

P m0 1 1 P m0 T

mu thun vi chn

nn

m0 1 tc l m0 2 .

m0

l s t nhin nh nht

gi thit quy np ta li phi c

m0 . nh l -c chng minh.

B-c i) trong chng minh bng ph-ng php quy np ton hc gi l b-c c s, b-c
ii) gi l b-c quy np.
D dng thy -c ph-ng php chng minh bng quy np ton hc trong nh l 5.2.1. t-ng
-ng vi
nh l 5.2.2. Gi s khng nh

Pn

ph thuc vo cc s t nhin n tho mn

i)

P 1

ng.

k n 1 suy ra P n 1 ng.

ii) T P k ng vi mi
Khi

P n

ng vi mi s t nhin.

V th tu theo tng bi ton m ng-i ta s dng quy np ton hc theo nh l 5.2.1 hay m
rng ca n l 5.2.2.
T-ng t nh- trn, chng minh mnh P(n) ng cho mi s t nhin n n 0 , n0 1 ta c
th thc hin theo hai b-c, trong b-c c s ch cn kim tra P( n0 ) ng.

V d 5.2.1. By gi ta chng minh bng ph-ng php quy np ton hc rng tp A vi


c 2 tp con.
n

A n

Tht vy

n 1, A a

i)

Vi

ii)

Gi s vi mi tp A m
c n+1 phn t th

Tht vy

B B1 b

c 2 tp con

A n

p B 2

trong

th

n 1

a c s quy np ng.

p A 2

ta chng minh khi nu B

B1 n, b B1 .

S tp con ca B gm s cc tp con ca B1 l 2n v sau c mi tp con ca B1 cho thm b


vo ta -c 1 tp con ca B.
Nh- vy c thm 2n tp con na cho nn

p B 2

2 n 2.2n 2 n 1

l iu phi chng minh.


5.3 quy v ng dng
Trong thc t chng ta th-ng gp nhng tr-ng hp phi nh ngha mt dy v hn
cc phn t. Thay v vit tt c cc phn t ny ra ng-i ta phi ch ra mt quy lut xc nh
phn t bt k ca dy ny.

Chng hn n ! l tch ca n s t nhin u tin. Chng ta khng th vit ht -c thm ch cc


giai tha ca 100 s u v ci gi l bng n giai tha vi n ln ln th n ! ln ln theo lu tha

nn .

Nh-ng chng ta li c nh ngha

F n n! nh- sau

F 0 1
F n 1 n 1 F n
R rng l khi p dng nhiu ln ta s c

F n 1 (n 1) F (n) (n 1)nF ( n 1) ( n 1)n 2.1 (n 1)!


Nh- vy ta c th ni mt i t-ng l quy nu n bao gm chnh n nh- mt b phn hoc
n -c nh ngha d-i dng ca chnh n. K thut xc nh i t-ng y -c gi l k thut
quy.
Ta c th s dng quy nh ngha cc dy s, hm s v tp hp. V d nh ngha mt
hm xc nh cc s nguyn khng m ta cho
1. Gi tr ca hm ti

n0

2. Cng thc tnh gi tr ca n ti s nguyn n t cc gi tr ca n ti cc s nguyn nh hn.


Xt mt v d c th sau
V d 5.3.1. Xt hm
thy

a0 1

F n an

trong n l s thc khc 0 v n l nguyn khng m. Ta

a n 1 a.a n a.a.a n1 a
.a
....
a . Vy ta c th -a ra nh ngha quy

ca hm F n a nh- sau

n ln

1.

F 0 1

2.

F n 1 a.F n

Ta cng c th nh ngha mt tp hp da vo quy bng cch, u tin ta -a ra tp xut


pht, sau l quy tc to cc phn t mi t cc phn t bit ca tp. Nhng tp -c m t
bng cch nh- vy -c gi l cc tp nh ngha tt, cc nh l v chng c th chng minh
bng cch s dng nh ngha quy ca chng.
V d 5.3.2. Gi s tp A -c nh ngha quy nh- sau

1.

3 A

2.

x y A

nu

x A v y A .

Chng minh rng A l tp cc s nguyn d-ng chia ht cho 3.


Gi B l tp cc s nguyn d-ng chia ht cho 3. chng minh A = B ta phi chng minh
v B A .

A B

Chng minh

A B : Gi P n

3 A v
ng A B .

vy, v

P 1

ng (theo nh

3n A , ta chng minh P n 1 ng. Tht


3n A nn theo nh ngha ta c 3 3n 3 n 1 A P n 1

ngha quy ca A). Gi s

Pn

l mnh 3n thuc tp A. Ta c
ng, tc l

Chng minh B A . Ta c 3 chia ht cho 3 l iu hin nhin nn 3 B . Tip theo ta chng


minh mi phn t ca A -c sinh ra theo qui tc th hai ca nh ngha cng thuc B. Gi s x,
y l hai phn t ca A, cng l hai phn t ca B. Theo nh ngha ca A th x y A v v x
v y u chia ht cho 3 nn
-c chng minh.

x y

cng chia ht cho 3

x y B . Vy B A

. nh l

nh ngha quy cn -c dng khi nghin cu cc lp ngn ng.


V d 5.3.3. Ng-i ta c th nh ngha quy tp cc xu nh- sau
Gi s l tp cc xu trn b hu hn ch ci
nh- sau
*

. Khi *

-c nh ngha bng quy

* , trong l mt xu rng (khng c phn t no).


wx *

nu

w *

x .

* . Phn sau khng nh mt xu mi to


vi mt xu ca * cng thuc * .

Phn u ca nh ngha ni rng xu rng thuc


nn bng cch ghp mt k t ca

Bi tp
Mnh
1. Trong cc cu sau y cu no l mnh ? Xc nh gi tr chn l ca cc mnh
?
a) 2 + 3 = 5
b) 5 + 7 = 10
c) x + 2 = 11
d)

x y y x

vi mi cp s thc x v y.

e) Khng -c i qua.
f)

By gi l my gi ?

g)

4 x 5

h)

x y y z

i)

x 1 5

nu

nu

xz

x 1

2. Tm ph nh ca cc mnh sau
a) Hm nay l th nm.
b) Khng c nhim H Ni.
c) 2 + 1 = 3
d) Ma h Hi Phng nng v nng.
3. Cho A v B l hai mnh
A : Ti mua v s tun ny.
B : Ti trng gii c c 200 triu ng vo hm th su.
Din t cc mnh sau bng cc cu thng th-ng:

a)

b)

A B

c)

AB

d)

A B

e)

A B

f)

A A B

4. Cho A, B l hai mnh


A : Nhit d-i khng.
B : Tuyt ri.
Dng A, B v cc php ton logic vit cc mnh sau
a) Nhit d-i khng v tuyt ri.
b) Nhit d-i khng nh-ng khng c tuyt ri
c) Nhit khng d-i khng v khng c tuyt ri.
d) Nu nhit d-i khng th c tuyt ri.
e) Hoc nhit d-i khng hoc c tuyt ri nh-ng s khng c tuyt ri nu nhit
d-i khng.
f)

Nhit d-i khng l iu kin cn v c tuyt ri.

5. Lp bng gi tr chn l i vi cc mnh phc hp sau


a)

AB

b)

A B B

c)

A B B A

d)

A B A B

A B A B

e)

A B A B

f)
Tp hp

6. Lit k cc phn t ca cc tp hp sau

x x l s thc sao cho x 1


2

a)

x x l s nguy n d-ng nh hn 12

b)

x x l b nh ph-ng ca 1 s nguy n v x 100

c)

x x l s nguy n sao cho x

d)

7. Xc nh xem mi cp tp hp sau y c bng nhau khng ?

1,3,3,3,5,5,5,5,5,5

a)

b)

c) v

1,1

5,3,1

8. Xc nh cc mnh sau ng hay sai ?


a.

x x

b.

x x

e.

x x

c.

x x

f.

d.

9. Cho A l tp cc sinh vin sng cch xa tr-ng trong vng bn knh mt km, v B l
tp hp cc sinh vin ang trn -ng ti lp. Hy m t cc sinh vin thuc mt
trong cc tp hp sau
a.

AB

c.

A B

b.

AB

d.

BA

10. Gi s A l tp sinh vin nm th hai tr-ng v B l tp cc sinh vin ang hc


mtnTon ri rc tr-ng. Hy biu din cc tp sau y qua A v B
a) Tp cc sinh vin nm th hai hc Ton ri rc tr-ng.
b) Tp cc sinh vin nm th hai khng hc Ton ri rc.
c) Tp hp cc sinh vin tr-ng hoc l nm th hai, hoc ang hc Ton ri rc.
d) Tp hp cc sinh vin tr-ng, hoc khng l sinh vin nm th hai, hoc khng
hc Ton ri rc.
11. Anh ch c th ni g v cc tp A v B nu cc ng thc sau l ng ?
a.

AB A

c.

AB A

b.

AB A

d.

AB B A

e.

AB B A

12. Cho A a, b, c, B x, y v C 0,1 . Tm


a.

A B C

c.

C B A

b.

C A B

d.

B B B

L-ng t v v t
13. Cho

P x, y

l cu x hc mn y, vi khng gian ca x l tp hp tt c sinh

vin trong lp , v khng gian ca y l tp hp tt c cc mn hc tr-ng . Hy din


t cc l-ng t sau thnh cu thng th-ng:
a)

x y P x, y

b)

x y P x, y

c)

y x P x, y

d)

x y P x, y

e)

y x P x, y

x y P x, y

f)
14. Cho

L x, y

l cu x yu y, vi khng gian ca c x v y l tp hp mi ng-i

trn th gii. Hy dng hm mnh v cc l-ng t din t cc cu sau:


a) Mi ng-i u yu Lan.
b) Mi ng-i u yu mt ai .
c) C mt ng-i m tt c mi ng-i u yu.
d) Khng c ai yu tt c mi ng-i.
e) C mt ng-i m Lan khng yu.
f)

C mt ng-i m khng ai yu.

g) Mi ng-i u yu chnh mnh.


h) C ng hai ng-i m Lan yu.
i)

C mt ng-i no khng yu ai ngoi chnh mnh.

15. Chng minh rng


chn l.

x P x Q x

xP x xQ x

c cng gi tr

Quan h
16. Lit k cc cp -c sp trong quan h R t

B 0,1,2,3

trong

a, b R

nu v ch nu

a. a = b

b. a + b = 4

c. a > b

d. a b

e. UCLN(a,b) = 1

f. BCNN(a,b) = 2

17. i vi mi quan h trn tp

1,2,3,4 cho d-i y hy xc nh xem n c l

phn x, i xng, phn i xng v bc cu khng ?


a)

A 0,1, 2,3, 4

2,2 , 2,3 , 2,4 , 3,2 , 3,3 , 3,4

b)
c)
d)
e)
f)

1,1 , 1, 2 , 2,1 , 2, 2 , 3,3 , 4, 4

2,4 , 4,2

1, 2 , 2,3 , 3, 4

1,1 , 2, 2 , 3,3 , 4, 4

1,3 , 1,4 , 2,3 , 2,4 , 3,1 , 3,4

18. Xc nh xem quan h R trn tp cc s nguyn c l phn x, i xng, phn i


xng v/hoc bc cu khng, vi

x, y R nu v ch nu

a) x y
b) x l bi s ca y
c) x y2
19. Cc quan h no trong quan h trn tp mi ng-i cho d-i y l cc quan h t-ng
-ng. Xc nh cc tnh cht ca mt quan h t-ng -ng m cc quan h khc
khng c
a)
b)
c)
d)

a, b a v b cng tui

a, b a v b c cng b m

a, b a v b gp nhau

a, b a v b ni cng mt th ting

20. Trong s cc tp hp ca cc tp con sau, tp hp no l phn hoch ca


1, 2,3, 4,5,6 ?

a) {1,2}, {2,3,4}, {4,5,6}


b) {1}, {2,3,6}, {4}, {5}
c) {2,4,6}, {1,3,5}

d) {1,4,5}, {2,6}
21. Trong s cc tp hp ca cc tp con sau, tp hp no l phn hoch ca tp cc s
nguyn ?
a) Tp con cc s chn v tp con cc s l.
b) Tp con cc s nguyn d-ng v tp con cc s nguyn m.
c) Tp con cc s nguyn chia ht cho 3, tp con cc s nguyn chia ht cho 3 cn
d- 1, tp con cc s nguyn chia ht cho 3 cn d- 2.
d) Tp con cc s nguyn nh hn 100, tp con cc s nguyn c tr tuyt i
khng v-t qu 100 v tp con cc s nguyn ln hn 100.
Suy lun ton hc
22. Quy tc suy lun no -c dng trong mi lp lun sau
a) An gii mn Ton. Do An gii mn Ton hoc mn Tin.
b) Hoa gii mn Ton v Tin. Do vy Hoa gii mn Ton.
c) Nu tri m-a th b bi s ng ca. Tri m-a do b bi ng ca.
d) Nu hm nay tuyt ri th tr-ng i hc s ng ca. Hm nay tr-ng i hc
khng ng ca. Do vy hm nay khng c tuyt ri.
e) Nu ti i bi th ti s phi nng -c nhiu. Nu ti phi nng nhiu th ti rm
nng. Do nu ti i bi th ti rm nng.
f)

Nu ti thc c m lm bi tp ny, th ti c th tr li -c tt c cc bi
tp. Nu ti tr li -c tt c cc bi tp th ti s hiu -c ti liu ny. Do
nu ti c m lm bi tp ny th ti s hiu -c ti liu ny.

23. Chng minh rng bnh ph-ng ca mt s chn l mt s chn bng ph-ng php
a) Chng minh trc tip.
b) Chng minh gin tip.
c) Chng minh phn chng.
24. Hy tm cng thc tnh tng n s nguyn chn u tin.
25. Chng minh bng quy np rng

3.5 3.5 ... 3.5


2

3 5n1 1
4

vi n l s nguyn khng m.
26. Chng minh bng quy np rng

1.2.3 2.3.4 ... n n 1 n 2

n n 1 n 2 n 3
4

vi mi n nguyn d-ng.
27. My tr tin t ng ngn hng ch c loi tin 20 v loi tin 50 nghn ng. My
c th tr -c cc khon tin l bao nhiu, nu s l-ng cc t giy bc thuc hai
loi trn l khng hn ch. Hy chng minh cu tr li bng quy np ton hc.
28. Hy tm

f 1 , f 2 , f 3

f 4 , nu f n

vi

f 0 1 v vi n 0,1,2...

a)

f n 1 f n 2

b)

f n 1 3. f n

c)

f n 1 2

d)

f n 1 f n f n 1

e)

f n 1 2. f n

f)

f n

f n 1 3

f n
3

29. Cho nh ngha quy ca


a) Tp cc s nguyn l.
b) Tp cc lu tha nguyn d-ng ca 3.
c) Tp cc a thc vi h s nguyn.

-c nh ngha bng quy

30. Cho F l hm sao cho


nh ngha quy ca

F n

F n .

l tng ca n s nguyn d-ng u tin. Hy -a ra

Ch-ng II.
Cc ph-ng php m v nguyn l Dirichlet
L thuyt t hp l mt phn quan trng ca ton ri rc, i t-ng chnh ca n l cc

cu hnh t hp, cc ph-ng php la chn phn t hoc b cc phn t trong tp hp hu hn

theo cc cch khc nhau. Cc kt qu ca n l c s xy dng nhiu thut ton c hiu qu


trong cc ch-ng trnh ng dng nh- cc thut ton vt cn, cc thut ton sinh phn t mi,
cc thut ton la chn ph-ng n ti -u, v..v

Ni dung ca ch-ng l nghin cu v gii quyt mt s bi ton c bn ca l thuyt t

hp, l cc bi ton m, cc bi ton v s tn ti v cc ph-ng php biu din cc cu


hnh t hp.

1. Cc nguyn l m c bn
Trong phn ny ta s cp n hai nguyn l m c bn nht. Vic m cc i t-ng

u c th quy v vic xt s l-ng cc phn t thuc mt tp hp no .

Cho A l mt tp hu hn cc phn t , ta k hiu s l-ng cc phn t ca A l N(A).

1.1. Nguyn l cng


Gi s c k cng vic khng th lm ng thi. Cng vic th i (i=1,2,k) c th lm

bng ni cch khc nhau. Khi s c n1+n2 ++nk cch lm mt trong k cng vic .

V d 1.1.1. Gi s B mn Ton c 17 cn b v B mn Khoa hc my tnh c 13 cn

b (mi cn b ch bin ch mt b mn!). Hi c bao nhiu cch chn mt i biu i d hi


ngh khoa hc trong s cc cn b ca hai b mn trn?

Gii: C 17 cch khc nhau chn mt cn b ca B mn Ton (vic th nht) v 13


cch khc nhau chn mt cn b ca B mn Khoa hc my tnh (vic th hai). R rng l
hai cng vic khng th tin hnh ng thi. Theo nguyn l cng ta c 17 + 13 = 30 cch
chn v i biu ny.
Xem xt theo cch khc, nu ta gi A l tp cc cn b B mn Ton v B l tp cc cn
b B mn Khoa hc my tnh. Hai tp l hai tp ri nhau (khng c phn t chung) v N(A)

= 17 v N(B) = 13. S cch chn i biu d hi ngh trong s cc cn b ca hai b mn chnh


l vic chn mt phn t bt k ca tp AB. Ta c N(AB) = N(A) + N(B) = 30.
V d 1.1.2. Mt thi trc nghim c th -c chn t mt trong ba b thi c lp

t-ng ng c 23, 17 v 29 . C bao nhiu cch chn khc nhau?

Gii: C 23 cch chn thi t danh sch th nht, 17 cch t danh sch th hai v 29
cch t danh sch th 3. V vy c 23 + 17 + 29 = 69 cch thi trc nghim.
K hiu ba b thi l A, B, C. T-ng t nh- v d 1.1.1, ta c s cch chn thi l

N(ABC) = N(A) + N(B) + N(C) = 23 + 17 + 29 = 69.

Bng ngn ng tp hp, nguyn l cng -c pht biu nh- sau:

Nguyn l cng. Cho A1,A2,., An l cc tp hu hn, khng giao nhau tng i mt.

Khi

N ( Ai ) N ( Ai )
n

i 1

i 1

1.2 Nguyn l nhn


Gi s mt cng vic no c th tch thnh k phn on. Phn on th i c th thc

hin bng ni cch sau khi phn on 1,2,, i-1 hon thnh. Khi s c n1n2 nk cch khc
nhau thc hin cng vic .

V d 1.2.1. K hiu ging -ng ca mt tr-ng i hc bt u bng mt trong cc ch

ci A, B, C, D, E, F v mt s ch s l s nguyn d-ng khng v-t qu 50. Hi nhiu nht c


bao nhiu ging -ng -c k hiu khc nhau?

Gii: Th tc ghi k hiu cho mt ging -ng gm hai vic, gn mt trong 6 ch ci A,


B, C, D, E, F v sau gn mt trong 50 s nguyn d-ng 1, 2,50. Nguyn l nhn ch ra
rng c 6 x 50 = 300 cch khc nhau k hiu cho mt ging -ng. Nh- vy nhiu nht c
th c 300 ging -ng -c k hiu khc nhau.
Nu gi tp ch ci nu trn l R v tp cc s th t cn nh s l S, ta c l N(R) = 6,
N(S) = 50. Nh- vy mi k hiu ging -ng s gm 2 phn: phn ch ci l mt phn t bt k
a R v phn s l mt phn t b S, tc l mt phn t (a,b) A x B - tch -cc ca hai
tp R v S. Ta c N(R x S) = N(R) x N(S) = 6 x 50 = 300.
V d 1.2.2. Mt sinh vin c 5 chic o s mi khc mu , 3 ci qun khc mu , 2 i

giy khc kiu. Nu mi ngy sinh vin mc mt kiu khc nhau, th sau bao nhiu ngy th
sinh vin s phi lp li cch trang phc ngoi?

Gii: Cc cch trang phc khc nhau khc nhau mt trong ba thnh phn o s mi, qun

v giy. chn o c 5 cch, chn qun c 3 cch v chn giy c 2 cch, nh- vy c tt c 5

x 3 x 2 = 30 cch. Ngha l ti a sau 30 ngy sinh vin s phi lp li cch trang phc ca
mnh.

T-ng t nh- v d 1.2.1 ta biu din tp A l tp o s mi, tp Q l tp qun, tp G l tp

giy, khi mt b trang phc gm o, qun v giy l mt phn t (a, q, g) ca tp tch -

cc A x Q x G. Vy tng s cch chn trang phc ngoi ca sinh vin l


N(AX QX G)=N(A) X N(Q) X N(G) = 5 X 3 X 2 = 30.

V d 1.2.3. C bao nhiu xu nh phn c di bng 8?


Gii: Mi mt trong 8 bt ca xu nh phn c th chn bng hai cch hoc bng 0 hoc

bng 1. Bi vy, quy tc nhn cho thy c tng cng 28 = 256 xu nh phn khc nhau c di
bng 8.

T-ng t, ta c tt c cc dy nh phn c di n l 2n.


V d 1.2.4. C nhiu nht bao nhiu bin ng k xe my trn 50 phn khi ca thnh

ph H Ni nu mi bin c ni dung v d nh- 29 H3-3907 s 29 l k hiu dnh cho H Ni,


tip l mt trong 26 ch ci, sau ch ci gm s ln hn 0 v nh 10, bn s cui bt k.

Gii: C tt c 26 cch chn ch ci; 9 cch chn cho ch s tip theo. V th theo quy tc

nhn, nhiu nht c 26 X 9 X 10 X 10 X 10 X 10 = 2 340 000 bin ng k xe.


t.

V d 1.2.5. ng dng v d 1.2.3 hy m tt c cc tp con ca tp A c N(A) = n phn


Gii: Gi s tp A = {a1, a2, an}. Ta c th biu din mi tp con ca A t-ng ng 1-

1 vi dy nh phn c di n trong nu ai th phn t th i ca dy nh phn t-ng ng

bng 1. T suy ra s cc tp con ca tp A c n phn t ng bng dy nh phn c di n


v bng 2n.

V d 1.2.6. Mi ng-i s dng h thng my tnh u c mt khu di t su n tm k

t, trong mi k t l mt ch Latinh vit hoa hay mt ch s. Mi mt khu phi cha t


nht mt ch s. Hi c bao nhiu mt khu?

Gii: Gi P l tng s mt khu c th v P6 , P7 , P8 t-ng ng l s mt khu di 6, 7, 8

k t. Theo quy tc cng ta c: P = P6 + P7 + P8. By gi chng ta s tnh P6, P7, P8. Tnh trc
tip P6 s rt kh. tm P6 d hn ta tnh s cc xu di 6 k t l cc ch in hoa hoc ch s,

ri bt i s cc xu di 6 k t l cc ch in hoa v khng cha ch s no. Theo quy tc nhn


s cc xu di 6 k t l 366 v s cc xu khng cha cc ch s l 266 . V vy:
P6 = 366 - 266 = 2 176 782 336 - 308 915 776 = 1 867 866 560
Hon ton t-ng t, ta c:
P7 = 367 - 267 = 78 364 164 096 - 8 031 810 176 = 70 332 353 920

P8 = 368 - 268 = 2 821 109 907 456 - 208 827 064 576 = 2 612 483 063 360
Cui cng ta -c: P = P6 + P7 + P8 = 2 684 483 063 360
Tng qut, bng ngn ng tp hp, nguyn l nhn c th pht biu nh- sau:

Nguyn l nhn. Cho A1,A2,., An l cc tp hu hn bt k, khi


N ( A1A2 ....An ) N ( Ai )
n

i 1

2. Mt s bi ton m c bn: Hon v, t hp, chnh hp


2.1. Chnh hp lp
Chnh hp lp chp k ca tp n phn t l mt cch sp xp c th t k phn t ly t tp

gm n phn t cho, mi phn t c th -c ly lp li.

V d 2.1.1. Tp A = {1, 2, 3, 4, 5} cc b (1, 1, 2) ; (1, 2, 1) ; (2, 3, 5) v (2, 3, 2 ) l cc

chnh hp lp chp 3 t 5 phn t.

V d 2.1.2. T tp = { a, b, c } c th t -c bao nhiu tn bin c di 4 k t?


Gii: Mi tn bin c 4 k t -c chn t tp l mt b 4 phn t -c ly t tp

nn n s l mt phn t ca tch ecac xxx, vy theo nguyn l nhn ta c s tn bin

c 4 k t -c chn t l N(xxx) = N()xN()xN()xN() = 3x3x3x3 = 81.


l:

T-ng t v d 2.1.2, c th -a ra cng thc tnh chnh hp lp chp k t tp n phn t

Ank n k
V d 2.1.3. Cc dy nh phn c di n l mt chnh hp lp chp n t hai phn t {0,

1}. Vy theo cng thc chnh hp lp chp n t 2 phn t l : 2n.

V d 2.1.4. B mn Khoa hc my tnh c 3 thnh vin l Anh, Bnh, Dng k hiu l (A,

B, D). C bao nhiu cch sp xp gio vin dy hai mn hc trong mt bui?

Gii: Mi cch sp xp gio vin l chnh hp lp chp 2 t 3 phn t. Theo cng thc

nu trn ta c s ph-ng n xp l 32 = 9. C th cc ph-ng n l: (A,A) (B,B) (D,D) (A,B)


(A,D) (B,D) (B,A) (D,A) (D,B).

2.2. Chnh hp khng lp


Chnh hp khng lp chp k t n phn t (gi tt l chnh hp chp k) l mt cch sp xp

c th t k phn t ca tp n phn t, mi phn t khng -c ly lp li.

V d 2.2.1. Tp A = {1, 2, 3, 4, 5} cc b (2, 3, 5); (2, 5, 3) l cc chnh hp khng lp

chp 3 t 5 phn t, cn cc b (1, 1, 2) ; (1, 2, 1) ; v (2, 3, 2) khng phi l chnh hp khng


lp chp 3 t 5 phn t, nh-ng mt khc li l chnh hp lp chp 3 t 5 phn t.
6} ?

V d 2.2.2. C bao nhiu s c 4 ch s khc nhau -c chn t cc s sau {1,3, 4, 5, 7,


Gii: K hiu s c bn ch s l a1a2a3a4. Ta c 6 kh nng chn s a1, sau khi chn

a1 ta ch c 5 kh nng chn ch s a2, sau cn 4 kh nng chn ch s a3 v cui cng ch

cn 3 kh nng chn ch s a4 . Vy tt c cc s c 4 ch s khc nhau c th c l S = 6 x 5 x


4 x 3 = 360.
t l:

T-ng t v d 2.2.2 ta c th -a ra cng thc tnh chnh hp khng lp chp k t n phn

Pnk n .( n 1)( n 2 ).....( n k 1)

n!
( n k )!

V d 2.2.3. C bn ng-i thi u c vua l Bnh, C-ng, Dng, Kin tranh hai v tr nht,

nh, hy tnh xc sut C-ng ot gii nht ?

Gii : Gi tp k th l = {B, C, D, K }. Mi kh nng phn chia gii l mt chnh hp

khng lp chp 2 t 4 phn t. Vy theo cng thc ta c S = 4.3.= 12. Cc kh nng l:


(B, C) (B, D) (B, K) (C B ) (C , D) (C, K) (D, B ) (D,C) (D, K) (K, B ) (K, C) (K,D)

Cc ph-ng n m C-ng ot gii ta c th chn nh- sau ghp C-ng vi mt trong 3

ng-i cn li, s ph-ng n l 3: (C B) (C, D) (C, K). Vy xc sut C-ng ot gii nht l

P = 3/ 12 = 25 %.

2.3. Hon v
Hon v ca n phn t khc nhau l mt cch sp xp c th t n phn t .
V d 2.3.1. C bn ng-i r nhu i chp nh l Anh, Bc, Cc, D-ng. Hy tnh xem c

bao nhiu kiu nh chp m tt c bn ng-i ng thnh mt hng ?

Gii : K hiu tn ca bn ng-i l {A, B, C, D}. u tin ta c 4 kh nng chn ng-i

ng bn tri cng, sau khi chn ng-i ng tri ta c 3 kh nng chn ng-i ng k theo, tip

ch c th chn -c 2 kh nng ng-i cn li, nh- vy theo nguyn l nhn ta c s kiu

nh c th chp khc nhau v v tr ng l 4.3.2.1= 4! = 24. Ta c th lit k cc kiu nh l:


(A, B, C, D) (A, B, D, C) (A, C, B, D) (A, C, D, B) (A, D, B, C) (A, D, C, B)
(B, A, C, D) (B, A, D, C) (B, C, A, D) (B, C, D, A) (B, D, A, C) (B, D, C, A)

(C, B, A, D) (C, B, D, A) (C, A, B, D) (C, A, D, B) (C, D, B, A) (C, D, A, B)


(D, B, C, A) (D, B, A, C) (D, C, B, A) (D, C, A, B) (D, A, B, C) (D, A, C, B)

T-ng t v d trn ta c th -a ra cng thc tnh s hon v Pn ca n phn t l:

Pn n .( n 1)( n 2 )..... 1 n !

Cng thc trn c th nhn -c t nh ngha mi hon v l mt chnh hp khng lp n

t n phn t, tc l khi k = n ta c

Pnn Pn

n!
n!
( n n )!

V d 2.3.2. C 5 phong b gi v 5 b-u nh gi cho 5 ng-i, c bao nhiu ph-ng n la

chn gi b-u nh cho 5 ng-i trn ?

Gii : Mi ph-ng n la chn b-u nh b phong b gi l mt hon v t 5 phn t,

vy s ph-ng n s l 5! = 5.4.3.2.1 = 120.

V d 2.3.3. Gi s mt ng-i bn hng rong nh i bn hng ti tm thnh ph. Ch ta

bt u cuc hnh trnh ca mnh ti mt thnh ph no , nh-ng c th n by thnh ph kia

theo th t bt k no m ch ta mun. Hi ch ta c th i qua tt c cc thnh ph ny theo bao


nhiu l trnh khc nhau.

Gii: V thnh ph u tin -c xc nh, cn by thnh ph cn li c th i theo th

t tu , nn s l trnh khc nhau chnh l s hon v ca tp gm 7 phn t. Do c 7! = 5


040 cch ng-i bn hng chn hnh trnh ca mnh. Nu mun tm l trnh ngn nht th ch

ta phi tnh tng qung -ng cn i cho mi l trnh c th, tc l tng cng phi tnh cho 5
040 l trnh.

2.4. T hp
T hp chp k t n phn t l cch chn khng phn bit th t k phn t ly t tp n phn

t cho, mi phn t khng -c ly lp li.

V d 2.4.1. Vi tp A = {1, 2, 3, 4, 5} th cc b (1, 2, 3 ), (1, 2, 4) l cc t hp chp 3 t

5 phn t, cn cc b (1, 1, 2 ), (2, 3, 2 ) khng phi l t hp chp 3 t 5 phn t cho. Theo


nh ngha hai b (2, 3, 5 ), (3, 5, 2) ch -c tnh l mt t hp chp 3.

V d 2.4.2. C 5 sinh vin l Anh, Bc, Cc, D-ng, Giang ca khoa CNTT. Ch nhim

khoa mun chn thnh lp mt t 3 ng-i thc hin mt ti khoa hc. Hi c bao nhiu

ph-ng n thnh lp t 3 ng-i ?

Gii: Ta k hiu tn ca cc sinh vin l A, B, C, D, G. Cc ph-ng n thnh lp t c th

l t hp chp 3 ca tp 5 phn t . C th l:
(A, B, C) (A, B, D) (A, B, G) (A, C, D) (A, D, G)
(B, C, D) (B, C, G) (B, D, G) (C, D, G) (A, C, G)

Nh- vy c tt c 10 ph-ng n la chn lp t 3 ng-i.


Ta nhn thy mi t hp chp k t n phn t, sau khi hon v ta -c k! chnh hp lp

khc nhau, tc l k! ln s t hp chp k t n phn t bng tng s chnh hp khng lp chp k t


n phn t . Do vy, nu k hiu Cnk l s t hp chp k ca n phn t, ta c cng thc sau:

Cnk

n!
( n k )! k !

T cng thc trn c th thy ngay:

Cnk

n!
n!

Cnn k
(n k )!k ! k !(n k )!

Tc l s t hp chp k ca n phn t bng s t hp chp n-k t n phn t.


V d 2.4.3. C 12 i bng tham d gii chuyn nghip quc gia, cc i thi u vng

trn mt l-t. Hi c bao nhiu trn u -c t chc ?

Gii : Mi trn u l mt cp 2 i -c chn t 12 i cho, khng k n th t v

phi khc nhau, vy s trn u l t hp chp 2 t 12 : 12!/(10!.2!) = 66 trn

V d 2.4.4. Tp c n phn t, C bao nhiu tp con c ng k phn t ?


Gii: Mi tp con ca c ng k phn t l mt t hp chp k t n phn t, vy s tp

con c ng k phn t l t hp chp k t n phn t


Cnk

n!
(n k )!k!

2.5. T hp lp
V d 2.5.1. Gi s trong mt a qu c to, cam, l mi loi c t nht 4 qu. Tnh s

cch ly 4 qu bt k t a ny nu khng phn bit th t cc qu -c chn v cc qu cng


loi l ging nh- nhau.

Gii: Mi ph-ng n chn 4 qu t 3 loi qu theo yu cu nu trn -c gi l mt t


hp lp chp 4 t tp 3 phn t to, cam, l. C tt c 15 cch chn 4 qu nh- sau:
4 to

4 cam

4 l

3 to, 1 cam

3 to, 1 l

3 cam, 1 to

3 cam, 1 l

3 l, 1 to

3 l, 1 cam

2 to, 2 cam

2 to, 2 l

2 cam, 2 l

2 to, 1 cam, 1 l

2 cam, 1 to, 1 l

2 l, 1 to, 1 cam

Trong tr-ng hp tng qut: T hp lp chp k t n phn t l mt b gm k phn t


khng phn bit th t, mi phn t c th -c ly lp li t n phn t cho.
V d 2.5.2. C bao nhiu cch chn 4 t giy bc t mt kt ng tin gm nhng t 2, 5,

10, 20, 50 v t 100 ngn ng, nu th t m cc t tin -c chn ra l khng quan trng, cc

t tin cng loi l khng phn bit v mi loi c t nht 4 t.

Gii: V ta khng k ti th t chn t tin, v v ta chn ng 4 ln, mi ln ly ra mt t

trong 6 loi tin nn bi ton ny lin quan ti vic tnh t hp lp chp 4 t 6 phn t. Chng ta
s dng k thut m cc t hp lp gii bi ton ny. Ta c th coi kt ng tin c 6 ngn,

cc ngn -c phn cch bng cc vch ngn, mi ngn ng mt loi tin nh- minh ho trn
hnh 1.

2000

5000

10 000

20 000

50 000

100 000

Hnh 1. Kt 6 ngn cha 6 loi tin


Vic chn 4 t tin t-ng ng vi vic t 4 vt nh du vo 6 ngn cha 6 loi tin .

Hnh 2a, 2b minh ho s t-ng ng ny cho 2 cch chn 4 t tin, trong 5 vch ngn -c
biu din bng 5 thanh ng, 4 t tin biu din bng 4 ngi sao.
1 t
2000

5000

3 t
10 000

20 000

50 000

100 000

***

Hnh 2a. Chn 1 t 5 ngn v 3 t 20 ngn ng.


1 t

1 t

2000

5000

2 t
10 000

20 000

* *

**

50 000

100 000

Hnh 2b. Chn 1 t 2 ngn, 1 t 5 ngn v 2 t 50 ngn ng.

Nh- vy, mi cch chn 4 t tin t-ng ng vi mt cch sp xp 5 thanh ng v 4 ngi

sao. Do , s cch chn 4 t tin bng s cch chn cc v tr cho 4 ngi sao vo 9 v tr c th,
tc l bng:

C115

9!
126
4!5!

Bng l lun t-ng t, ta c th chng minh: S t hp lp chp k t n phn t bng

Rnk C nk k 1

( n k 1)!
k !n!

V d 2.5.3. Ph-ng trnh sau c bao nhiu nghim nguyn khng m:


x1 + x2 + x3 = 11
Gii: Chng ta nhn thy mi nghim ca ph-ng trnh ng vi mt cch chn 11 phn t

t mt tp c 3 loi, sao cho c x1 phn t loi 1, x2 phn t loi 2, x3 phn t loi 3 -c chn.
V vy s nghim bng s t hp chp 11 t tp gm 3 phn t. Theo cng thc trn ta c:
11
11
2
R311 C 11
3 1 C 13 C 13

13.12
78
1.2

Chng ta cng c th tm -c s nghim ca ph-ng trnh ny khi cc n s chu nhng

rng buc no . V d, tm s nghim nguyn ca ph-ng trnh tho mn iu kin x1 1, x2

2, x3 3. D thy mt nghim ca ph-ng trnh tho mn nhng iu kin ny ng vi mt cch

chn 11 phn t gm x1 phn t loi 1, x2 phn t loi 2, x3 phn t loi 3, trong c t nht

mt phn t loi 1, hai phn t loi 2 v ba phn t loi 3. V th, tr-c tin ta chn 1 phn t
loi 1, 2 phn t loi 2 v 3 phn t loi 3, sau chn thm 5 phn t na thuc 3 loi. Do vy,

trong tr-ng hp ny ta c s nghim l:

R 35 C 55 3 1 C 57 C 72

7.6
=21
1 .2

2.6. Hon v ca tp hp c cc phn t ging nhau


Trong mt s bi ton m, mt s phn t c th ging nhau. Khi cn phi cn thn,

trnh m chng hn mt ln. r hn chng ta xt v d sau.

V d 2.6.1 C th nhn -c bao nhiu xu khc nhau bng cch sp xp li cc ch ci

ca t SUCCESS.

Gii: T SUCCESS c 7 ch ci, trong c 3 ch S, 1 ch U, 2 ch C v 1 ch E. Do

vy cu tr li khng phi l s hon v ca 7 ch ci -c. xc nh s xu khc nhau c th


to ra -c ta nhn thy c C(7,3) cch chn 3 ch cho 3 ch S, cn li 4 ch trng. Khi c

C(4,1) cch chn mt ch cho mt ch U, cn li 3 ch trng. C th t ch C bng C(3,2)

cch, v C(1,1) cch t ch E vo xu. Theo quy tc nhn, s cc xu khc nhau c th to


-c l:

C 73 C 41 C 32 C 11

7 ! 4 !3 !1!
7!

420
3 !4 !1!3 ! 2 !1!1!0 ! 3 !2 !1!1!0 !

Bng cch l lun t-ng t nh- trong v d trn chng ta c th -a ra cng thc tnh s

hon v ca n phn t trong c n1 phn t nh- nhau thuc loi 1, n2 phn t nh- nhau thuc

loi 2, , v nk phn t nh- nhau thuc loi k, bng

n!
n 1 ! n 2 !... n k !

Chng minh. xc nh s hon v tr-c tin chng ta nhn thy c C n cch gi n1 ch


n1

cho n1 phn t loi 1 , cn li n - n 1 ch trng. Sau c C nn2n1 cch t n 2 phn t loi 2 vo

hon v, cn li n - n 1 - n2 ch trng. Tip tc t cc phn t loi 3, loi 4, , loi k - 1 vo


ch trng trong hon v. Cui cng c Cnk n1 n2 .. nk 1
n

cch t nk phn t loi k vo hon v.

Theo quy tc nhn tt c cc hon v c th l:

C nn 1 C n 2 n ... C nn k n
n

.... n k 1

n!
( n n 1 )!
...
n 1 ! ( n n 1 )! n 2 ! ( n n 1 n 2 )!

( n n 1 ... n k 1 )!
n!

n k !0 !
n 1 ! n 2 !... n k !
2.7. Phn b cc vt vo trong hp

Mt s bi ton m c th gii bng cch lit k nhng cch t cc i t-ng khc nhau

vo trong nhng hp khc nhau. Chng ta xt v d sau:

V d 2.7.1. C bao nhiu cch chia nhng xp bi 5 qun cho mi ng-i trong 4 ng-i

chi t mt c bi chun 52 qun ?

Gii: Chng ta s dng quy tc nhn gii bi ton ny. Tr-c tin chng ta thy ng-i
u tin c th nhn -c 5 qun bi bng C525 cch. V ch cn 47 qun bi, ng-i th hai c th
5
-c chia 5 qun bi bng C 47
cch. Tip , ng-i th ba c th nhn -c 5 qun bi bng C425

cch. Cui cng, ng-i th t- nhn -c 5 qun bi bng C375 cch. V vy tng cng c:
5
C 52
C 457 C 452 C 357

52 ! 47 ! 42 ! 37 !
52 !

39953049823666054595520 !
4 7 !5 ! 4 2 !5 ! 3 7 !5 ! 3 2 !5 ! 5 !5 !5 !5 !3 2 !

cch chia cho 4 ng-i mi ng-i mt xp 5 qun bi.


Ch : Li gii ca v d 2.7.1 cng bng s hon v ca 52 phn t chia thnh 4 loi, mi

loi c 5 phn t ging ht nhau v loi th nm c 32 phn t. Tht vy, c th xy dng -c


mt php t-ng ng mt - mt gia cc hon v v cc cch chia bi nh- sau. Gi s c 52
-c nh s t 1 ti 52. Mi mt trong ng-i chi -c "pht" 5 . Nhng ca ng-i th nht
l loi 1, nhng chia cho ng-i th hai l loi hai, v.v. v 32 cn li l cc loi 5. Khi
mt cch chia xp 5 qun bi cho 4 ng-i chnh l mt cch sp xp mi qun bi vo mt .

V d 2.7.1 l mt bi ton in hnh v vic phn b cc vt khc nhau vo cc hp

khc nhau. Cc vt l 52 qun bi, cn 5 hp l 4 ng-i chi (mi hp xp 5 qun bi) v mt

bn t s qun bi cn li (32 qun). Trong tr-ng hp tng qut ta c: S cch chia n vt


khc nhau vo trong k hp khc nhau sao cho c n 1 vt -c t vo hp th i (i = 1, 2, ,

k) bng

n!
n 1 ! n 2 !... n k !

2.8. So snh cc cu hnh t hp

Cc cu hnh t hp m ta xt trn l b cc phn t -c chn t tp cc phn t cho


theo hai tiu chun: th t ly v iu kin ly lp li. phn bit r cc cu hnh t hp ta
c -a ra bng so snh sau:

Khi nim
K n th t ly
Chnh hp lp chp k t n
C
phn t

Lp li
C

Cng thc

Ank n k

Chnh hp khng lp chp


C
k t n phn t

Khng

T hp chp k t n phn t Khng

Khng

Cnk

T hp lp chp k t n
Khng
phn t

Rnk Cki 1Cnk i

Hon v t n phn t

Khng

Khng

Pnk

n!
(n k )!

n!
(n k )!k!
k -1

i 0

Pn n!

3. Sinh cc cu hnh t hp
Chng ta m t cc ph-ng php m s cc cu hnh t hp vi cc thuc tnh khc

nhau. Tuy nhin, trong nhiu tr-ng hp, ta cn phi lit k cc cu hnh , hoc la chn cc
cu hnh c th theo nhng tiu chun khc nhau. Trong v d 2.3.3 ta nu mt minh ho c
th, khi m ng-i bn hng cn chn cho mnh mt l trnh i ti -u nht trong s 5040 l trnh

khc nhau i qua 7 thnh ph. Mt v d khc, gi s c mt tp gm 6 s. Hy chn ra mt s


phn t ca tp ny sao cho tng ca chng bng 100. Mt cch tm cc phn t ny l to ra tt

c 26 = 64 tp con ca tp cho ri kim tra tng cc phn t ca chng. Hai v d nu trn

chng t rng, trong mt s bi ton m, cn phi sinh ra cc cu hnh t hp, trn c s la


chn cu hnh c th gii quyt cc bi ton t ra.
3.1. Sinh cc hon v
Mi phn t ca tp n phn t bt k u c th t-ng ng mt-mt vi tp cc s

nguyn A = 1, 2, , n. Do vy, lit k cc hon v ca tp ta c th sinh ra cc hon v

ca tp A, sau thay th cc s nguyn bng cc phn t ca c ch s t-ng ng. Chng ta

s m t mt trong cc ph-ng php lit k cc hon v ca tp A: ph-ng php sinh hon v

theo th t t in. Hon v a1 a2an -c gi l ng tr-c (nh hn) hon v b1b2 bn nu vi k


no (1 k n) ta c: a1 = b1, a2 = b2, , ak-1 = bk-1, v ak < bk. Ni cch khc, mt hon v

ca tp n s nguyn d-ng u tin -c gi l ng tr-c (theo th t t in) mt hon v

khc nu ti v tr u tin m hai hon v khc nhau ch s ca hon v u nh hn ch s


thuc hon v th hai.

V d 3.1.1. Hon v 23145 ca tp 1, 2, 3, 4, 5 l ng tr-c hon v 23514, v nhng

hon v ny trng nhau hai v tr u tin, nh-ng s 1 v tr th ba ca hon v u nh hn s

5 cng v tr th ba ca hon v sau. T-ng t hon v 41532 l ng tr-c 52143.

Thut ton sinh ra cc hon v ca tp A da trn th tc xy dng hon v k tip theo th

t t in ca hon v cho tr-c a1a2an. Cch xy dng hon v lin tip ny.-c tin hnh

nh- sau. u tin ta gi s an-1 < an. R rng nu i ch an-1 v an cho nhau th s nhn -c
hon v mi ng sau (ln hn) hon v cho v khng th c mt hon v no khc ln hn
hon v xut pht m li b hn hon v nhn -c bng vic i ch an-1 v an. V d, hon v
lin tip sau hon v 234156 l hon v 234165. Nu an-1 > an th khng th nhn -c hon v

ln hn bng cch i ch hai s hng ny trong hon v cho. By gi ta hy xem xt ba s

hng cui cng trong hon v a1a2an. Nu an-2 < an-1 th c th sp xp li ba s cui cng

c th nhn -c mt hon v mi lin sau hon v xut pht. Trong hai s an-1 v an ta chn s
nh hn t vo vo v tr n - 2, sau t s nguyn cn li v s an - 2 vo hai v tr cui cng

theo th t tng dn. V d, hon v lin sau hon v 234165 l 234516.

Nu an-2 > an-1 (v an-1 > an), th khng th nhn -c hon v ln hn bng cch i ch

ba s hng cui cng ca hon v. Da trn nhng nhn xt ny ta c th m t ph-ng php


tng qut to hon v lin sau (theo th t t in) ca hon v cho tr-c a1 a2an nh- sau:

Tr-c tin, tm cp s nguyn lin k u tin tnh t phi qua tri ca hon v m s u

nh hn s sau, tc l tm cc s nguyn aj v aj+1 sao cho aj < aj+1 v aj+1 > aj+2 > >an. Sau
, nhn -c hon v lin sau, ta t vo v tr th j s nguyn nh nht trong cc s ln hn

aj ca tp aj+1, aj+2, , an , ri lit k theo th t tng dn ca cc s cn li ca aj+1, aj+2, , an

vo cc v tr j + 1, , n. C th thy rng, khng c hon v no ln hn hon v xut pht v

nh hn hon v va to ra.

V d 3.1.2. Tm hon v lin sau theo th t t in ca hon a= (a1a2a3 a4a5 )=362541.


Gii: Cp s nguyn u tin phi tnh t phi qua tri c s tr-c nh hn s sau l a3 =

2 v a4 = 5. S nh nht trong cc s bn phi ca s 2 m li ln hn 2 l s 4. t s 4 vo v

tr th 3. Sau t cc s 2, 5, 1 theo th t tng dn vo ba v tr cn li. Hon v lin sau theo

th t t in ca hon v cho 362541 l 364125.

Nu chng ta p dng lin tip (n! - 1) ln th tc m t trn, bt u bng hon v

nh nht theo th t t in, tc l 12n, ta s nhn -c tt c n! cc hon v ca tp n s


nguyn A = 1, 2, , n.

V d 3.1.3. S dng thut ton nu trn to cc hon v ca cc s nguyn 1, 2, 3 theo th

t t in.

Gii: Chng ta bt u bng hon v 123. i ch 3 v 2 ta nhn -c hon v tip theo l

132. Tip theo, v 3 > 2 v 1 < 3 nn phi xt c 3 s trong nhm hon v ny. t s nh nht

trong hai s 3 v 2 (s 2) vo v tr th nht, sau l hai s cn li (1 v 3) vo v tr th 2 v


th 3 theo th t tng dn, tc l ta c 213. i ch 1 v 3 cho nhau v 1 < 3 -c 231. V 2 < 3

nn -c t 3 vo v tr u tin v sau l 1 v 2, tc l 312. Cui cng, i ch 1 vi 2 ta

nhn -c hon v ln nht 321. Kt qu trnh t cc hon v xy dng -c l : 123; 132; 213;

231; 312; 321.

3.2. Sinh cc t hp
V t hp ca mt tp hu hn chnh l mt tp con ca tp , nn ta c th dng php

t-ng ng mt - mt gia cc tp con ca tp = a1 , a2, , an v cc xu nh phn di n


sinh cc t hp. Mt xu nh phn ng vi mt tp con, ti v tr k, s c s 1 hoc s 0 tu

theo phn t ak thuc vo tp con hay khng. Nu c th lit k tt c cc xu nh phn di

n th ta s nhn -c tt c cc tp con ca tp n phn t (v cng chnh l cc t hp ca tp


).

Mt khc, mt xu nh phn di n cng l khai trin nh phn ca mt s nguyn nm

gia 0 v 2n - 1. Khi 2n xu nh phn c th lit k theo th t tng dn ca s nguyn trong

biu din nh phn ca chng. Chng ta s bt u t xu nh phn b nht 000 (n s 0). Mi


b-c tm xu lin sau mt xu nh phn cho tr-c bao gm vic tm v tr u tin tnh t phi

qua tri m l s 0, sau thay tt c s 1 bn phi s ny bng 0 v t s 1 vo chnh v


tr ny.

V d 3.2.1. Tm xu nh phn lin sau ca b=( b1b2b3b4b5b6 b7b8b9)=100011111.


Gii: Bit u tin t bn phi sang bng 0 l bit th t-, ta thay n bng s 1 v 5 bit 1 bn

phi n -c thay bng s 0, t ta -c xu nh phn lin sau ca xu cho l 10010

0000.

V d 3.2.2. p dng thut ton trn vi n=3.


Gii: '000' '001' '010' '011' '100' '101' '110' '111'

Tip theo chng ta s trnh by thut ton to cc t hp chp k t n phn t 1, 2, ,


n. Mi t hp chp k c th biu din bng mt xu tng, do vy c th lit k cc t hp theo
th t t in. R rng, theo th t t in, t hp u tin l (1, 2, . . ., k) v t hp cui cng l
(n-k+1, n-k+2, . . .,n). Gi s (a1a2ak) l t hp ang c ch-a phi l t hp cui cng theo
th t t in, khi c th chng minh -c rng t hp k tip trong th t t in c th xy
dng bng cch thc hin cc quy tc bin i sau i vi t hp ang c:

Tm t bn phi dy a1a2ak phn t ai n-k+i ; Nu khng thy th dng.

Thay ai bi ai +1

Thay aj bi aj +j - 1 vi j = i+1, i+2, k.


minh ho ta xt v d sau.
V d 3.2.3. Tm t hp chp 4 t tp 1, 2, 3, 4, 5, 6 i lin sau t hp 1, 2, 5, 6.
Gii: Li t phi qua tri ta thy a2 = 2 l s hng u tin ca t hp cho tho mn

iu kin ai 6 - 4 + i (trong tr-ng hp ny i=2). nhn -c t hp tip sau ta tng ai ln

mt n v, tc a2 = 3, sau t a3 = 3+1 = 4 v a4 = 3 + 2 = 5. Vy t hp lin sau cho l

1, 3, 4, 5.

V d 3.2.4. Lit k cc t hp chp 3 t tp 1, 2, 3, 4, 5


Gii: Ta c : C53

5!
10 . p dng thut ton bt u t t hp 1 2 3 ta thu -c tt c
(5 3)!2!

cc t hp cn tm:
1 2 3 1 2 4 1 2 5 1 3 4 1 3 5 1 4 5 2 3 4 2 3 5 2 4 5 3 4 5
3.3. Nh thc Newton
Mt s tnh cht quan trng lin quan n cc h s t hp c ng dng rng ri. Trong

tit ny ta s -a ra mt vi tnh cht v cc ng dng ca chng. C th chng minh -c cc


cng thc sau:

a. Cn0 C nn 1

b. Cnk Cnk 1 C nn1


c. Cnk Cnk 1 C nk1
d. ( x y) n Cin x i y n i
n

i 0

Chng minh. Cc cng thc a-c c th chng minh mt cch trc tip. V d, xt cng

thc c, ta c:

C nk C nk 1

n!
n!
n!
1
1
[
]

(n - k)! k! ( n k 1)! ( k 1)! ( n k )! ( k 1)! k ( n k 1)


n!
n 1
( n 1)!

C nk 1
( n k )! ( k 1)! ( n k 1) k
( n k 1)! k !

Cng thc c cng c th -c chng minh nh- sau, gii s c tp = a1, a2, , an

{a} c n+1 phn t, c bao nhiu tp con ca c k phn t ? R rng l, s tp con c k phn

t ca l Cn 1 , mt khc s tp con ca c th chia thnh 2 loi gm s tp con khng


k

cha phn t {a} l C nk 1 v s tp con cha phn t {a}l C nk t ta c: Cnk Cnk 1 C nk1 .
chng minh cng thc mc d, ta c th dng ph-ng php quy np theo n.
B-c 1. Cng thc d ng vi n=1: (x+y) = x+y v n=2 : (x + y)2 = x2 + 2xy + y2

B-c 1. Gi s cng thc d ng vi n k, ta cn chng minh n cng ng vi n = k+1.

Tht vy, ta c

( x y)k 1 ( x y)k (x y) Cik xi y k i ( x y) Cik ( xi1 y k i xi y (k 1)i ) Ck0xyk


k

C y
0
k

k 1

C x y
1 2
k

k 1

i 0

C xy C x y
1
k

2 3
k

k 2

i 0

.... C xk 1 Ckk x k y y k 1 (Ck0 Ck1 ) xyk


k
k

(Ck1 Ck2 ) x2 y k 1 ....... (Ckk 1 Ckk ) x k y x k 1 Ck01 x0 y k 1 Ck11 xyk


k 1

Ck21 x2 y (k 1)2 ...... Ckk1 xk y Ckk11 x k 1 Cki 1 xi y ( k 1)i


i 0

L-u ta s dng ng thc Pascal trong qu trnh bin i trn. Cng thc d th-ng

-c gi l cng thc trin khai nh thc Newton. D dng thu -c mt s h qu hin nhin
ca nh thc Newton:

Khi x = y =1, ta c: C ni 2 n
n

i 0

Khi x = -1 , y =1 ta c : ( 1) i C ni 0

i
i
( 1) C n

i
i
( 1) Cn

[n / 2]

[ n / 2]

i0

i 2 k ; k 0

i 2 k 1; k 1

Chng ta xt thm mt vi v d.

V d 3.3.1. Tnh tng cc h s ca cc s hng cha x2 trong trin khai biu thc

(x + y + z)n

Gii : Theo cng thc ca nh thc Newton ta c:


n

( x y z ) n [ x ( y z )] n Cni x i ( y z ) n i
i 0

s hng cha x khi i=2 , tc l cc s hng sau


2

n2

x2 ( y z)n2 Cn2 ( Cnj2 y j z n2 j ) x2


j0

vy tng cc h s ca cc s hng cha x2 l


n2

Cn2 ( C nj 2 )
j 0

n( n 1) n 2
2
2 n 3 n( n 1)
2

k i

V d 3.3.2. Chng minh hng ng thc Vandermonde: Cn m CnC m


k

i 0

Gii : V tri ca ng thc l s tp con c k phn t ca tp c (n + m) phn t. Gi s

c tp = a1, a2, , an , an+1, , an+m}= 1 2 , trong 1 = a1 , a2, , an v 2 = an+1,

an+2 , , an+m nh- vy chn tp con c k phn t ca ta c th chn theo cc ph-ng n

sau:

Chn tt c k phn t t tp 2 , ta c Cmk tr-ng hp,

Chn 1 phn t t tp 1 v (k-1) phn t t tp 2, ta c Cn1Cmk - 1 tr-ng hp,

....

Chn i phn t t tp 1 v (k-i) phn t t tp 2 , ta c Cni Cmk -i tr-ng hp,

....

Chn c k phn t t tp 2 ,ta c Cnk tr-ng hp.

Theo nguyn l cng ta c

Cnk m Cmk Cn1Cmk 1 Cn2 Cmk 2 ... Cnk 1Cm1 Cnk Cni Cmk i
n

i 0

V d 3.3.3. Chng minh cng thc sau : kC k


n
k 1

n2 n -1

Gii : Ta c th gii bi ny bng hai cch sau :


n

Cch 1. Theo nh thc Newton ta c (1 x ) n C k x k , x. o hm theo x c hai


n
v ta -c:

k 0

k 0

k 1

n(1 x)n1 kCnk xk 1 kCnk xk 1


n

t x = 1 ta thu -c iu phi chng minh: kC k n2n-1


n
k 1

V d 3.3.4. Chng minh cng thc tnh t hp lp chp k t n phn t:


k -1

Rnk Cki 1Cnk i Cnk k 1


i 0

Gii : Ta c ( x 1) n k 1 ( x 1) k 1 ( x 1) n
Trin khai cng thc nh thc Newton c hai v:
Xt v tri:

( x 1) n k 1

n k 1

j 0

k
k
Cnj k x j . Do vy, h s ca s hng cha x l Cn k 1

Xt v phi:

k
k 1
n
( x 1) k 1( x 1) n ( Cki 1 xi )( Cnj x j ) . Do vy, h s ca s hng cha x l

i 0

j 0

k 1 i
k i
Ck 1Cn
i 0

V h s ca s hng cha xk hai v ca ng thc phi bng nhau, ta suy ra iu phi chng
minh.

V d 3.3.5. Chng minh cng thc sau:


k

Cn i
i

i 0

Cnk k 1

Gii: p dng h thc Pascal ta c:


k

Cni
i

i 0
Cn1 2

Cn0 Cn11 Cn13 . . . . Cnkk Cn01 Cn11 Cn2 2 . . . . Cnkk

Cn22 . . . . Cnk k Cn23 . . . . Cnk k . . . . Cnk k 1

V d 3.3.6. Chng minh ng thc sau:


n

C2nn (Cni ) 2
i 0

Gii : p dng ng thc Vandermonde v d 3.4.2 khi k=n; m=n v tnh cht b ta c:

i0

i 0

C2nn Cni Cnn i (Cni ) 2


4. Nguyn l Dirichlet
4.1. M u

Nguyn l Dirichlet do nh ton hc ng-i c ni ting l Dirichlet xut t th k


19 -c p dng chng minh s tn ti nghim trong nhiu bi ton t hp. Nguyn l ny
-c pht trin t mt mnh rt n gin gi l nguyn l Chung chim b cu: Gi s c
mt n chim b cu bay vo chung. Nu s chim nhiu hn s ngn chung th chc chn c
t nht mt ngn c nhiu hn mt con chim.

Hnh 1. Minh ho 6 con chim v 4 ngn chung

Mt cch tng qut, nguyn l Dirichlet -c pht biu nh- sau: Nu xp nhiu hn n
i t-ng vo n ci hp th tn ti t nht mt hp cha khng t hn 2 i t-ng.
Vic chng minh nguyn l ny c th tin hnh bng lp lun phn chng rt n gin:

Gi s khng hp no cha nhiu hn mt i t-ng, th ch c nhiu nht l n i t-ng -c


xp trong cc hp, tri vi gi thit l s i t-ng ln hn n.

V d 4.1.1. Mt nm c nhiu nht 366 ngy. Do vy trong s 367 ng-i bt k bao gi


cng c t nht 2 ng-i c cng ngy sinh.
V d 4.1.2. Thang im bi kim tra -c cho t 0 n 10, tc l c 11 thang im
khc nhau. Do vy, trong s 12 sinh vin bt k ca lp s c t nht 2 ng-i c kt qu bi kim
tra ging nhau.

V d 4.1.3. Cp bc qun hm ca s quan c 8 cp t thiu u n i t. Do vy trong


mt n v c 9 s quan th s c t nht hai ng-i cng cp bc.

V d 4.1.4. C 20 thnh ph, gia cc thnh ph c th c -ng giao thng ni trc


tip vi nhau hoc khng. Chng minh rng c t nht 2 thnh ph c s thnh ph khc ni trc
tip vi chng l nh- nhau.

Gii. Ta gi ai l s thnh ph c -ng giao thng ni trc tip vi thnh ph th i v


= a1, a2, , a20 l tp cc gi tr , khi mi gi tr 0 ai 19 , khng th ng thi
c mt gi tr 0 v 19 v nu trong tp c gi tr 0 tc l c mt thnh ph c lp th s khng
c thnh ph no -c ni vi c 19 thnh ph cn li, ng-c li cng vy. Do tp ch c
ti a 19 gi tr khc nhau. Theo nguyn l Dirichlet s tn ti t nht mt cp i j sao cho ai=
aj .

V d 4.1.5. Cho nm im M1(x1,y1), M2(x2,y2), M3(x3,y3), M4(x4,y4), M5(x5,y5)


c cc to nguyn trn mt phng to Decac. Chng minh rng tn ti t nht hai im m
c to trung im ca on thng ni hai im l cc s nguyn.

Gii. t = M1, M2,M3, M4,M5, ta xy dng nh x f(Mi)=(xi mod 2, yi mod 2)


tc l f : -> B2 , trong B2 l tp cc dy nh phn c di 2. Ta c N() =5, N(B2)=4,
theo nguyn l Dirichlet tn ti Mi Mj sao cho f(Mi) = f(Mj) tc l cc to ca hai im
theo trc x v y u cng chn hoc cng l, do vy trung im ca on thng ni hai im ny
c to l cc s nguyn.

V d 4.1.6. Khi kim k danh mc 80 chi tit, mi chi tit c th -c nh gi l tt

hoc khng tt, c 50 chi tit -c nh gi l tt. Chng minh rng c t nht hai chi tit

-c nh gi l khng tt c s th t cch nhau 3 hoc 6 n v.

Gii. = a1, a2, , a30, 1 ai 80 l tp cc s th t khc nhau ca cc chi tit

-c nh gi l "khng tt", ta xy dng tp * = a1, a2, , a30, a1+3, a2+3, , a30 +3,

a1+6, a2+6, , a30+6} = v k hiu l * = b1, b2, , b90}, theo u bi c 1

bi 86. Tp * c 90 s nguyn nhn 86 gi tr khc nhau, theo nguyn l Dirichlet tn ti t

nht mt cp hai s bi = bj , cc s bi , bj khng ng thi nm trong , bi, bj cng khng ng

thi nm trong v bi, bj cng khng ng thi nm trong . Vy ch c th l bi = ai v

bj = ak +3 , tc l ai = ak +3 hoc bi = ai v bj = aq +6 , tc l ai = ak +6 hoc bi
= ak+3 v bj = aq +6 , tc l ak = aq +3.
4.2. Nguyn l Dirichlet tng qut

Nu xp n i t-ng vo k hp, th tn ti t nht mt hp cha khng t hn [n/k] i


t-ng (k hiu [x] l s nguyn nh nht khng nh hn x). Theo ngn ng tp hp v nh x,
nguyn l Dirichlet tng qut c th pht biu nh- sau :

Cho X v Y l tp hu hn f : X -> Y l nh x t X vo Y, gi N(X) = n, N(Y) = m v


k = [n/m], khi tn ti khng t hn k phn t ca tp X : x1 x2 . . . xk sao cho f(x1) =
f(x2)=. . . = f(xk).

Chng minh. Ta dng ph-ng php phn chng, khng gim tnh tng qut ta k hiu ta
k hiu X = x1, x2, , xn v Y = y1, y2, , ym, Xi = {x X / f(x)= yi}. Gi s N(Xi) k1 vi 1 i m, n= N(X) = N(Xi) (k-1)(n/m)<k(n/m)<n, t suy ra tn ti N(Xi) > k.
V d 4.2.1. Trong 150 ng-i c t nht [150/12] = 13 ng-i cng thng sinh.
V d 4.2.2. Cn phi c ti thiu bao nhiu sinh vin ghi tn vo lp ton hc Ri rc

chc chn rng s c t nht 6 ng-i t cng mt im thi, nu thang im gm 6 bc

0,1,2,3,4,5?

Gii. c t nht 6 ng-i cng im thi s sinh vin ti thiu l s nguyn nh nht sao

cho [N/6] = 6. S l N = 6.6 + 1 = 37.

V d 4.2.3. Chng minh rng c p s 1 v q s 0 xp thnh vng trn theo trt t ngu

nhin, nu p, q, k l cc s nguyn d-ng tho mn iu kin p kq th tn ti t nht mt dy


khng t hn k s 1 i lin nhau.

Gii. Xp q s 0 to thnh q khe -a p cc s 1 vo. Khi theo nguyn l Dirichlet


tn ti t nht mt khe c khng t hn [p/q] k s 1.

4.3. Mt vi ng dng th v ca nguyn l Dirichlet


Trong nhiu p dng ca nguyn l Dirichlet cc i t-ng t vo hp cn phi -c

chn mt cch khn kho. By gi chng ta s m t mt vi p dng nh- vy.

V d 4.3.1. Trong mt thng 30 ngy mt cng nhn lm t nht mi ngy mt sn phm,

nh-ng c thng lm khng qu 45 sn phm. Chng minh rng c nhng ngy lin tip ng-i
ny lm ra 14 sn phm.

Gii. Gi aj l s sn phm ng-i lm k t ngy u thng ti ht ngy j. Khi

a1, a2, , a30 l mt dy s nguyn d-ng phn bit v tng dn vi 1 aj 45. Hn th na


a1 + 14, a2 + 14, , a30 + 14 cng l mt dy cc s nguyn d-ng phn bit v tng dn vi
15 a1 + 14 59.

Su m-i s nguyn d-ng a1, a2, , a30, a1 + 14, a2 + 14, , a30 + 14 lun nh hn

hoc bng 59. Theo nguyn l Dirichlet c t nht hai trong 60 s ny bng nhau. V cc dy a1,

a2, , a30 v a1 + 14, a2 + 14, , a30 + 14 gm cc s phn bit nn tn ti cc ch s i, j


sao cho ai = aj + 14 (j < i). iu ny c ngha l t ngy j + 1 ti ht ngy i ng-i lm

ng 14 sn phm.

V d 4.3.2. Chng t rng trong n + 1 s nguyn d-ng khng v-t qu 2n tn ti t

nht mt s chia ht cho mt s khc.

Gii: Ta vit mi s nguyn a1, a2, , an+1 d-i dng tch ca mt lu tha c s 2 vi

mt s l. Ni cch khc ta c aj = 2 j .q j trong kj l mt s nguyn khng m cn qj l mt


k

s d-ng l nh hn 2n. V ch c n s nguyn d-ng l nh hn 2n nn theo nguyn l

Dirichlet tn ti hai trong s l q1, q2, , qn+1 bng nhau, tc l c hai ch s i v j sao cho qi

qj = q (q l gi tr chung ca chng). Khi ai = 2 q v aj = 2 q . Suy ra nu ki kj th aj


ki

kj

chia ht cho ai cn trong tr-ng hp ng-c li ai chia ht cho aj.

S dng kho lo nguyn l Dirichlet c th chng minh s tn ti ca dy con tng hay

gim c di no trong mt dy cc s nguyn khc nhau cho tr-c. Tr-c khi trnh by

cc p dng ny chng ta s -a ra mt vi nh ngha. Gi s a1, a2, , an l dy cc s thc.


Dy con ca dy ny l dy c dng a i , a i ,..., a i trong 1 i1 i2 < , im N. Ni cch
1

khc, dy con l dy nhn -c t dy xut pht v gi nguyn th t bn u ca chng. Mt

dy gi l thc s tng nu mi s hng ln hn s hng lin tr-c n, v mt dy gi l thc s


gim nu mi s hng nh hn s hng lin tr-c n.

V d 4.3.3. Mi dy n2 + 1 s thc phn bit u c mt dy con di n + 1 hoc l thc

s tng hoc l thc s gim.

Gii. Gi s a1 , a2 ,..., an2 1 l dy n2 + 1 s thc khc nhau.


Gi ik l di ca dy con thc s tng di nht bt u t ak, cn dk l di ca dy

con thc s gim bt u t ak. Nh- vy ta kt hp s hng ak vi mt cp s nguyn (ik, dk).

Gi s khng c dy con thc s tng hoc thc s gim c di n + 1. iu ny c

ngha l c hai s d-ng ik v dk u nh hn hay bng n (vi k = 1, , n2 + 1). Theo quy tc

nhn c tt c n2 cp (ik, dk) v theo nguyn l Dirichlet c t nht hai trong s n2 + 1 cp bng

nhau. Ni cch khc, tn ti hai s hng as v at (vi s < t) sao cho is = it v ds = dt . iu ny l

khng th xy ra. Tht vy, v cc s hng ca dy l khc nhau, nn hoc l as < at hoc l as >
at . Nu as < at, khi , v is = it ta xy dng t as dy con tng c phn t av. Dy ny c di

l it + 1 = is + 1, iu ny l v l v dy con tng di nht bt u t as c di is ch


khng phi is + 1. Hon ton t-ng t i vi tr-ng hp as > at .

V d 4.3.4. Gi s trong mt nhm 6 ng-i m mi cp hai hoc l bn hoc l th.

Chng t rng trong nhm c ba ng-i l bn ln nhau hoc c ba ng-i l k th ln nhau.

V d ny trnh by cch p dng nguyn l Dirichlet vo l thuyt t hp m vn quen

gi l l thuyt Ramsey, tn ca nh ton hc ng-i Anh. L thuyt Ramsey -c dng gii

quyt nhng bi ton phn chia cc tp con ca mt tp cc phn t.

Gii. Gi A l mt trong 6 ng-i. Trong s 5 ng-i ca nhm hoc l c t nht ba ng-i

l bn ca A hoc c ba ng-i l k th ca A, iu ny suy ra t nguyn l Dirichlet tng qut,

vi [5/2] = 3. Trong tr-ng hp u ta gi B, C, D l bn ca A. Nu trong ba ng-i ny c hai


ng-i l bn th h cng vi A lp thnh mt b ba ng-i bn ln nhau (khng ai l k th ca ai

c), ng-c li, tc l nu trong ba ng-i B, C, D khng c ai l bn ai c th chng t h l b ba


ng-i th ln nhau. T-ng t c th chng minh trong tr-ng hp c t nht ba ng-i l k th
ca A.

5. H thc truy hi
5.1. Khi nim v cc v d

Mt s bi ton m gii -c bng cc k thut m trong cc tit tr-c cng c th gii

-c bng cch tm cc mi quan h, gi l quan h truy hi, gia cc s hng ca dy.

V d 5.1.1. Trong mt qun th vi sinh vt s l-ng cc c th tng gp i sau mi gi.

Sau 4 gi s l-ng chng l bao nhiu, nu ban u c tt c 5 c th? gii bi ton ny ta

gi s s vi trng sau n gi l an. V s vi trng tng gp i sau mi gi nn ta c quan h an =

2an-1 vi n l s nguyn d-ng tu , vi iu kin ban u a0 = 5. T y ta c th d dng xc


nh duy nht an i vi mi n khng m. C th, vi n=4 ta c: a4 = 2a3 = 2.2a2 = 2.2.2a1

= 2.2.2.2a0 = 24.5=80.
Trong tit ny v tit sau, chng ta s cp ti mt s ph-ng php tm cng thc

t-ng minh cho cc s hng ca dy c h thc truy hi thuc mt s loi no .

nh ngha 5.1.1. Xt dy s an. Nu c mt cng thc biu din an qua mt hay nhiu

s hng i tr-c ca dy a1 , a2, , an-1 vi mi n nguyn v n n0, trong n0 l nguyn khng

m, th cng thc -c gi h thc truy hi (cng thctruy hi, biu thc truy hi)
i vi dy an. Dy s an -c gi l li gii hay l nghim ca h thc truy hi nu cc s

hng ca n tho mn h thc truy hi ny.

V d 5.1.2. Cho an l dy s tho mn h thc truy hi an = an-1 - an-2 vi n = 2, 3, 4, ,

v gi s a0 = 5, a1 = 9. Tm a2 v a3 ?

Gii: T h thc truy hi ta c a2 = a1 - a0 = 9 - 5 = 4 v a3 = a2 - a1 = 4 - 9 = -5.


V d 5.1.3. Dy an, vi an = 3n vi mi n nguyn khng m c l li gii ca h thc

truy hi an = 2an-1 - an-2 vi n = 2, 3, 4, , hay khng? Cu hi cng nh- vy i vi an = 2n

v an = 5.

Gii: Gi s an = 3n vi mi n nguyn khng m. Khi vi n 2 ta thy rng an = 2an-

1 - an-2

= 2[3(n - 1)] - 3(n-2) = 3n. Do , an trong an = 3n l li gii ca h thc truy hi

cho.

Gi s, an = 2n vi mi n nguyn khng m. R rng a0 = 1, a1 = 2 v a2 = 4. V a2 2a1

- a0 = 2.2 - 1 = 3, do vy dy an trong an = 2n khng l mt li gii ca h thc truy hi


cho.

Gi s, an = 5 vi mi n nguyn khng m. Khi vi n 2 ta thy an = 2an-1 - an-2 =

2.5 - 5 = 5. Do , dy an trong an = 5 l li gii ca h thc truy hi cho.

Cc iu kin u i vi dy s nh r gi tr cc s hng i tr-c s hng u tin k t

h thc truy hi c hiu lc. Chng hn, trong v d 5.1.2, a0 = 3 v a1 = 5 l cc iu kin


u. Cc iu kin u v h thc truy hi cho php ta xc nh duy nht dy s. Tr-c khi i

su tm hiu cc ph-ng php gii h thc truy hi, chng ta xt mt s v d lin quan n vic
m hnh ho bng cc h thc truy hi.

V d 5.1.4. Li kp. Mt ng-i gi 10 triu ng vo ti khon ca mnh ti mt ngn

hng vi li sut kp 8% mi nm. Hi sau 20 nm anh ta c bao nhiu tin trong ti khon ca
mnh?

Gii: Gi Fn l tng s tin c trong ti khon ngn hng sau n nm. V s tin c trong

ti khon sau n nm bng s c sau n - 1 nm cng li sut nm th n, nn dy Fn tho mn h

thc truy hi sau:


Fn = F n-1 + 0,08Fn-1 = 1,08F n-1

iu kin u l F0 = 10 triu ng.


Chng ta c th dng ph-ng php lp tm cng thc trn cho Fn. Ta c:
F1 = (1,08)F0
2

F2 = (1,08)F1 = (1,08) F 0

Fn = (1,08)Fn-1 = (1,08) F 0.

Thay iu kin u F0 = 10 triu ng vo ta nhn -c cng thc Fn = (1,08)n 10 triu

ng. Khi n = 20 ta c:

F20 = (1,08)

20

.10 20 406 506 ng.

V d 5.1.5. H nh th v s Fibonaci. Bi ton sau y -c t ra u tin bi Fibonaci

vo th k th 13 trong tc phm mang tn Liber abaci. Mt cp th mi sinh (mt con c v


mt con ci) -c th ln mt hn o. Gi s rng mi cp th khng c kh nng sinh sn
-c tr-c khi chng y 2 thng tui. T khi y 2 thng tui, mi thng chng s -c mt

i th con. Tm cng thc truy hi tnh s cp th trn o sau n thng, nu coi cc con th l
tr-ng th.

Gii: Gi s Fn l s cp th sau n thng. S l-ng cc cp th c th tnh -c bng h

thc truy hi. Cui thng th nht s cp th trn o l F1 = 1. V cp th ny vn ch-a n

tui sinh sn -c nn trong thng th hai cng l F2 = 1. tm s cp th sau n thng, ta


cng s cp th trn o thng tr-c Fn-1 v s cp th mi l Fn-2 v mi cp th con -c

sinh ra t cp th c t nht 2 thng tui.

Do vy dy Fn tho mn h thc truy hi:


Fn = F n-1 + F n-2

vi n 3 cng vi iu kin u F1 = 1 v F2 = 1. Dy s xc nh nh- trn -c gi l dy

Fibonaci. C th thy iu kin u v h thc truy hi xc nh duy nht dy s ny nn s cc


cp th trn o sau n thng -c cho bi s Fibonaci th n.

V d 5.1.6. Thp H ni. Tr chi xp hnh rt ph cp vo cui th k 19 gi l Thp H

ni. T-ng truyn rng ti mt ngi thp c mt tm bng ng trn c 3 ci cc bng


kim c-ng. Lc khai thin lp a, trn mt trong ba ci cc th-ng 64 chic a bng

vng vi -ng knh gim dn (hnh 1). Ngy m cc nh s- dch chuyn a sang mt ci cc
khc theo quy tc: mi ln ch -c dch chuyn mt a, mt a c th dch chuyn t mt cc
ny sang cc khc bt k, nh-ng khng -c mt chic a ln trn mt a khc c -ng
knh nh hn. Ngy tn th s n khi tt c cc a -c chuyn sang mt ci cc khc.

Gi s Hn l s ln dch chuyn cn thit gii bi ton Thp H ni c n a. Hy lp

h thc truy hi i vi dy Hn.

Hnh1. Trng thi ban u ca thp H Ni

Gii: Thot u n a trn cc A. Chng ta c th dch chuyn n - 1 chic a nm trn

sang cc C, theo quy tc nu trn, v phi dng Hn-1 ln dch chuyn, (xem hnh 2) v chic

a ln nht -c gi c nh trong khi di chuyn ( n - 1) a b trn. Tip theo, ta chuyn chic

a ln nht ny bng mt ln dch chuyn t cc A sang cc B. Cui cng ta mt Hn-1 ln dch


chuyn (n - 1) chic a t cc C sang cc B v t ln trn chic a ln nht vn -c gi c

nh khi di chuyn (n - 1) a b. Do vy, ta c h thc truy hi:


Hn = 2Hn-1 + 1.

iu kin u H1 = 1, v ch cn mt ln dch chuyn mt a cc A sang cc B theo

ng quy tc ca cuc chi.

B
Trng thi trung gian ca Thp H ni.

Dng ph-ng php lp gii h thc truy hi ny, ta nhn thy:


Hn = 2Hn-1 + 1
2

= 2(2Hn-2 + 1) + 1 = 2 Hn-2 + 2 + 1.
2

= 2 (2Hn-3 + 1) + 2 + 1 = 2 Hn-3 + 2 + 2 + 1.

=2

n-1

=2

n-1

H1 + 2

n-2

+ . . . 2 + 1.

- 1.

Trong ng thc tr-c ng thc cui cng ta s dng iu kin u H1 = 1. ng

thc cui cng chnh l cng thc tnh tng cc s hng ca mt cp s nhn.

Ph-ng php lp cho ta nghim ca h thc truy hi Hn = 2Hn-1 + 1 vi iu kin u

H1 = 1. Cng thc ny cng c th chng minh bng quy np ton hc.


Khi thay n = 64 vo h thc trn ta -c:
H64 = 264 - 1 = 18 446 744 073 709 551 615.

l s ln cn thit dch chuyn tt c cc a t mt cc ny sang cc khc. Nu

mi ln dch chuyn mt mt giy, th cn hn 500 t nm cc nh s- dch chuyn tt c cc


a sang cc khc.

V d 5.1.7. Tm h thc truy hi v iu kin u tnh s cc xu nh phn di n v

khng c hai s 0 lin tip. C bao nhiu xu nh phn nh- th c di bng 5?

Gii: Gi an l s cc xu nh phn di n v khng c hai s 0 lin tip. nhn -c

h thc truy hi cho an, ta thy rng theo quy tc cng, s cc xu nh phn di n v khng
c hai s 0 lin tip bng s cc xu nh- th kt thc bng s 1 cng vi s cc xu nh- th kt

thc bng bit 0. Chng ta gi s rng n

3, tc l cc xu nh phn c t nht ba bit.

Cc xu nh phn di n, khng c hai s 0 lin tip v kt thc bng bit 1 chnh l xu

nh phn nh- th di n - 1 v thm s 1 vo cui ca chng. Vy chng c tt c l an-1.

Cc xu nh phn di n, khng c hai s 0 lin tip v kt thc bng bit 0, cn phi c

bit th n - 1 bng 1, nu khng th chng c hai s 0 hai bit cui cng. T suy ra, cc xu

nh phn di n, khng c hai s 0 lin tip v kt thc bng bit 0 chnh l cc xu nh phn
khng c hai s 0 lin tip c di n - 2 v thm 10 vo cui ca n. Do vy s chng l an-2.
Cui cng ta rt ra:

an = an-1 + an-2 vi n 3.

iu kin u l a1 = 2 v c hai xu di 1 u khng c hai s 0 lin tip v a2 = 3,

v c 3 xu nh- th l 01, 10 v 11.

nhn -c a5, ta s dng h thc truy hi ba ln:


a3 = a2 + a1 = 3 + 2 = 5
a4 = a3 + a2 = 5 + 3 = 8
a5 = a4 + a3 = 8 + 5 = 13.
Ch : Ta thy rng an tho mn h thc truy hi ging nh- l dy Fibonaci. V a1 =F3

v a2 = F4 ta suy ra an = Fn+2.

V d tip theo ch ra cch s dng h thc truy hi m hnh ho s cc t m cho


php khi thc hin mt kim tra tnh hp l no .
V d 5.1.8. Tnh s t m. Mt h my tnh coi mt xu cc ch s h thp phn l mt

t m hp l nu n cha mt s chn ch s 0. Chng hn, 102304070869 l hp l cn

120987045608 l khng hp l. Gi s an l s cc t m hp l di n. Hy tm h thc truy


hi cho an.

Gii: C hai cch to xu hp l n ch s t xu n - 1 ch s.


u tin, xu hp l n ch s c th nhn -c bng cch ni thm ch s khc khng

vo xu hp l n - 1 ch s. Ta c 9 cch ni thm, vy bng cch ny c th to -c 9an-1 xu


hp l n ch s.

Tip theo, xu hp l n ch s c th nhn -c bng cch ni thm ch s 0 vo xu

khng hp l n- 1 ch s. S cc xu hp l n ch s loi ny chnh bng s cc xu khng hp

l n - 1 ch s, tc l bng 10n-1 - an-1, v c tt c 10n-1 xu n - 1 ch s trong c an-1 xu l


hp l.

V tt c cc xu hp l n ch s s -c to ra bng mt trong hai cch trn, nn ta suy

ra s xu hp l n ch s tho mn h thc truy hi:

an = 9an-1 + (10n-1 - an-1) = 8an-1 + 10n-1

5.2. Gii cc h thc truy hi

Cc h thc truy hi khc nhau c th gii bng nhiu cch khc nhau. Mt s c th gii

bng cch lp nh- trong cc v d nu trn, mt s khc c th gii bng nhng k thut c
bit. D-i y chng ta s cp ti mt ph-ng php gii t-ng minh mt lp cc h thc truy
hi theo mt cch h thng.

nh ngha 5.2.1. H thc truy hi c dng:

a n c 1a n 1 c 2 a n 2 ... c k a n k
trong c1, c2, , ck l cc s thc v ck 0, -c gi l h thc truy hi tuyn tnh thun

nht bc k vi h s hng s.

H thc truy hi trong nh ngha l tuyn tnh v v phi l tng cc tch ca cc s hng

tr-c ca dy vi mt h s. N l thun nht v mi s hng u c dng aj. Cc h s ca cc


s hng ca dy u l hng s, thay v l cc hm ph thuc n. Bc l k v an -c biu din
qua k s hng tr-c ca dy.

Nu cho tr-c k iu kin u: a 0 C 0 , a 1 C 1 ,..., a k 1 C k 1 , th theo nguyn l th

hai ca quy np ton hc, dy s tho mn h thc truy hi nu trong nh ngha s -c xc


nh duy nht.

V d 5.2.1. H thc truy hi Pn = (1,08)Pn-1 trong v d 5.1.4 l h thc truy hi tuyn

tnh thun nht bc 1. H thc truy hi Fn = Fn-1 + Fn-2 trong v d 5.1.5 l h thc truy hi
tuyn tnh thun nht bc 2. H thc truy hi an = an-5 l h thc truy hi tuyn tnh thun nht

bc 5.

V d 5.2.2. H thc an = an-1 + (an-2)2 khng l h thc truy hi tuyn tnh. H thc truy

hi Hn = 2Hn-1 + 1 l khng thun nht. H thc truy hi Bn = nBn-1 khng c h s hng s.

Ph-ng php c bn gii h thc truy hi tuyn tnh thun nht l tm nghim d-i

dng an = rn, trong r l hng s. Ta c an = rn l nghim ca h thc truy hi an= c1an-1 +

c2an-2 + + ckan-k khi v ch khi:

r n c 1r n 1 c 2 r n 2 ... c k r n k
Chia c hai v cho rn-k v tr v phi cho v tri chng ta nhn -c ph-ng trnh t-ng

-ng:

r k c 1 r k 1 c 2 r k 2 ... c k 1 r c k 0
Vy, dy an vi an = rn l nghim khi v ch khi r l nghim ca ph-ng trnh i s

trn. Ph-ng trnh ny -c gi l ph-ng trnh c tr-ng ca h thc truy hi v nghim ca n


-c gi l nghim c tr-ng ca h thc truy hi. Nh- chng ta s thy sau ny, cc nghim
c tr-ng dng cho cng thc t-ng minh ca tt c cc nghim ca h thc truy hi.

Tr-c tin chng ta s trnh by cc kt qu i vi h thc truy hi tuyn tnh thun nht

bc hai v xt tr-ng hp khi ph-ng trnh c tr-ng c hai nghim phn bit.

nh l 5.2.1. Cho c1, c2 l hai s thc. Gi s ph-ng trnh r2 - c1r - c2 = 0 c hai

nghim phn bit r1 v r2. Khi dy an l nghim ca h thc truy hi an = c1an-1 + c2an-2

khi v ch khi an 1r1n 2 r2n vi n = 1, 2, trong 1 v 2 l cc hng s.


Chng minh. Gi s r1 v r2 l hai nghim ca r2 - c1r - c2 = 0, tc l, r1 c1 r1 c 2
2

v r2 c1 r2 c 2 . Ta thc hin cc bin i sau:


2

c1an 1 c2 an 2 c1 ( 1r1n 1 2 r2n 1 ) c2 (1r1n 2 2 r2n 2 )


1r1n 2 (c1r1 c2 ) 2 r2n 2 (c1r2 c2 )

1r1n 2 r12 2 r2n 2 r22


1r1n 2 r2n
an

iu ny chng t dy an vi a n 1r1n 2 r2n l nghim ca h thc truy hi cho.


Ng-c li, gi s an l mt nghim bt k ca h thc truy hi. Ta s chn 1 v 2
sao cho dy an vi a n 1 r1n 2 r2n tho mn cc iu kin u a0 = C0 v a1 = C1. Tht
vy,
a0 = C0 = 1 + 2
a1 = C1 = 1r1 + 2r2
T ph-ng trnh u ra -c 2 = C0 - 1. Thay vo ph-ng trnh sau ta c:
C1 = 1r1 + (C0 - 1)r2 = 1(r1 - r2) + C0 - r2
Suy ra:
1

(c1 C 0 r2 )
r1r2

2 C 0 1 C 0

( C 1 C 0 r2 )
r1 r2

( C 0 r1 C 1 )
r1 r2

Vy khi chn cc gi tr 1 v 2 ny th dy an vi a n 1r1 2 r2 tho mn cc


n

iu kin u. V h thc truy hi v cc iu kin u xc nh duy nht dy, nn


a n 1 r1n 2 r2n . nh l -c chng minh.
Cc nghim c tr-ng ca h thc truy hi tuyn tnh thun nht vi h s hng s c
th l cc s phc. nh l 5.2.1 vn cn ng trong tr-ng hp ny, tuy nhin h thc truy hi
vi nghim c tr-ng phc s khng -c trnh by trong cun sch ny.
V d 5.2.3. Tm nghim ca h thc truy hi an = an-1 + 2an-2 vi a0 = 2, a1 = 7.
Gii: Ph-ng trnh c tr-ng ca h thc truy hi ny c dng r2 - r - 2 = 0. Nghim ca n

l r = 2 v r = -1. Theo nh l 5.2.1 dy an l nghim ca h thc truy hi khi v ch khi an =

12n + 2(-1)n vi cc hng s 1 v 2 no . T cc iu kin u ta suy ra:

a0 = 2 = 1 + 2
a1 = 7 = 12 + 2(-1).
Gii ra ta -c 1 = 3 v 2 = -1. Vy nghim ca h thc truy hi v iu kin u l dy

an vi an = 3.2n - (-1)n.

V d 5.2.4. Tm cng thc t-ng minh ca cc s Fibonaci.


Gii: D dng c cc nghim c tr-ng l:

r1

1 5
1 5
v r2
2
2

Suy ra cc s Fibonaci -c cho bi cng thc sau:

1 5
1 5
2

f n 1

2
2

vi cc hng s 1 v 2 no . Cc iu kin u f0 = 0, f1 = 1 cho php ta xc nh cc hng


s ny:

f 0 1 2 0
1 5
1 5
2

f1 1

2 1.
2

T hai ph-ng trnh ny cho ta 1

1
5

,2

1
5

Do cc s Fibonaci -c cho bng cng thc t-ng minh sau:


fn

1 1 5
1 1 5

5 2
5 2
n

Trong tr-ng hp ph-ng trnh c tr-ng ca h thc truy hi tuyn tnh thun nht bc 2 c

mt nghim c tr-ng l nghim bi hai, bng cch chng minh t-ng t nh l 5.2.1, ta c nh
l sau y :

0. Gi s r2 - c1r - c2 =0 ch c mt
nghim r0. Dy an l nghim ca h thc truy hi an = c1an-1 + c2an-2 khi v ch khi
a n 1r0n 2 nr0n vi n = 0, 1, 2, Trong 1 v 2 l cc hng s.
nh l 5.2.2. Cho c1 v c2 l cc s thc v c2

V d 5.2.5. Tm nghim ca h thc truy hi:


an = 6an-1 - 9an-2
vi cc iu kin u a0 = 1 v a1 = 6.

Gii: Ph-ng trnh c tr-ng r2 - 6r + 9 = 0 c nghim kp r = 3. Do nghim ca h

thc truy hi c dng:

an = 13n + 2n3n,
xc nh cc hng s 1 v 2 , ta s dng cc iu kin u:
a0 = 1 = 1.
a1 = 6 = 13 + 23.
Gii ra ta -c

kin u cho l:

1 = 1, 2 = 1. Do vy nghim ca h thc truy hi tho mn cc iu

an = 3n + n3n.

By gi chng ta s pht biu kt qu tng qut v nghim ca cc h thc truy hi tuyn


tnh thun nht vi h s hng s trong bc c th ln hn hai v vi gi thit ph-ng trnh c
tr-ng c cc nghim phn bit. Chng ta s khng chng minh kt qu ny y.
nh l 5.2.3. Cho c1, c2, , ck l cc s thc. Gi s ph-ng trnh c tr-ng rk - c1rk-1 -

- ck = 0 c k nghim phn bit r1, r2, , rk. Khi dy an l nghim ca h thc truy hi
an = c1an-1 + c2an-2 + + ckan-k khi v ch khi

a n 1r1n 2 r2n ... k rkn


vi n = 0, 1, 2, , trong 1,

2, , k l cc hng s.

V d 5.2.6. Tm nghim ca h thc truy hi


an = 6an-1 - 11an-2 + 6an-3
vi iu kin u a0 = 2, a1 = 5, a2 = 15.
Gii: C th thy r = 1, r = 2, r = 3 l 3 nghim c tr-ng phn bit ca a thc c tr-ng

t-ng ng vi h thc truy hi:

r3 - 6r2 + 11r - 6=0


Do vy nghim ca h thc truy hi c dng:
an = 11n + 22n + 33n.
tm cc hng s 1, 2, 3, ta dng cc iu kin u:
a0 = 2 = 1 + 2 + 3
a1 = 5 = 1 + 22 + 33

a2 = 15 = 1 + 24 + 39.

Gii h ph-ng trnh trn ta nhn -c 1 = 1, 2 = -1, 3 = 2. V th, nghim duy nht

ca h thc truy hi ny tho mn cc iu kin ban u cho l dy an vi


an = 1 - 2n + 2.3n.

kt thc mc ny, chng ta xt v d s dng h thc truy hi nh gi phc tp

(thi gian thc hin) thut ton sp xp theo cch la chn nh- l mt minh ho cho vic ng

dng cc h thc truy hi. Ni chung, phc tp ca mt thut ton l mt hm s ph thuc


vo kch th-c ca d liu u vo. Do vy, ng-i ta th-ng tm cch xy dng h thc truy

hi vi dy {sn}, trong sn l thi gian (tt nht, trung bnh hoc ti nht) cn thit thc

hin thut ton vi u vo c kch th-c n, sau gii h thc truy hi -a ra cng thc
nh gi phc tp ca thut ton nu.

V d 5.2.7. Cho dy cc s thc a1, a2, . . . , an, cn sp xp cc phn t ca dy theo

th t tng dn. Ta c th thc hin iu ny bng thut ton sp xp theo cch la chn nhsau: Tm phn t ln nht ca dy gm n s cho, sau i ch phn t xung v tr cui

cng ca dy, thc hin t-ng t vi cc dy con c (n-1), (n-2),1 s u ca dy. Thut ton
-c m t nh- sau:

Input : Dy cc s thc ai, a2, . . . , an ; n 1.


Output: Dy ai, a2, . . . , an cc s thc khng gim
1. [Tr-ng hp c s: n=1] Nu n=1, dng thut ton (dy -c sp xp).
2. [Tm phn t ln nht] max_index=1 (Bt u vi gi thit a1 l s ln nht)
3. Vi i ln l-t nhn gi tr t 2 ti n (i:=2 to n) thc hin dng 4 (Tm phn t ln nht)
4. Nu si >smax_index , t max_index:=i (Tm thy phn t ln hn, cn thay i ch s

max_index)

5. [Chuyn phn t ln nht xung v tr cui dy] i ch sn v smax_index


6. [ quy] Thc hin cc b-c trn vi dy ai, a2, . . . , an-1
Thi gian thc hin ch yu ca thut ton trn chnh l qu trnh so snh tm phn t ln

nht ca cc dy. nh gi phc tp thut ton, chng ta i xc nh s cc php so snh

sn cn thc hin dng 4 sp xp dy s theo th t tng dn. C th thy ngay l s1 = 0.


nhn -c h thc truy hi cho dy s1, s2,, sn chng ta xt thut ton sp xp trn vi n>1. R

rng l cc dng 1-3 khng i hi thc hin bt k php so snh no; dng 4 i hi n-1 php

so snh. Tip , dng 5 cng khng c php so snh no -c thc hin, cn dng 6, do li

gi quy thut ton sp xp dy c n-1 phn t, cn phi thc hin sn-1 php so snh. T cc
phn tch trn, ta nhn -c h thc truy hi sau:
sn = sn-1 + (n-1)
Gii h thc truy hi ny ta thu -c:
sn = sn-1 + (n-1)
= (sn-2 + (n-2)) + (n-1)
= (sn-3 + (n-3)) + (n-2) + (n-1)
......
2

= s1 + 1 + 2 + . . . .+ (n-1)= n(n-1)/2 = 0(n )

Nh- vy, thi gian cn thit thc hin thut ton sp xp theo cch la chn l 0(n2).

5.3. Quan h chia tr


Nhiu thut ton quy chia bi ton vi cc thng tin vo cho thnh mt hay nhiu

bi ton nh hn. S phn chia ny -c p dng lin tip cho ti khi c th tm -c li gii
ca cc bi ton nh mt cch d dng.

V d 5.3.1. Thut ton tm kim nh phn. Tm mt gi tr no (gi l kho - key) trong

mt dy xp theo th t tng dn cho tr-c.

Input: Dy si, si+1,... ,sj, i1, xp theo th t tng dn v gi tr key


Output: Cho gi tr ch s k sao cho sk = key, trong tr-ng hp khng tn ti gi tr key

trong dy th cho gi tr k=0.

1. [Khi to] left:=i; right:=j


2. [Khng tm thy?] Nu left > right th t k:=0. Dng thut ton (Khng tm thy gi tr

key trong dy).

3. [Xc nh im gia] t k:=(left + right)/2.


4. [Tm thy gi tr key] Nu key = sk, dng thut ton.
5. [Xc nh chiu tm kim] Nu key < sk, t j:=k-1 (thc hin vic tm kim tip theo

na bn tri). Trong tr-ng hp ng-c li, tc l key > sk, t i:=k+1 (thc hin vic tm kim
tip theo na bn phi).

6. [ quy] Thc hin cc b-c ca thut ton tm gi tr key trong dy si, si+1,... ,sj,

vi cc gi tr i, j mi.
Nh- vy, vic tm kim nh phn -c chng ta thc hin bng cch rt gn vic tm kim

mt phn t trong mt danh sch ti vic tm phn t trong mt danh sch c di gim i

mt na. Chng ta rt gn lin tip nh- vy cho ti khi cn li mt phn t. Cc thut ton kiu

nh- vy -c gi l cc thut ton chia tr. D-i y chng ta s nghin cu cc h thc truy
hi th-ng gp khi phn tch phc tp ca cc thut ton loi ny.

Gi s mt thut ton phn chia mt bi ton c n thnh a bi ton nh, trong mi bi

ton nh c c n/b ( n gin ta gi s n chia ht cho b; trong thc t cc bi ton nh th-ng

c c bng s nguyn gn nht ln hn hoc nh hn hay bng n/b). Cng vy, ta gi s tng
cc php ton thm vo khi thc hin phn chia bi ton c n thnh cc bi ton c c nh hn

l g(n). Khi , nu f(n) l s cc php ton cn thit gii bi ton cho, th f(n) tho mn
h thc truy hi sau:

n
f ( n ) af g( n )
b

H thc ny c tn l h thc truy hi chia tr.


V d 5.3.2. Xt thut ton tm kim nh phn. Thut ton ny -a bi ton tm kim c n

v bi ton tm kim phn t ny trong dy tm kim c n/2, khi n chn. Khi thc hin vic rt

gn cn hai php so snh: Mt xc nh na danh sch no -c tip tc s dng cho vic tm


kim v php so snh th hai xc nh xem danh sch c cn phn t no khng?. V th,

nu f(n) l s php so snh cn phi lm khi tm kim mt phn t trong danh sch tm kim c

n ta c f(n) = f(n/2) + 2, nu n l s chn.

V d 5.3.3. Xt thut ton nh v cc phn t ln nht v nh nht ca dy a1, a2, , an.

Nu n = 1, th a1 l phn t ln nht v nh nht ca dy. Nu n > 1 th ta chia dy ny thnh

hai dy hoc l chng c cng s l-ng cc phn t hoc l mt dy c nhiu hn dy kia 1


phn t. Bi ton thu v vic tm phn t ln nht v nh nht ca mi mt trong hai dy nh
hn. Li gii ca bi ton xut pht s nhn -c bng cch so snh cc phn t ln nht v nh
nht ca hai dy con.

Gi f(n) l tng s cc php so snh cn phi thc hin tm phn t ln nht v nh

nht ca tp n phn t. Chng ta ch ra rng bi ton c n c th -a v hai bi ton c n/2

(khi n chn), bng vic dng hai php so snh, mt l so snh cc phn t ln nht v sau l

so snh cc phn t nh nht ca hai tp con. V th, chng ta nhn -c h thc truy hi f(n) =
2f(n/2) + 2 khi n chn.

V d 5.3.4. Dng k thut chia tr c th xy dng -c cc thut ton c hiu qu

hn thut ton thng th-ng nhn hai s nguyn. D-i y ta s trnh by mt trong cc thut

ton nh- vy. l thut ton nhn nhanh. Tr-c tin ta phn chia mi mt trong hai s nguyn

2n bit thnh hai khi mi khi n bit. Sau php nhn hai s nguyn 2n bit ban u -c thu
v ba php nhn cc s nguyn n bit cng vi cc php dch chuyn v cc php cng.
Gi s a v b l cc s nguyn c cc biu din nh phn di 2n (trong tr-ng hp cn

thit, c th thm cc bit u bng 0 vo cc biu din ny lm cho chng c di nh-

nhau):

a ( a 2 n 1 a 2 n 2 ...a1 a0 ),b ( b2 n 1b2 n 2 ...b1b0 )

D dng thy rng c th biu din a v b d-i dng:


a 2 n A1 A0

Trong :

A1 ( a 2 n 1 a 2 n 2 ...a n 1 a n )2 ,

B1 ( b2 n 1b 2 n 2 ...b n 1b n )2 ,

b 2 n B1 B 0

A0 ( a n 1 ...a1 a 0 )2

B 0 ( b n 1 ..b1 b0 )2

Thut ton nhn nhanh cc s nguyn -c xy dng da trn ng thc:


ab ( 2 2 n 2 n ) A1 B1 2 n ( A1 A0 )( B0 B1 ) ( 2 n 1 ) A0 B0

ng thc ny ch ra rng php nhn hai s nguyn 2n bit c th thc hin bng cch

dng ba php nhn cc s nguyn n bit v cc php cng tr v php dch chuyn. iu c

ngha l nu f(n) l tng cc php ton nh phn cn thit nhn hai s nguyn n bit th

f(2n) = 3f(n) + Cn.

Ba php nhn cc s nguyn n bit cn 3f(n) php ton nh phn. Mi mt trong cc php

cng, tr, hay dch chuyn dng mt hng s nhn vi n ln cc php ton nh phn v Cn l
tng cc php ton nh phn -c dng khi lm cc php ton ny.

Nh- cc v d trn ch ra, cc h thc truy hi dng f(n) = af(n/b) + g(n) xut hin

trong nhiu bi ton khc nhau. Ta c th nh gi kch th-c ca cc hm tho mn mt h


thc truy hi nh- th. Gi s n = bk vi k l s nguyn d-ng. Khi :

n
f ( n ) af g ( n )
b
n
n
a 2 f 2 ag g( n )
b
b
n

n
n
a 3 f 3 a 2 g 2 ag g( n )
b
b
b
...
n k 1
n
ak f k a jg j
b j0
b

V b k 1 ta suy ra:
n

f ( n ) ak f (1 )

a
k 1
j0

n
g j.
b

Ph-ng trnh ny c th dng -c l-ng cc hm tho mn h thc chia tr.


nh l 5.3.1. Cho a, b l cc s nguyn ln hn 1, c l s thc d-ng. Gi s vi mi n

chia ht cho b hm f(n) l mt hm tng tho mn h thc truy hi:


n
f ( n ) af c
b

Khi :

O( n log b a )
f(n)
O(log n )

nu a 1
nu a 1

Chng minh. Tr-c tin gi s n = bk. T cng thc ca f(n) nhn -c trong phn bn

lun tr-c nh l ny vi g(n) = c, ta c:

f ( n ) a k f ( 1 ) a jc a k f ( 1 ) c a j
k 1

k 1

j 0

j0

Khi a = 1 ta -c f(n) = f(1) + ck. V n = bk nn k = logbn. V th


f(n) = f(1) + clogbn.
Khi n khng l lu tha ca b, ta c bk < n < bk+1, vi k l s nguyn d-ng. V f l hm

tng ta suy ra f(n) f(bk +1) = f(1) + c(k+1) = (f(1) + c) + ck (f(1) + c) + clogbn. Do , trong

c hai tr-ng hp, f(n) = O(logn) khi a = 1.

By gi gi s a > 1. Nu n = bk vi k l mt s nguyn d-ng, t cng thc tnh tng ca

cp s nhn suy ra:

c( a k 1)
a 1
c
c

a k f (1)

a 1 a 1

f ( n ) a k f (1)

C 1 n log b a C 2

c
c
V a k a log b n n log b a , trong C 1 f (1)
v C 2
a 1
a 1

Nu n khng l lu tha ca b, ta c b k n b k 1 , vi k l mt s nguyn d-ng. V f

l hm tng ta suy ra:

f ( n ) f ( b k 1 ) C 1 a k 1 C 2
(C1a )a

(C1a )n

log b n
log b a

C2

C2

v k lognn < k + 1.
Do vy, chng ta c f(n) = O ( n log b a ) .
n

V d 5.3.5. Cho f(n) = 5 f

+ 3 v f(1) = 7. Hy tm f(2k) trong k l s

nguyn d-ng. Hy nh gi f(n) nu f l hm tng.


Gii: T nh l 5.3.1, vi a = 5, b = 2 v c = 3 ta c, nu n = 2k th
c
c

f ( n ) a k f (1)

a
1
a

3 3
31 3

5 k 7 5 k .
4 4
4 4

Nh- vy, nu f l hm tng, th:


f ( n ) O ( n log

) O ( n log 2 5 )

V d 5.3.6. Hy -c l-ng s php ton so snh dng trong thut ton tm kim nh phn.
Gii: Trong v d 5.3.2 ta chng minh rng f(n) = f n + 2 khi n chn, cn f l s
2

php ton so snh cn dng trong tm kim nh phn trong mt dy c n. V th t nh l 5.3.1

ta suy ra f(n) = O(logn).

V d 5.3.7. Hy -c l-ng s php ton so snh dng trong thut ton cho trong v d

5.3.3 nh v cc s ln nht v nh nht.

Gii: Trong v d 5.3.3 ch ra rng f(n) = 2 f n + 2 khi n chn, cn f l s php


2

ton so snh cn dng trong thut ton ny. V th t nh l 5.3.1 suy ra:
f(n) = O ( n log 2 ) = O(n).
By gi chng ta s pht biu mt nh l tng qut hn v phc tp hn nh-ng rt c
ch khi phn tch phc tp ca thut ton chia tr.
nh l 5.3.2. Gi s f l hm tng tho mn h thc truy hi:

n
f ( n ) af cn d ,
b

n = b k,

trong k l s nguyn d-ng, a 1, b l s nguyn ln hn 1 cn c v d l cc s thc d-ng.

Khi :

O ( n d )
nu a b d

f ( n ) O (n d log n ) nu a b d

log a
nu a b d
O (n b )

V d 5.3.8. Hy -c l-ng s php ton nh phn cn dng khi nhn hai s nguyn n bit

bng thut ton nhn nhanh.

n
+ Cn, khi n chn, trong f(n) l s cc
2

Gii: V d 5.3.4 ch ra rng f(n) = 3f

php ton nh phn cn dng khi nhn hai s nguyn n bit bng thut ton nhn nhanh. V th, t
nh l 5.3.2 ta suy ra f ( n ) O( n log 3 ) . Ch l log3

1,6. V thut ton nhn thng

th-ng dng O(n2) php ton nh phn, thut ton nhn nhanh s thc s tt hn thut ton

nhn thng th-ng khi cc s nguyn l ln.


6. Nguyn l b tr
6.1. M u

Nh- ta bit, s cc phn t trong hp ca hai tp A v B bng tng cc phn t ca mi

tp tr i s phn t ca giao hai tp hp, tc l:

N(AB) = N(A) + N(B) - N(AB).


Cng thc xc nh s cc phn t ca hp hai tp hp th-ng -c dng trong nhiu bi

ton m.

V d 6.1.1. Trong k thi hc sinh gii cp thnh ph, mt tr-ng PTCS c 20 hc sinh t

gii mn Ton, 11 hc sinh t gii mn Vn, trong s c 7 em t gii ng thi c Vn v


Ton. Hi tr-ng c bao nhiu hc sinh t gii hc sinh gii?

Gii: Gi A l tp cc hc sinh t gii mn Ton, B l tp cc hc sinh t gii mn Vn.

Khi , tng s hc sinh t gii ca tr-ng l N(AB) v N(A B) l s cc hc sinh t gii


c hai mn Vn v Ton. Do vy,

N(AB) = N(A) + N(B) - N(AB) = 20 + 11 - 7 = 24.


V d 6.1.2. Xc nh s l-ng cc s nguyn d-ng nh hn hoc bng 1000 chia ht cho

9 hoc 11?

Gii: Gi A l tp cc s nguyn d-ng nh hn hoc bng 1000 chia ht cho 9, v B l

tp s cc s nguyn d-ng nh hn hoc bng 1000 chia ht cho 11. Khi A B l tp cc


s nguyn d-ng nh hn hoc bng 1000 chia ht cho 9 hoc 11 v A B l tp cc s nguyn

d-ng nh hn hoc bng 1000 chia ht cho c 9 v 11. Trong s cc s nguyn d-ng nh hn
hoc bng 1000 c [1000/9] s chia ht cho 9 v [1000/11] s chia ht cho 11. V 9 v 11 l hai

s nguyn t cng nhau nn s nguyn chia ht cho c 7 v 11 l s nguyn chia ht cho


9.11=99. S cc s ny l [1000/99]. T suy ra:
N(AB) = N(A) + N(B) - N(AB)
10 00 1 000 10 00

11 1 90 10 19 1
9 1 1 99

Tc l c 191 s nguyn d-ng nh hn hoc bng 1000 chia ht cho 9 hoc 11.
V d 6.1.3. Gi s mt tr-ng i hc c 1503 sinh vin nm th nht. Trong s c

453 sinh vin tham gia Cu lc b (CLB) tin hc, 267 sinh vin tham gia CLB ton hc v 99

sinh vin tham gia c hai CLB. Hi c bao nhiu sinh vin khng tham gia c CLB ton hc
cng nh- CLB tin hc?

Gii: S sinh vin khng tham gia CLB ton hc cng nh- CLB tin hc s bng tng s

sinh vin tr i s sinh vin tham gia mt trong hai CLB. Gi A l tp cc sinh vin nm th
nht tham gia CLB tin hc, cn B l tp cc sinh vin tham gia CLB ton hc. Khi ta c N(A)

= 453, N(B) = 267 v N(AB) = 99. S sinh vin tham gia hoc CLB tin hc hoc CLB ton hc
l:

N(AB) = N(A) + N(B) - N(AB) = 453 + 267 - 99 = 621.


Do vy c 1503 - 621 = 882 sinh vin nm th nht khng tham gia CLB ton cng nh-

tin hc.

Phn sau ca tit ny chng ta s -a ra cch tm s cc phn t ca hp mt s hu hn

cc tp hp.

6.2. Nguyn l b tr
Tr-c tin, chng ta s trnh by cng thc tnh s phn t ca hp 3 tp hp A, B, C.

Tr-c khi xy dng cng thc ny ta l-u rng N(A) + N(B) + N(C) m mt ln nhng phn
t ch thuc mt trong ba tp, m hai ln nhng phn t thuc ng hai trong ba tp, v m 3

ln nhng phn t thuc c ba tp.

Hnh 1. Hp ca 3 tp hp
loi b vic m trng lp cc phn t thuc nhiu hn mt tp cn phi tr i s cc

phn t thuc giao ca tt c cc cp ca ba tp hp ny, tc l ta c:


N(A) + N(B) + N(C) - N(A B) - N(A C) - N(BC).

Biu thc ny m mt ln cc phn t ch thuc mt trong ba tp hp cho. Cc phn

t xut hin trong ng hai tp cng -c m mt ln. Cc phn t thuc c ba tp ch-a m

ln no. Do vy, ta cn thm vo s cc phn t thuc giao ca ba tp (xem hnh v). Biu thc
cui cng ny tnh mi phn ng mt ln d n thuc mt hay hai hoc ba tp:

N(ABC)=N(A) + N(B) + N(C) - N(A B) - N(A C) - N(BC) +N(ABC).


V d 6.2.2. Bit rng c 1202 sinh vin hc ting Anh, 813 sinh vin hc ting Php, 114

sinh vin hc ting Nga, 103 sinh vin hc c ting Anh v ting Php, 23 hc c ting Anh v
ting Nga, 14 hc c ting Php v ting Nga. Nu tt c 2092 sinh vin u theo hc t nht mt
ngoi ng, th c bao nhiu sinh vin hc c ba th ting?

Gii: Gi E l tp cc sinh vin hc ting Anh, F l tp cc sinh vin hc ting Php, R l

tp cc sinh vin hc ting Nga. Khi :

N(E) = 1202

N(F) = 813 N(R) = 114

N(EF) = 103

N(ER) = 23

N(FR) = 14

N(S FR) = 2092.

Thay vo cng thc tng qut nu trn:


N(EFR) = N(E) + N(F) + N(R) - N(ER) - N(EF) - N(FR) + N(EFR )
ta nhn -c:

2092 = 1202 + 813 + 114 - 103 - 23 - 14 + N(EFR)


Gii ra ta -c: N(EFR) = 3.

Do vy c 3 sinh vin theo hc c ba th ting.

nh l 6.2.1. Nguyn l b tr. Cho A1, A2, , An l cc tp hu hn. Khi :

N ( Ai ) N ( Ai )
n

i 1

i 1

1 i j n

N ( Ai A j )

1 i j k n

N ( Ai A j Ak ) ....

(1) n 1 N ( Ai )
n

i 1

Chng minh. Chng ta s chng minh cng thc trn bng cch ch ra rng mi phn t

ca hp n tp hp -c m ng mt ln. Gi s a l phn t chung ca ng r tp trong cc


tp A1, A2, , An trong 1 r n. Phn t ny -c m C r1 ln trong tng N ( Ai ) . N
-c m C 2 ln trong N ( Ai A j ) . Tng qut n -c m:
r

C r1 C r2 .... ( 1 ) r 1 C rr
ln khi tnh gi tr v phi ca cng thc h s t hp ta c

C r0 C r2 .... ( 1) r C rr 0
V th:

1 C r0 C r1 C r2 .... ( 1 ) r 1 C rr 1

Do vy, mi phn t ca hp -c m ng mt ln khi tnh gi tr v phi ca cng

thc cho. Nguyn l b tr -c chng minh.

Nguyn l b tr cho ta cng thc tnh s phn t ca hp n tp hp vi mi n nguyn

d-ng. N gm c 2n - 1 s hng.

Nguyn l b tr c th -c ng dng v pht biu d-i dng sau :

H qu 6.2.1. Cho Ak , 1 k

n l cc tp con ca tp hu hn X tho mn tnh cht Ak


no , khi s phn t ca X khng tho mn bt c tnh cht Ak no l:
N(X) - N(A1A2...Am) = N - N1+ N2-....+(-1)m Nm
Trong Nk l s cc phn t ca X tho mn k tnh cht ly t m tnh cht cho.
ng dng h qu cho bi ton b th- sau:

V d 6.2.3. C n l th- v n phong b ghi sn a ch, b ngu nhin cc l th- vo cc

phong b. Hi xc sut xy ra khng mt l th- no b ng a ch l bao nhiu

Gii : C tt c n cch b th-. Vn cn li l m s cch b th- sao cho khng c l

th- no ng a ch. Gi X l tp hp tt c cc cch b th- v Ak l tnh cht l th- th k b


ng a ch. Khi theo h qu ca nguyn l b tr ta c:
N = N - N1 + N2 + ... + (-1)n Nn
Trong N l s cn tm, N = n cn Nk l s tt c cc cch b th- sao cho c k l thng a ch. Nhn xt rng, Cnk l tng theo mi cch ly k l th- t n l, vi mi cch ly k l
th- c (n-k) cch b k l ny ng a ch ta nhn -c:

N k C nk ( n k )!
v

n!
k!

1 1
( 1) n
N n! 1 ...
1! 2!
n!

T xc sut cn tm l
_

N
1 1
1
1 ..... (1) n
N
1! 2!
n!

Mt iu l th l xc sut ny dn n e-1 ( ngha l cn ln hn 1/3 ) khi n kh ln. S

N trong bi ton trn -c gi l s mt th t v -c k hiu l Dn.

Bi tp
1. Trong mt lp hc c 50 sinh vin, c 26 sinh vin t yu cu mn thi th nht, 21

sinh vin t yu cu mn thi th hai. Nu 17 sinh vin khng t yu cu c hai mn thi th c


bao nhiu sinh vin t yu cu c hai mn thi ?

2. Trong lp c 52 sinh vin, c 27 sinh vin khng phi thi li mn K thut s, 20

sinh vin khng phi thi li mn Ton ri rc. Nu c 15 sinh vin phi thi li c hai mn, th

c bao nhiu sinh vin khng phi thi li mn no c?

3. Trong mt tr-ng i hc c 18 sinh vin ton v 325 sinh vin tin hc.
a) C bao nhiu cch chn hai i din sao cho mt l sinh vin ton cn ng-i kia l

sinh vin tin hc?

b) C bao nhiu cch chn mt i din hoc l sinh vin ton hoc l sinh vin tin hc ?

4. C bao nhiu kh nng chn 2 s nguyn trong cc s t 1 n (n-1) sao cho tng

ca chng ln hn n ?

5. Xu thun nghch l mt xu m khi vit theo th t ng-c li cng bng chnh n.

Hy tnh s xu nh phn c di bng n l thun nghch ?

6. Mt l-i vung n x m mt con kin cn leo t im A ln im B. Mi ln leo ch b

dc theo cc cnh ca sang phi hoc ln trn. Hi c bao nhiu -ng c th xy ra ?


B (n,m)

A (0,0)
7. Mt l-i vung n x m (s hng m < n s ct) mt con kin cn leo t im A ln

im B. Mi ln leo ch b dc theo cc cnh ca : ln ng mt bc hoc sang phi (c th


nhiu ). Hi c bao nhiu -ng c th xy ra ?

B (n,m)

A (0,0)
8. Mt nhm sinh vin gm n nam v n n. C bao nhiu cch xp thnh mt hng sao

cho nam n ng xen nhau?

9. Mt cu lc b c 25 thnh vin
a) C bao nhiu cch chn 4 thnh vin vo u ban th-ng trc?
b) C bao nhiu cch chn ch tch, ph ch tch, th- k v th qu?

10. C bao nhiu xu nh phn cha ng nm s 0 v m-i bn s 1 v ngay sau mi s

0 nht thit l hai s 1?

11. Trong tt c cc s c 5 ch s c bao nhiu s m ch s ng bn tri ln hn hoc

bng ch s bn phi ?
nht ?

12. C bao nhiu kh nng chia 2n ng-i thnh n cp, ngi i din hai bn mt bn ch
13. C bao nhiu kh nng chn 2 s nguyn trong 100 s nguyn u tin sao cho tng

ca chng l s chn, s l ?

14. a) Ch ra rng trong 5 s chn t 8 s nguyn d-ng u tin nht thit phi c mt

cp c tng bng 9 ?

b) iu khng nh trong cu a) c ng khng nu chn 4 ch khng phi 5 s.


15. a) Ch ra rng trong 7 s chn t 10 s nguyn d-ng u tin nht thit c t nht hai

cp c tng bng 11.

b) iu khng nh trong cu a) c ng khng nu thay cho 7 ta chn 6 s.


16. Chng minh cng thc sau :

C
17. Chng minh cng thc sau :

k
n 1

(n 1)C nk 1

C nr C kr C nk C nr kk

trong n, r, k l cc s nguyn khng m vi r n v k r.

18. Tm cng thc tnh h s ca x trong khai trin ca x


x

100

nhin.

2 1
19. Tm cng thc tnh h s ca x trong khai trin ca x
x

100

nhin

trong k l s t

trong k l s t

20. Tnh tng cc h s ca cc s hng cha s m chn ca x trong biu thc :


(x+y+z) n
21. Tnh tng cc h s ca cc s hng cha s m l ca x trong biu thc :

(x+y+z) n
22. Chng minh cng thc sau :
k 2
n 1
k ( C n ) nC 2 n 1
n

k 1

23. Ph-ng trnh


x1 + x2 + x3 + x4 + x5 +x6 = 29
c bao nhiu nghim nguyn khng m sao cho :
a) xi >= 1
b) xi >= i vi i=1...4 x5 >5 v x6 >=6
c) x1 <= 5
d) x1 < 8 v x2 >8
24. Bt ng thc :
x1 + x2 + x3 <= 11
c bao nhiu nghim nguyn, khng m.
25. Ch ra rng c t nht hai tp con khc nhau gm 5 phn t ca tp 10 s nguyn

d-ng khng v-t qu 50 c cng tng.

26. C bao nhiu xu 20 ch s ca h thp phn cha ng hai ch s 0, bn ch s 1,

ba ch s 2, mt ch s 3, hai ch s 4, ba ch s 5, hai ch s 7 v ba ch s 9?

27. C bao nhiu xu nh phn khc nhau nu chng -c bt u bng bit 1 v cha

thm ba bit 1 na, n cn cha tt c 12 bit 0 v sau mi bit 1 c t nht hai bit 0?

28. Ch ra rng nu c nm im phn bit trong mt hnh vung cnh bng 2 th c t

nht hai trong 5 im ny c khong cch khng xa hn

2.

29. Ch ra rng trong dy m s nguyn tn ti mt hay nhiu hn cc s hng lin tip c

tng chia ht cho m.

30. Khi chn tu n+1 s nguyn d-ng khc nhau, chng minh rng tn ti t nht 2 s

ng d- theo modulo n.

31. Ch ra rng trong mt nhm c 5 ng-i (trong hai ng-i bt k hoc l bn hoc l

th) lun c ba ng-i l bn hoc bn ng-i l k th ln nhau v c nhm ba ng-i l k th


hoc bn ng-i l bn ca nhau.

32. Cho (Xi, Yi ,Zi ) i=1,,9 l to nguyn ca 9 im trong khng gian ba chiu.

Chng minh rng tn ti t nht mt on thng ni 2 im trong s c trung im to


nguyn.

33. Mt ng-i li xe but tr thu cu -ng bng cch th vo cc my thu thu t

ng ln l-t cc ng 5 xu v 10 xu.

34. Tm h thc truy hi tnh s cch khc nhau m ng-i li xe c th tr mt khon thu

n xu (trong trt t theo cc ng xu -c th vo my l quan trng).


a) Tm cc iu kin u.

b) C bao nhiu thng bo khc nhau m ng-i li xe c th tr mt khon thu 45 xu?


35. Tm h thc truy hi m Rn tho mn, trong Rn l s min ca mt mt phng b

phn chia bi n -ng thng nu khng c hai -ng no song song v khng c ba -ng no
cng i qua mt im. Tnh Rn bng ph-ng php lp.

36.a) Tm h thc truy hi m R n tho mn, trong Rn l s min ca mt mt cu b

phn chia bi n -ng trn ln (tc l giao ca mt cu vi cc mt phng i qua tm mt cu)


nu khng c ba -ng no trong s n -ng trn ny cng i qua mt im.
b) Tnh Rn bng ph-ng php lp.
37.a) Tm h thc truy hi m Sn tho mn, trong Sn l s min trong khng gian ba

chiu b phn chia bi n mt phng nu mi b 3 mt phng gp nhau ti mt im, nh-ng


khng c b 4 mt phng no i qua cng mt im.
b) Tnh Rn bng ph-ng php lp.
38. Gi s s tm hm b nh bt trong mt nm bng trung bnh cng s b nh bt

trong hai nm tr-c .

a) Hy tm h truy hi cho Ln, trong Ln l s tm b nh bt trong nm th n.


b) Hy tm Ln nu nm u 100 000 tm hm b nh bt, nm th hai 300 000 tm hm

b nh bt.

39. Kt qu mt cuc iu tra mc sng ca cc gia nh M cho bit 96% c t nht

mt my thu hnh, 98% c in thoi, v 95% c in thoi v t nht mt my thu hnh. Tnh t
l phn trm cc gia nh M khng c in thoi hoc khng c my thu hnh.
40. C bao nhiu s nguyn d-ng nh hn 200 l:
a)

Lu tha bc hai hoc cao hn ca s nguyn?

b)

Lu tha bc hai hoc cao hn ca s nguyn hoc ca s nguyn t?

c) Khng chia ht cho bnh ph-ng ca mt s nguyn ln hn 1?


d) Khng chia ht cho lp ph-ng ca mt s nguyn ln hn 1?
e) Khng chia ht cho t nht ba s nguyn t?

Ch-ng III.

th v ng dng
Nhng t-ng c bn ca l thuyt th -c xut t th k 18, bt u t bi bo

ca Euler cng b nm 1736 lin quan n li gii bi ton ni ting v cc cy cu


Konigsberg. Tuy nhin, cho ti nay, mi quan tm n l thuyt th vn khng h suy gim.

L do ca s quan tm y chnh l do nhng ng dng ht sc rng ri ca th trong rt nhiu


lnh vc khc nhau, bao gm c tin hc, ho hc, vn tr hc, k thut in, ngn ng v kinh
t Trong ch-ng ny chng ta s cp n cc khi nim c bn ca l thuyt th v mt

s bi ton trn th, ng thi xt n mt s dng c bit ca th l cy v mng.


1. Cc khi nim c bn
1.1. Khi nim v thut ng

nh ngha 1.1.1: th v h-ng G l mt cp (V,E) gm tp V m cc phn t ca n

-c gi l cc nh v tp E cc cp khng th t ca cc nh thuc V -c gi l cc cnh.

Nu ch c mt cnh eE duy nht ni hai nh v, w ta vit e = (v,w) hoc e


= (w, v). Theo ngha , cp khng th t (v, w) xc nh mt cnh trn th v
h-ng G.
nh ngha 1.1.2: th c h-ng G l mt cp (V, E) gm cc nh V v tp E cc cnh

(cc cung) sao cho mi cnh e E lin thuc mt cp c th t cc nh thuc V. Ta cng vit e
= (v, w) nu e ni cp c th t hai nh (v, w). Cnh e trn th (c h-ng hoc v h-ng) ni

hai nh v, w -c gi l lin thuc hai nh v v w; cc nh v v w -c gi l cc nh k


nhau v cn -c gi l cc im u mt ca cnh e. Nu (u, v) l mt cnh c h-ng th u l

im u v v l im cui hoc cng c th ni cung (u,v) i ra khi nh u v i vo nh v.


Ta s lun gi thit V, E l cc tp hu hn v V l tp khng rng.

v1

V d 1.1.1: Trn hnh 1 l th

v h-ng. Tp cc nh l v1, v2 , v3 ,
v4 , v5 , v6 , v7 ; tp cc cnh l e1, e2

e7

, e3 , e4 , e5 , e6, e7, e8, e9 ; trong


cnh e1 l cnh lin thuc hai nh

(v1, v2), cnh e2 l cnh lin thuc hai


nh (v2 ,v3)

v6

e9

e1
e5

v5

V2

V6

e4

Hnh 1.

e2

e6

v4

e3

v3
e8

Cn trn hnh 2 l mt th c h-ng. Trn th , cnh

e1 ni cp nh (v1 ,v2), cnh e2 ni cp nh (v2 ,v1), cn cnh

v1

(c h-ng hoc v h-ng) -c gi l mt khuyn ca th.

e4

e5 ni nh v4 vi chnh n. Nhng cnh kiu nh- e5 trn th

e1

v2

e2

e3
v4

v3

nh ngha 1.1.1 nu trn hon ton cho php c th c

nhng cnh khc nhau lin thuc vi cng mt cp nh (cc

cnh song song vi nhau). Cng c th tn ti nhng nh khng

Hnh .2

c cung no lin thuc vi n (nh c lp - v6 trn hnh 1).

e5

th khng c cc cnh song song v khng c khuyn -c gi l n th. Trong hnh 1 e8 l


mt khuyn ca nh v3; e5 v e7 l 2 cnh song song, v6 l nh c lp. D-i y chng ta s
xt mt vi v d minh ho cc ng dng ca th.

V d 1.1.2: th cnh tranh trong sinh thi hc. S t-ng tc ca cc loi vt trong mi

tr-ng t nhin c th -c m hnh ho bng th v h-ng. Trn th mi loi -c


biu din bng 1 nh, hai nh t-ng
ng vi hai loi cng chung ngun

Sc

Chut

thc n (cnh tranh nhau) -c ni

vi nhau bi mt cnh. Hnh 3 l mt


minh ho th sinh thi hc ca mt
khu rng.

Chut tr

Nhn vo ta thy C,

Qu

Ct

Hnh 3. th sinh thi

Qu, Ct hoc Chut v Chut tr

l cnh tranh vi nhau cn chut v sc th khng.

V d 1.1.3: th nh h-ng. Cc nh tm l hc rt ra rng trong mt tp

th tnh cch ca mt s ng-i ny c th nh h-ng n mt s ng-i khc. S nh

h-ng c th -c m hnh ho bng mt th c h-ng. Mi ng-i ca nhm -c


biu din bng mt nh. Nu ng-i -c biu din bng nh a c nh h-ng n ng-i
-c biu din bng nh b th hai
nh a, b -c ni bng mt cnh

Lan

Tuyt

c h-ng (a, b). Hnh 4 l mt v


d v th nh h-ng ca mt

nhm Lan c nh h-ng ln Tuyt Ngn


v Ngn nh-ng khng ai nh
h-ng ti Lan, cn Thu v
Nhung c th nh h-ng ln nhau.

Thu
Hnh 4.

Nhung

i 1

V d 1.1.4: th thi u vng trn. Mt cuc

thi u th thao trong tng i phi gp tt c cc


i tham d mi i ng mt trn c th -c m
hnh ha bng th c h-ng. Trong mi i

i 2

-c biu din t-ng ng bng 1 nh. Mt cnh c i 3


h-ng i t nh a n b, k hiu l (a, b), nu i a
thng i b. Trong v d hnh 5 i 1 thng tt c
cc i cn li, cn i 5 khng thng trn no, i

i 4

3 thua i 1, i 4 thng i 3, i 5...

Phn cui ca mc ny chng ta s

i 5

Hnh 5.

cp thm mt s thut ng c bn ca l thuyt th. u tin ta xt cc th v


h-ng.
nh ngha 1.1.3: Bc ca mt nh v (k hiu l deg (v)) trong th v h-ng

l s cc cnh lin thuc vi n, nhng khuyn ti mt nh -c tnh 2 ln cho bc ca


n .
V d 1.1.5: Bc ca cc nh trong th hnh 1 t-ng ng l:

deg (v1) = 3, deg (v2) = 3, deg (v3) = 4, deg (v4) = 3, deg (v5 ) = 4, deg (v6) = 0, deg (v7) = 1.
Nh- vy, nh c lp cng c th nh ngha l nh c bc bng 0.
nh c bc bng 1 -c gi l nh treo, tc l nh lin k (lin thuc) vi ng 1 nh

khc (nh v7 trong hnh 1 l mt v d v nh treo).

Mi cnh (u,v) ca th -c tnh mt ln trong bc ca u v 1 ln trong bc

ca v. Do vy nu ta cng bc tt c cc nh ca mt th ta s -c hai ln s cnh


ca th y. Tc l ta c:
nh l 1.1.1: Cho G = (V, E) l mt th v h-ng c m cnh. Khi :

2m deg v
vV

T nh l trn ta suy ra.


H qu: Trong th v h-ng s nh c bc l l mt s chn.
Chng minh: Tht vy nu chia cc nh ca th thnh hai tp: Tp v1 l cc nh c bc

chn, v2 l cc nh c bc l. Ta c.

2m deg v deg v deg v


vV

vV1

vV2

Do deg (v) l chn vi v V1 nn tng th nht trong v phi trn l s chn. Suy ra s s

hng ca tng th hai phi l s chn v mi s hng ca n l s l.

V cc cnh ca th c h-ng l cc cp c th t cho nn khi nim bc ca nh -c

thay i nh- sau.

nh ngha 1.1.4: Trong th c h-ng bc vo ca nh v k hiu l deg-(v) l s cnh

ca th nhn v l nh cui (nh vo), bc ra ca v k hiu deg+(v) l s cnh nhn v l nh

u (nh ra).

Theo nh ngha 1.1.4 mt khuyn trn th c h-ng s ng gp 2 n v vo bc-

vo v bc-ra ca nh (t-ng ng).

V d 1.1.6: Bc vo, bc ra ca th trn hnh 2 t-ng ng l:


v1= (1,2), v2= (2,1), v3= (1,0), v4 = (1,1).
V mi cnh c mt nh u v mt nh cui nn tng cc bc-vo v tng cc bc-ra ca

tt c cc nh trong th c h-ng l nh- nhau v bng s cnh ca n. Ngha l ta c:


nh l 1.1.2: Gi s G (V, E) l mt th c h-ng c m cnh. Khi

m deg v deg v
vV

vV

Nhiu tnh cht ca th c h-ng khng ph thuc vo h-ng trn cc cnh ca n. Do

vy, trong nhiu tr-ng hp s thun tin hn nu ta b qua cc h-ng ca th. th v


h-ng nhn -c t th c h-ng bng cch b qua h-ng ca cc cnh -c gi l th v
h-ng t-ng ng ( th v h-ng nn) ca th c h-ng cho.
1.2. -ng i. Chu trnh. th lin thng
nh ngha 1.2.1: Gi s n l mt s nguyn d-ng. -ng i di n trong th t

nh u ti nh v l mt dy lin tip cc cnh e1, e2,, en ca th sao cho: ei = (vi-1 ,vi), vi


V, v0 = u, vn = v.

Khi th G l th n ta c th biu din -ng i ny bng dy cc nh u = v0, v1

, vn = v. -ng i -c gi l mt chu trnh nu n bt u v kt thc ti cng mt nh, tc

l u = v. -ng i hoc chu trnh -c gi l


n nu khng c cnh no c mt trong

-ng i hoc chu trnh y qu 1 ln.

V d 1.2.1: Trong th hnh 1 a, b, c,

d l mt -ng i di 3 v (a, b), (b, c),


(c, d) u l cc cnh ca th. Tuy vy a,

b, c, e khng phi l -ng i v (c, e) khng


phi l mt cnh ca th. -ng i a, b,

d
Hnh 1

d, e, a, b, di 5 khng l -ng i n v cnh (a, b) c mt trong -ng i 2 ln. Cn a, b,


c, d, b, e, a, l mt chu trnh v n l -ng i bt u v kt thc nh a.

V d 1.2.2: Trong th c h-ng hnh 2

th a, b, c, d l mt -ng i, cn a, b, c, e li

khng phi l mt -ng i v (d, e) khng phi

l 1 cnh ca th (cnh ni 2 nh e v d l
cnh (e, d)!). Cng trn th a, b, c, d, b, e,

a hoc a, b, e, a l cc chu trnh.

Hnh 2

Nh- vy, trn cc th c h-ng, khi xt

cc -ng i v cc chu trnh ta phi c bit ch n h-ng ca cc cnh ca chng.


Khi xt mt mng my tnh, mt cu hi thc t t ra l liu mi cp my tnh trong mng

u c th trao i thng tin vi nhau bng cch gi cc thng ip cho nhau mt cch trc tip
hoc qua cc my trung gian hay khng? Nu biu din mng my tnh bng th, trong
mi my -c biu din bng mt nh, cn mi -ng truyn thng -c biu din bng mt

cnh th cu hi ni trn theo ngn ng th s l: C lun tn ti -ng i gia hai nh trong


1 th bt k khng? D dng thy cu tr li y l khng.

nh ngha 1.2.2: Mt th v h-ng -c gi l lin thng nu lun c mt -ng i ni

hai cp nh phn bit bt k ca th.

Nh- vy hai my tnh bt k trong mng ch c th truyn -ng thng -c vi

nhau khi v ch khi th t-ng ng vi mng l th lin thng.

V d 1.2.3: th hnh 2 l th lin thng, tuy nhin th hnh 3 l khng lin

thng v khng c -ng i no ni hai nh a vi e.

nh ngha 1.2.3: th H = (W, F) -c gi l th con ca th G = (V, E) nu W

V, F E.

Trong tr-ng hp th G khng lin thng th n c th chia thnh mt s cc


th con lin thng i mt khng c
a
b

nh chung. Cc th con lin thng


ri nhau nh- vy -c gi l cc thnh

phn lin thng ca th ang xt.


Nh- vy th lin thng l th ch
c mt thnh phn lin thng.

V d 1.2.4: th hnh 3 c hai

thnh phn lin thng.

Hnh 3

i khi vic xo i mt nh v tt c cc cnh lin thuc vi n s to ra mt


th con mi c nhiu thnh phn lin thng hn so vi th ban u. Cc nh nh- vy
-c gi l cc nh ct (nh r nhnh, im khp). Nh- vy, vic xo nh r nhnh

khi mt th lin thng s to ra mt th con khng lin thng. T-ng t nu vic


xo mt cnh no trn th m lm cho c nhiu thnh phn lin thng hn th
cnh y -c -c gi l mt cu (cnh ct).

V d 1.2.5: Cc nh r nhnh ca th trn hnh 4 l b, c, v e; cc cu ca th l

cc cnh (a, b) v (c, e).

i vi th c h-ng ta c hai khi nim lin

thng tu thuc vo vic ta c quan tm ti h-ng ca


cc cnh khng.

nh ngha 1.2.4: th c h-ng -c gi l lin

thng mnh nu lun c mt -ng i ni hai nh bt


k ca th. th c h-ng -c gi l lin thng

Hnh 4

yu nu th v h-ng t-ng ng vi n l th v
h-ng lin thng.

C th thy rng nu th c h-ng l lin thng mnh th n cng lin thng yu, song

iu ng-c li khng phi bao gi cng ng.

c
e

e
Hnh 5

V d 1.2.6: th G v H trn hnh 5 u l lin thng yu. Tuy nhin ch c G l lin

thng mnh, cn H khng phi l lin thng mnh. Tht vy, trn th H khng c -ng i
(c h-ng) no ni a vi tt c cc nh cn li.
1.3. Mt s dng th n c bit
D-i y l mt vi lp th n th-ng gp trong cc ng dng.
th y . th y n nh, k hiu l K n l mt n th cha ng 1

cnh ni mi cp nh phn bit. Cc th Kn . vi n=1, 2, 3, 4, 5, 6 -c biu din trn

hnh 1.

Hnh 1. Cc th Kn , 1 n 6 .

Chu trnh ( th vng). Chu trnh Cn, n 3 l mt th c n nh v1 , v2 , , vn v

n cnh (v1 , v2 ), (v2 , v3 ), , (vn-1 , vn ), (vn , v1 ). Cc chu trnh C3, C4, C5, C6 biu din trn hnh 2.

Hnh 2. Cc chu trnh Cn , 3 n 6 .


th hnh bnh xe. Khi thm 1 nh vo chu trnh Cn vi n3 v ni nh ny vi mi

mt nh ca Cn bng nhng cnh mi, ta s nhn -c th hnh bnh xe. Cc bnh xe W3,

Hnh 3. Cc bnh xe W n , 3 n 6 .
W4, W5, W6 biu din trn hnh 3.

th khi n chiu. th khi n chiu (cc khi n chiu) k hiu l Qn, l

cc th c 2 n nh mi nh biu din bng xu nh phn di n. Hai nh l lin

k nu v ch nu cc xu nh phn biu din chng khc nhau ng 1 bit. Cc th


Q1 , Q2 , Q3 -c biu din trn hnh 4.

11

10
1

0
Q

01

00
Q

10

11
10

11

00

01
00

01

Hnh 4. Cc khi n chiu Qn , 1 n 3 .


th phn i ( th hai

pha). Mt th n G -c gi l

th phn i nu tp cc nh V c th
phn thnh 2 tp con khng rng ri

nhau V1 v V2 sao cho mi cnh ca


th ch ni mt nh ca V1 vi mt

nh ca V2 . V d, th biu din

V2
v
v
v

V1
v
v
v

Hnh 5. Minh ho C6 l th phn i

quan h hn nhn ca mt lng, mi ng-i biu din bng mt nh, cn cnh biu th quan h
v chng gia hai ng-i. Tp cc nh c th chia thnh hai tp con, mt tp gm n ng v
mt tp gm ton n b.

th phn i y ( th hai pha y ). th phn i y K m,n l th

c tp nh -c phn thnh hai tp con t-ng ng c m nh v n nh v c mt cnh gia


hai nh khi v ch khi mt nh thuc tp con ny v nh th hai thuc tp con kia. Cc
th phn i y K2,3 , K3,3 , K3,5 v K2,6 -c biu din trn hnh 6.

K2,

K3,

K3,

K2,

Hnh 6. Mt s th phn i y

2. Biu din th
C nhiu cch biu din th. Trong cc mc trn chng ta dng hnh hc

biu din th. Cng c th biu din th bng cch lit k tt c cc nh v cc cnh.

Trong mc ny chng ta s cp thm hai ph-ng php biu din th na bng cch s dng
ma trn. Tu thuc vo tng bi ton c th m la chn cch biu din th cho ph hp.

i khi hai th c dng ng nh- nhau theo ngha tn ti php t-ng ng mt - mt gia

cc nh ca chng m vn bo tn cc cnh. Trong tr-ng hp y ta ni rng hai th l


hai th ng cu vi nhau. Vic xc nh hai th c ng cu vi nhau khng l mt bi
ton quan trng ca l thuyt th cng s -c nghin cu trong tit ny.
2.1. Ma trn k, ma trn trng s.
Gi s G = (V, E) l mt th, trong V l tp nh -c lit k mt cch tu v1, v2,

vn . Ma trn k th G k hiu l A = [aij] -c xc nh nh- sau: aij = k. Trong k l s


cnh ni hai nh vi v vj ca th. Nh- vy nu G c n nh th ma trn A l ma trn vung

cp n. Cng c th thy rng ma trn k ca th v h-ng l ma trn i xng, cn ma trn k


ca th c h-ng ni chung khng phi l i xng. Ma trn k l ma trn 0-1 (cc phn t
ch gm cc s 0 v 1) nu th t-ng ng l n th. Ta cng c th thy tng cc gi tr
theo hng (ct) ca ma trn k chnh l bc ca nh t-ng ng trn th.

V d 2.1.1: Ma trn k ca cc th hnh 1 (tit 1.1), hnh 1 v 2 (tit 1.2) t-ng ng l


cc ma trn sau:

v1
v1 0
v2 1
v3 0

v4 0
v5 2

v6 0
v7 0

v2

v3

v4

v5

v6

1
0

0
1

0
1

2
0

0
0

1
0

1
0

0
1

1
0

0
0

0
0

0
0

0
0

0
1

0
0

v7
0
0
0

0
1

0
0

a b c d e
a 0
b 1
c 0

d 0
e 0

a b c d

1 0 0 0

0 1 1 1
1 0 1 0

1 1 0 1
1 0 1 0

a 0
b 0
c 0

d 0
e 1

1 0 0 0

0 1 0 1
0 0 1 0

1 0 0 0
0 0 1 0

Ng-i ta chng minh -c rng nu A l ma trn k ca th v h-ng v


phn t t-ng ng ca ma trn Ap = AA (p tha s) th

a ijp

a ijp

s cho ta s -ng i di p

khc nhau i t nh vi ti nh vj.


Trong nhiu bi ton ng dng, mi cnh e = (u, v) ca th -c gn mt gi tr c(e)

no (cn vit l c(u, v)) gi l trng s ( di) ca cnh e. th trong tr-ng hp ny -c


gi l th c trng s. biu din th n c trng s ng-i ta cng c th dng ma trn

-c xy dng t-ng t nh- ma trn k, c th nh- sau:

Gi s G = (V, E) l n th c trng s v V = {v1, v2, vn}. Khi ma trn trng

s C l ma trn vi cc phn t.

c vi , v j nu vi , v j
Cij
nu vi , v j E

Trong gi tr ca , tu tng tr-ng hp c th, c th t bng mt trong cc gi tr


0, , .
Cch biu din th bng ma trn k (hoc ma trn trng s) c -u im l xc nh

2 nh u v v c phi l hai nh k nhau trn th hay khng ta ch phi thc hin mt php so
snh. Tuy nhin, khng ph thuc vo s cnh ca th, cch biu din ny lun i hi phi
dng n2 n v b nh trong my tnh l-u tr n (n = s nh ca th).
2.2. Ma trn lin thuc.

Mt cch biu din th th-ng dng khc l dng ma trn lin thuc. Gi s G = (V, E)

l mt th v h-ng, V= {v1 , v2, vn }, E = {e1, e2en} khi ma trn lin thuc theo th t
trn ca V v E l ma trn

1
mij
0

M = [mij] trong :

Nu cnh ej ni nh vi

Nu cnh ej khng ni vi nh vi

V d 2.2.1: Ma trn lin thuc ca th trn hnh 1 (tit 1.1) l ma trn sau:
v1

e1
1

e2
0

e3
0

e4
0

e5
1

e6
0

e7
1

e8
0

e9
0

v4
v5

0
0

0
0

1
0

1
1

0
1

1
0

0
1

0
0

0
1

v2
v3

v6
v7

1
0

1
1

0
0

0
0

0
1

0
0

0
0

0
0

0
0

0
0

1
0

0
0

0
0

0
0

0
1

0
0

0
0

0
1

C th thy rng mt khuyn ca th t-ng ng vi 1 ct ch c 1 phn t bng 1 trong

ma trn lin thuc; cn 2 cnh song song t-ng ng vi 2 ct ging nhau trong ma trn lin

thuc. Nu th khng c khuyn th tng cc phn t trn mt hng ca ma trn lin thuc
chnh l bc ca nh t-ng ng trn th.
2.3. S ng cu ca cc th
Trong ho hc th th-ng -c dng to m hnh cc hp cht. Cc hp cht khc

nhau c th c cng cng thc phn t nh-ng cu trc c th khc nhau. Khi y cc th t-ng

ng vi chng khng th v -c bng cng mt cch. Chnh v vy, th biu din cc hp


cht bit c th -c dng xc nh mt hp cht no c phi l hp cht mi hay
khng. Cc th c cng cu trc -c gi l cc th ng cu. Chnh xc hn ta c:

nh ngha 2.3.1. Cc n th G1=(V1, E1) v G2 =(V2, E2) -c gi l ng cu vi

nhau nu tn ti mt song nh f t V 1 ln V2 bo ton quan h lin k gia cc nh: nu a v b l


hai nh lin k trong G1, th f(a) v f(b) cng l hai nh lin k trong G2. Hm f -c gi l

mt php ng cu.

V d 2.3.1. Chng t cc th G=(U,E) v H=(V,F) d-i y l cc th ng cu.

u1

u3

u2

u4

v1

v2

v3

v4

Hnh 1. G v H l hai th ng cu

Gii: C th xy dng song nh f nh- sau:

f (u1 ) v1 , f (u 2 ) v4 , f (u3 ) v3 , f (u4 ) v2 .

D dng kim tra -c f bo ton tnh cht lin k ca cc nh. Do vy G v H l hai

th ng cu.

Nhn chung, vic xc nh s ng cu gia hai th l mt bi ton khng n gin, v

gia hai th c cng n nh tn ti ti n! song nh gia hai tp nh. Tuy nhin, trong thc t,

c th xc nh hai th khng ng cu vi nhau bng cch ch ra chng khng c chung mt

tnh cht no m cc th ng cu cn phi c. Tnh cht nh- th -c gi l mt bt bin


qua php ng cu. S l-ng nh, s l-ng cnh, bc ca cc nh l cc bt bin nh- vy.
V d 2.3.2. Chng t hai th sau khng ng cu vi nhau:

Hnh 2. Hai th khng ng cu vi nhau


Gii: Hai th trn cng c 4 nh v 4 cnh, song th th hai c mt nh bc 3 cn

th th nht tt c cc nh u c bc bng 2.

Mc d s nh, s cnh, bc ca nh l cc bt bin i vi php ng cu, song ngay

c khi cc i l-ng y nh- nhau th ch-a chc cc th l ng cu. V d d-i y chng


t iu .

V d 2.3.3. Cc th sau c ng cu vi nhau khng?

Hnh 3. Hai th khng ng cu


Gii: C hai th u c 4 nh, 5 cnh, 2 nh bc 2, 2 nh bc 3. Tuy nhin, c th

thy rng mi nh bc 2 ca th th nht lin k vi tt c cc nh bc 3, cn cc nh bc 2


ca th th hai li khng c tnh cht . Do vy hai th trn khng ng cu vi nhau.

3. Cc thut ton tm kim trn th


Nhiu ph-ng php gii cc bi ton trn th -c xy dng trn c s ln l-t duyt

tt c cc nh ca th. Do vy cn thit phi c cch thc duyt mt cch h thng cc


ph-ng n c th c. Cc cch duyt nh- vy -c gi l cc thut ton tm kim trn th.

D-i y chng ta s cp ni dung chnh ca hai thut ton tm kim ph bin trn th: tm

kim theo chiu rng v tm kim theo chiu su. Vic chi tit ha hai thut ton ny ty thuc
vo tng yu cu tm kim c th.

3.1. Tm kim theo chiu su


Thut ton bt u tm kim t mt nh v0 no ca th. Sau chn u l mt nh

k ty vi v0 v lp li qu trnh i vi u. b-c tng qut, gi s ta ang xt nh v. Nu


trong s cc nh k vi v tm -c nh w cn ch-a -c xt th ta s xt nh ny (n s tr

thnh nh -c xt) v bt u t n ta tip tc qu trnh tm kim. Trong tr-ng hp khng


cn nh no k vi v m ch-a -c xt th ta ni rng nh ny duyt xong v quay tr li

tip tc tm kim t nh m tr-c ta n -c nh v. Nu nh v0 -c duyt xong (v=v0)


th qu trnh tm kim kt thc.

V d 3.1.1. Xy dng thut ton kiu tm kim theo chiu su tm -ng i gia 2 nh s

v t ca th G cho tr-c.

Input: Cho th G vi tp nh V -c sp xp theo mt th t no v1, v2, , vn v

hai nh s, t trong s cc nh ca V.

Output: Tm -ng i gia 2 nh s v t.


1. [Khi to] t v0:=s. K hiu S :={v0} l tp cc nh -c xt.
2. [nh k tip theo] Chn v l nh cui cng -c -a vo S. Trong s cc nh
ch-a -c xt ca V chn nh vk k vi v sao cho k nh nht c th -c v t
S : S {vk } ri chuyn ti b-c 3. Nu khng tm -c nh vk no nh- th th
chuyn ti b-c 4.

3. [Tm thy -ng i] Nu vk t , kt thc thut ton: c -ng i t s ti t (danh


sch trong S l -ng i ). Trong tr-ng hp vk t , quay tr li b-c 2.

4. Nu v v0 , dng thut ton: khng c -ng i gia 2 nh s, t. Trong tr-ng hp


ng-c li, t S : S \{v} v quay li b-c 2.

V d 3.1.2. -ng i t nh a ti nh d trn th hnh 1 -c xy dng bng thut

ton tm kim theo chiu su l a, b, c, d (cc cnh nm trn -ng i -c t m).

e
Hnh 1. -ng i t a ti d xy dng
bng thut ton tm kim theo chiu su
3.2. Tm kim theo chiu rng
Thut ton bt u tm kim t mt nh v0 no ca th. b-c tng qut, gi s ta

c tp S l tp cc nh -c xt v T l tp nh -c duyt xong (lc u


S : v0 , T : ). Ta -a vo T tt c cc nh ca S tha mn iu kin chng khng c nh

k no na trong G m cn ch-a -c xt. Tip thay th S bng tp cc nh khng thuc T


nh-ng k vi S . Qu trnh -c thc hin cho ti khi tp S l tp trng.

V d 3.2.1. Xy dng thut ton kiu tm kim theo chiu rng xc nh xem c tn ti

-ng i gia 2 nh s v t ca th G cho tr-c hay khng?

Input: Cho th G vi tp nh V -c sp xp theo mt th t no v1, v2, , vn v

hai nh s, t trong s cc nh ca V.

Output: Xc nh xem c tn ti -ng i gia 2 nh s v t hay khng?

1. [Khi to] t v0:=s. K hiu S :={v0} l tp cc nh -c xt; T : l tp


cc nh -c duyt.

2. [Khi to tp cc nh trung gian] t S ' :


3. [nh -c duyt tip theo] Chn v l nh c th t nh nht trong S. t
T : T {v}, S : S \ {v} . Gi S(v) l tp cc nh khng thuc T nh-ng k vi v.

Nu t S ( v) th kt thc thut ton: C -ng i t s ti t. Trong tr-ng hp ng-c

li chuyn ti b-c 4.

4. [B sung nh k mi] t S ' : S ' S (v) . Nu S th chuyn ti b-c 5.


Trong tr-ng hp ng-c li: S th quay li b-c 3.

5. [Khng c -ng i?] Nu S ' th kt thc thut ton: Khng c -ng i ni 2


nh s v t. Trong tr-ng

hp ng-c li t S : S '

ri quay li b-c 2.

Nhn xt: Vi mt vi thay i

nh trong thut ton trn, ta cng c th

xc nh -c -ng i (nu n tn ti)

ni 2 nh s v t.

V d 3.2.2. -ng i t nh a

ti nh d trn th hnh 1 -c xy

dng bng thut ton tm kim theo

Hnh 1. -ng i t a ti d xy dng


bng thut ton tm kim theo chiu rng

chiu rng l a, e, d (cc cnh nm trn


-ng i -c t m).

4. th Euler v th Hamilton
Mt s bi ton thc t nh- bi ton ng-i -a th-, bi ton ng-i du lch dn n

vic nghin cu mt s dng c bit ca th l th Euler v th Hamilton. Trong phn


ny thut ng th -c dng ch chung a th v h-ng hoc c h-ng, cn thut ng
cnh dng ch chung cnh ca th v h-ng hoc cung ca th c h-ng.

4.1 -ng i Euler v chu trnh Euler


Thnh ph Konigsberg thuc Ph (by gi gi l Kaliningrad thuc Cng ho Nga),

-c chia thnh bn vng bng cc nhnh sng Pregel. Cc vng ny gm hai vng bn b
sng, o Kneiphof v mt min nm gia hai nhnh ca sng Pregel. Vo th k th 18

ng-i ta xy 7 chic cu ni cc vng ny vi nhau. Hnh 1 v cc vng v cc cu qua


sng ca thnh ph.

Hnh 1. Thnh ph Konigsberg th k 18

Vn t ra l c th xut pht ti mt

a im no trong thnh ph i qua tt c cc

ln, ri li tr v im xut pht -c khng?

cu, mi chic cu khng i qua nhiu hn mt

Nh ton hc Thy s, Leonhard Euler l

ng-i u tin gii bi ton ny. ng m

hnh ho s phn b ca cc cy cu bng mt a


th, bn vng -c biu din bng 4 nh, cc
cu l cc cnh nh- trn hnh 2. Vic tm -ng

Hnh 2. M hnh a th ca thnh


ph Konigsberg

i qua tt c cc cu mi cu i qua khng qu mt ln c th -c pht biu bng m hnh


ny nh- sau: C tn ti chu trnh n trong a th trn cha tt c cc cnh?

nh ngha 4.1.1. Chu trnh n cha tt c cc cnh ca th G -c gi l chu

trnh Euler. -ng i Euler trong G l -ng i n cha mi cnh ca G. th -c gi

l th Euler nu n c chu trnh Euler; th -c gi l th na Euler nu n c


-ng i Euler

V d 4.1.1. Trn hnh 3, th G1 c chu trnh Euler a, e, c, d, e, b, a; c hai th

G2 v G3 u khng c chu trnh Euler. Tuy nhin G3 c -ng i Euler, c th l a, b, c, d,

a, c, e, b, cn G2 khng c -ng i Euler.

e
d

G1

c
G3

e
c

G2

Hnh 3. Cc th v h-ng G1 , G2 v G3 .
V d 4.1.2. Trn hnh 4, th H2 c chu trnh Euler, v d a, b, c, d, e, a. C hai

th H1 v H3 u khng c chu trnh Euler. H3 c -ng Euler, c th l a, b, c, a, d, c, nh-ng


H1 khng c -ng i Euler.

c
d

H1

H2

H3

Hnh 4. Cc th c h-ng H1 , H2 , H3 .
Cc v d trn cho thy, mt th c th l mt th Euler/na Euler hoc cng

c th khng. Euler ch ra mt tiu chun n gin khng nh tnh cht Euler/na


Euler ca th khi gii bi ton cu Konigsberg. Chng ta gi s trong mc ny ch xt cc
th hu hn, tc l cc th c mt s hu hn cc nh v cc cnh.

Gi s mt a th v h-ng lin thng c chu trnh Euler. Chu trnh Euler bt u

t mt nh no i qua tt c cc cnh ca th, mi cnh ng 1 ln v tr li nh ban


u. Nu xp cc cnh trong th li theo th t xut hin trong chu trnh Euler v trn 2
u cnh ghi tn cc nh ca th th ta thy rng: bc ca mt nh bt k s bng ng

s ln xut hin ca n trong dy lit k. Nh- v d 2 vi th H2 ta c chu trnh Euler

(a, b, c, d, e, a) t-ng ng vi dy cnh (a,b), (b,c), (c,d), (d,e), (e,a), vy


deg(a)=deg(b)=deg(c)=deg(d)=deg(e)=2. D thy rng gia hai cnh lin tip l 1 nh, n
-c ghi cui cnh bn tri v u cnh bn phi (2 ln trn 2 cnh lin tip), v th s ln
xut hin ca cc nh trong dy lit k l s chn. Ring vi nh xut pht cnh u tin
n xut hin 1 ln v cnh cui cng n cng xut hin 1 ln, v n s xut hin s chn
ln trong khong gia dy lit k (nu c). V th bc ca nh xut pht cng l s chn.
Vy nu th c chu trnh Euler th cc nh ca th u c bc chn. Nh- vy ta c:

nh l 4.1.1. Nu mt a th v h-ng lin thng l th Euler th mi

nh ca n u c bc chn.

iu ng-c li cng ng, tc l nu cc nh ca mt th v h-ng lin thng u

c bc chn th th l th Euler. iu ny c th chng minh bng ph-ng php quy


np theo s nh ca th. Tuy nhin, chng ta s khng dng li chng minh ny.

Quay tr li bi ton cu Konigsberg, ta thy v a th biu din cc cu


ny nh- trn hnh 2, c 4 nh bc l, nn th khng c chu trnh Euler. Do ,

khng c cch no mt ng-i c th xut pht ti mt im no , i qua mi


chic cu ng mt ln v li tr v im xut pht.

Vic xy dng chu trnh Euler cho mt th Euler bt k c th thc hin theo thut

ton sau (thut ton Fleury): Chu trnh Euler -c xy dng bt u t mt nh bt k ca


th v to chu trnh bng cch chn ln l-t cc cnh. Mi ln mt cnh -c chn, cnh
-c xo i. Cc cnh -c chn lin tip sao cho mi cnh bt u ti ni m cnh lin
tr-c kt thc v ch chn cu nu khng cn cnh no khc chn.

H qu 4.1.1: Mt a th v h-ng, lin thng c -ng i Euler (v

khng c chu trnh Euler) nu v ch nu n c ng hai nh bc l.

Chng minh: Tr-c tin, ta gi s th cho c -ng i Euler t a ti b,

nh-ng khng c chu trnh Euler. Cnh u tin ca -ng i gp 1 vo deg(a). Sau

mi ln -ng i li qua nh a th n tng thm 2 n v cho deg(a). Do vy, a l


mt nh bc l. Cnh cui cng ca -ng i gp 1 cho deg(b) v mi ln i qua b
n cng tng thm 2 cho deg(b). V th bc ca b l l. Cc nh trung gian u c
bc chn v mi ln -ng i ti ri li ri n nn tng 2 n v cho bc ca n. Vy,
th cho c ng 2 nh bc l.

By gi ta gi s ng-c li, th c ng hai nh bc l, chng hn a v b.

Ta xt th rng hn to nn bng cch thm mt cnh ni a vi b vo th xut


pht. khi bc ca tt c cc nh ca th mi ny u l s chn, theo nh l

1, n c chu trnh Euler. Xo cnh mi v thm vo ta s nhn -c -ng i Euler


ca th xut pht.
T y ta thy rng, th m hnh ho cc cy cu Konigsberg c 4 nh

bc l, theo h qu trn n khng c -ng Euler. Tc l cng khng th c hnh


trnh xut pht t mt im no i qua tt c cc cy cu, mi cy cu ch qua
ng mt ln v kt thc hnh trnh mt im no trong thnh ph.

V d 4.1.4. Trn hnh 5, th G1 c ng hai nh bc l l b v d. Do ,

n c -ng i Euler nhn b v d l cc im u nt. Mt trong cc -ng i Euler


l d, a, b, c, d, b. T-ng t G2 cng c ng hai nh bc l b v d. Do n c
-ng i Euler nhn b v d l cc im u nt. Mt trong cc -ng i Euler l b,
a, g, b, c, g, f, c, d, f, e, d. Cn G3 khng c -ng i Euler v n c 6 nh bc l.

f
d

G1

G2

Hnh 5. Ba th v h-ng.

G3

c
d

4.2. -ng i v chu trnh Hamilton

Chng ta nghin cu iu kin cn v tn ti -ng i v chu trnh


i qua mi cnh ca th, mi cnh qua ng mt ln. Liu ta c th lm t-ng t

i vi -ng i v chu trnh cha mi nh ca th ng mt ln khng? Hai bi


ton nghe nh- t-ng t nh-ng thc ra l hai bi ton hon ton khc hn nhau.
nh ngha 4.2.1. -ng i x0 , x1 , ..., xn-1 , x n trong th G = (V, E) -c gi

l -ng i Hamilton nu V = {x 0 , x1 ,...., xn-1 , xn } v xi xj vi 0 i < j n. chu


trnh x 0 , x1 ,...., xn-1 , xn , x0 (n>1) trong th G = (V, E) -c gi l chu trnh
Hamilton nu x0 , x 1 ,...., x n-1 , x n l -ng i Hamilton.

G1

G2

G3

Hnh 1. Ba n th

V d 4.2.1. G1 c chu trnh Hamilton a, b, c, d, e, a. Khng c chu trnh

Hamilton trong G2 (v bt c chu trnh no cha mi nh cng phi cha cnh {a,
b} hai ln). Nh-ng G2 c -ng i Hamilton a, b, c, d. G3 khng c c chu trnh

Hamilton ln -ng i Hamilton, v bt k -ng i no cha mi nh cng phi


cha mt trong cc cnh {a,b}, {c,d} v {e, f} qu mt ln.
Cho n nay vn ch-a c cc thut ton c hiu qu nhn bit mt th

c chu trnh Hamilton hay khng, tuy rng c nhng th d dng xy dng -c
chu trnh Hamilton nh- v d sau.
V d 4.2.2. th Kn c chu trnh Hamilton vi mi n 3. Chng ta c th

xy dng chu trnh Hamilton trong Kn xut pht t bt k nh no. Mt chu trnh

nh- th c th xy dng bng cch gh thm cc nh theo mt th t tu chn, sao


cho -ng i bt u v kt thc ti mt nh, mi nh u -c gh thm ng mt
ln. iu l c th v gia hai nh bt k ca Kn u c cc cnh ni chng.
nh l sau cho ta iu kin tn ti chu trnh Hamilton.

nh l 4.2.1. Gi s G l mt n th lin thng c n nh, n 3. Khi

G c chu trnh Hamilton nu bc ca mi nh t nht bng n/2.

Chng minh: Thm vo G cc nh mi v ni chng vi tt c cc nh ca

G. Gi k l s t nht cc nh cn thm vo cho th thu -c G l th

Hamilton. K hiu v, p, w, , v l mt chu trnh Hamilton trong G trong v, w l


cc nh ca G, cn p l mt trong s cc nh mi thm vo. R rng v v w khng
k nhau, v nu chng k nhau ta c th ni trc tip v vi w v bt -c nh p, iu
ny mu thun vi k l s nh nht. Mt khc trong G, mi nh k vi w (w chng

hn) khng th i lin sau nh k vi v (v chng hn) v khi c th thay chu


trnh v p w v w v bi chu trnh v v w w
v bng cch o ng-c on ca chu trnh nm gia w v v khng cn s

dng ti p. T y suy ra sau mi nh k vi v phi l mt nh khng k vi w, tc


l s nh trong G khng k vi w t ra phi l (n/2+k), mt khc s nh k vi w t
ra cng phi l (n/2+k). Do khng c nh no va khng k li va k vi w nn s
nh ca G phi khng t hn n+2k. iu ny mu thun vi vic G c ng n+k
nh.
By gi ta s -a ra mt ng dng ca chu trnh Hamilton m ho.

V d 4.2.3. M Gray. V tr ca kim ch th xoay trn c th -c biu din

d-i dng s. Mt cch th hin l chia -ng trn thnh 2 n cung c di bng
nhau v cho mi cung ng vi mt xu nh phn di n. Trn hnh 2 biu din 2
cch chuyn i nh cc xu nh phn di 3.
111 000

110

101
100

001

011

010

100 000

101

111
110

001

010

011

Hnh 2. Chuyn i v tr ca kim


ch th thnh dng s

Biu din s ca v tr ca kim ch th c th xc nh bng n ch tip xc.

Mi tip xc -c dng c mt bit trong biu din s ca v tr. iu ny -c


minh ho trn Hnh 3.

Bt th ba l 1
y
Bt th

l 1

hai
l 1

l 1

Bt th hai

ba
l 1
l 1

Bt th ba

Bt th

Bt th

Bt th nht

Bt th nht l 1 y

Bt th ba

l 1

Bt th ba l 1
y

ba
l 1
Bt th hai l 1 y

Hnh 3. Biu din s ca kim ch th

Khi kim gn gip ranh ca hai cung, c th d lm ln khi c v tr ca n.

iu ny c th gy nn cc li ln khi c cc xu nh phn. V d, theo s m


ho Hnh 2a, nu c mt li nh khi xc nh v tr ca kim, xu nh phn 100 -c
c thay cho 011. C 3 bt u sai.
gim n mc ti thiu nh h-ng ca li xc nh v tr ca kim, ng-i ta

gn cc xu nh phn cho 2 n cung sao cho cho hai xu biu din bng hai cung k
nhau ch sai khc nhau mt bit. iu ny -c th hin trong s m ho trn Hnh
2b. Li khi xc nh v tr ca kim ch th cho xu 010 thay cho xu 011. Ch mt bit
b sai.

M Gray l cch gn nhn cho cc cung ca -ng trn sao cho cc cung k nhau -c

gn bng cc xu khc nhau ng mt bit. Cch gn trong Hnh 2b l mt m Gray. Chng


ta c th tm m Gray bng cch lit k tt c cc xu nh phn di n sao cho mi xu ch

khc xu tr-c n ti ng mt v tr. V xu cui cng khc vi xu u tin cng ch

mt v tr. Chng ta c th m hnh bi ton ny bng khi hp n chiu Qn . Ci m chng ta


Hamilton trong Qn . Cc chu trnh Hamilton nh-

th d dng thu -c. V d chu trnh Hamilton 100


cho Q3 -c th hin trn Hnh 4. Dy cc xu

nh phn sai khc nhau ng mt bt -c sinh


bi chu trnh Hamilton l 000, 001, 011, 010,

gi di m Gray. D-i y chng ta m t

101
010

110, 111, 101, 100, 000.

di ca cc dy nh phn nh- trn -c

111

110

cn gii bi ton ny l mt chu trnh

000

011
001

Hnh 4. Chu trnh Hamilton i vi Q3

thut ton sinh m Gray di l s n nguyn d-ng bt k (ng thi cng chng minh
th -c biu din bng khi hp n chiu Qn c chu trnh Hamilton).
Gn-1

Gi s G1 l dy 0, 1 (G1 l m Gray di 1). M Gray Gn di n -c xy dng t


bng cch nh- sau: Gi s GnR1 l G n-1 -c vit theo th t ng-c li; Gn' 1 l tp nhn

-c t G n-1 bng cch gn thm ng tr-c mi phn t ca Gn-1 s 0; Gn'' 1 l tp nhn -c

t GnR1 bng cch gn thm ng tr-c mi phn t ca GnR1 s 1. Khi tp Gn bao gm


tt c cc phn t ca Gn' 1 tip l cc phn t ca Gn'' 1 .

V d 4.2.4. S dng thut ton trn xy dng m Gray G 3 bt u t G1 . Ta c:

G1
R
1

'
1

''
1

G2
R
2

'
2

''
2

G3

0 1

1 0

00 01
11 10

00 01 11 10
10 11 01 00

000 001 011 010


110 111 101 100

000 001 011 010 110 111 101 100

5. Bi ton tm -ng i ngn nht


5.1. th c trng s
Chng ta nhc li th c trng s l th m mi cnh ca n -c gn mt s

thc gi l trng s ( di) ca cnh . Nhiu bi ton thc t c th -c m hnh ho


bng cc th c trng s.

V d 5.1.1. Gi s trong qu trnh gia cng ch to my ng-i ta cn khoan cc l

trn mt tm thp (hnh 1) sau bt vt cc chi tit my. Cc l khoan c th -c thc


hin d-i s iu khin ca my tnh. Nhm tit kim chi ph, qu trnh khoan cn -c
thc hin cng nhanh cng tt. Chng ta s m hnh ho bi ton t ra bng th.

Cc nh ca th t-ng ng vi cc l khoan. Mi cp nh -c ni vi nhau bi

mt cung l cnh ca th. Trn mi cnh ta vit cc s t-ng ng vi thi gian dch
chuyn t l khoan ny n l khoan kia. Ta thu -c th c trng s hnh 2. Trn
th , di cnh (d,b) ni nh d vi nh b l 12, di cnh (c,e) l 5.

a
c

4
d

2 c

3
4

12

Hnh 1. Phn b cc l khoan Hnh 2. th m hnh ho qu trnh khoan


Trong th c trng s, di ca -ng i l tng di cc cnh nm trn

-ng i y. Trn hnh 2, -ng i t a ti b qua nh c c di bng 8. Bng 1 cho ta

di cc -ng i bt u t a ti e qua tt c cc nh ca th mi nh ng 1 ln. So


snh di cc -ng i y, ta thy, nu yu cu mi khoan u tin l a, mi khoan cui
cng l e th vic thc hin cc mi khoan theo trnh t a, b, c, d, e s cho ta chi ph thi
gian nh nht bng 19.

Trnh t thc hin cc mi khoan Thi gian thc hin


a, b, c, d, e

6+6+3+4=19

a, b, d, c, e

6 +12+3+5=26

a, c, d, b, e

2+3+12+9=26

a, c, b, d, e
a, d, b, c, e
a, d, c, b, e

2+6+12+4=24
4+12+6+5=27
4+3+6+9=22

Bng cch xc nh di tt c cc hnh trnh c th c qua mi nh ca

th hnh 2, mi nh 1 ln, ta c th tm ra -c trnh t thc hin cc mi khoan


gii bi ton t ra.

V d 5.1.1. Mng l-i giao thng ni cc im du lch trong mt khu du lch

sinh thi cng c th m hnh ho bng mt th c trng s, trn di mi


cung c th l khong cch hoc thi gian cn thit i t im du lch ny ti
im du lch kia. Trn hnh 3 l mt v d nh- vy: 6 im du lch -c k hiu l a,
b, c, d, e, z; cc cung ni cc cp im ch ra rng gia cc cp im c -ng
ni trc tip chng vi nhau; cc s trn cc cung l di ca cung .

Hnh 3. S mng l-i giao thng mt khu du lch sinh thi

C th ch ra nhiu v d khc dn ti m hnh th c trng s t-ng t nh-

bi ton trn. Trong s cc bi ton , nhiu bi ton i hi xc nh mt -ng i

ti -u ni hai nh cho tr-c trn th t-ng ng (khng nht thit phi i qua tt
c cc nh nh- v d 5.1.1) nh- cc bi ton ny sinh trong vn chuyn hng ho
v du lch, trong thit k cc mng my tnh sao cho chi ph cp truyn nh nht...
(Cn l-u khi nim di y khc vi khi nim di l s cc cnh trong

mt -ng i ca th khng trng s). Phn tip theo chng ta s cp ti mt


trong cc thut ton kh hiu qu tm -ng i ngn nht gia hai nh ca th.
5.2. Thut ton tm -ng i ngn nht Dijkstra

Thut ton ny do E.Dijkstra, nh ton hc ng-i H Lan, xut nm 1959.

D-i y ta gi s th l v h-ng, cc trng s l d-ng. Ch cn thay i i


cht l c th s dng thut ton ny gii bi ton tm -ng i ngn nht trong
th c h-ng.

Thut ton Dijkstra -c thc hin bng cch tun t tm di ca -ng i

ngn nht t a ti nh u tin, di ca -ng i ngn nht t a ti nh th hai


.v.v... cho ti khi tm -c di ca -ng i ngn nht t a ti z.
Thut ton ny da trn mt s dy cc b-c lp. Mt tp c bit cc nh S,

u tin gm mt nh a, -c xy dng bng cch cng thm mt nh trong mi

b-c lp. xc nh nh -a vo tp S, mt th tc gn nhn -c thc hin trong


mi ln lp. Trong th tc gn nhn ny, nh w -c gn nhn bng di ca
-ng i ngn nht t a ti w v ch i qua cc nh thuc tp S. nh -c thm
vo tp S l nh c nhn nh nht so vi cc nh ch-a c trong tp . Chi tit ca
thut ton Dijkstra -c trnh by nh- sau.

Input: G = (V, E) l th lin thng c trng s l cc s d-ng. a v z l 2

nh ca th.

Output: L(z) - di -ng i ngn nht t a ti z.

1. [Khi to] t L(a ) : 0; S : . Vi mi nh v a, t L(v) := .


2. [Kt thc?] Nu z S , dng thut ton (L(z) l di -ng i ngn nht
t a ti z).

3. [Tm nh tip theo] Chn v V \ S sao cho L(v) c gi tr nh nht. t


S : S {v} .
4. [Gn li nhn cho cc nh] Vi mi nh x lin thuc vi v nh-ng khng
thuc S t: L( x) : min{L ( x), L (v) w(v, x)} . Quay tr li b-c 2.
Ta xt mt v d minh ho cc b-c thc hin ca thut ton Dijkstra.

V d 5.2.1. Tm di ca -ng i ngn nht gia hai nh a v z ca th

c trng s trn hnh 4a.

Gii: Cc b-c dng thut ton Dijkstra tm di ca -ng i ngn nht

gia hai nh a v z -c biu din trn hnh 4. Ti mi b-c lp ca thut ton,

phn bit cc nh thuc tp c bit S vi cc nh khc ta s khoanh trn cc nh

. Thut ton kt thc khi nh z -c khuyn trn. -ng i ngn nht t a ti z l


a, c, b, d, e, z vi di bng 13 -c cho trn hnh 4g.

a0

4
2

(a)

b
1

2
3

10

b
4

3(a,c)

10
2(a)

2
3

10

e 12(a,c
)

0
a

10(a,c,b,d)

2(a)

3(a,c)

2
10

2(a)

14(a,c,b,d)

8(a,c,b)
6

e 14(a,c,d,b
)

0 4
a
1

10

(g)

d 8(a,c,b)

(e)

c 2(a) e

(c)

14(a,c,b,d)

b 3(a,c d
5)

10

b 3(a,c)

8(a,c,b)
6

0
a

c 2(a) e

2(a)

5
1

10

(d)

(f)

(b)

d 8(a,c,b)

5
1

0
a

b 3(a,c)

0
a

0
a

b 4(a) d

13(a,c,b,d,e)

10(a,c,b,d)

Hnh 4: Dng thut ton Dijkstra tm -ng i ngn nht t nh a ti nh z

Bng ph-ng php quy np ton hc, c th chng minh -c rng thut ton

Dijkstra lun cho di -ng i ngn nht gia nh a v z trong mt th v


h-ng lin thng c trng s.
6. th phng

6.1 Khi nim


Xt bi ton ni ba ngi nh vi

ba thit b sinh hot ring r nh- trn


hnh 1. Liu c cch no ni cc ngi

nh vi cc thit b ny sao cho khng c


-ng no ct nhau? Bi ton ny c th
-c m hnh bng th phn i y
K3,3 . Khi cu hi trn c th din

Hnh 1. Ba ngi nh v ba thit b

t nh- sau: Tn ti hay khng cch v K3,3 trn mt mt phng sao cho khng c hai
cnh no ct nhau?
Cu hi t-ng t cng th-ng -c t ra vi cc th khc, nht l trong

nhng ng dng th thit k cc bo mch in t. Ho ra l khng phi th


no cng c th v -c trn mt phng sao cho cc cnh ca chng khng ct nhau.
D-i y chng ta s nghin cu vn ny.
nh ngha 6.1.1: Mt th -c gi l th phng nu ta c th v n trn

mt mt phng sao cho khng c cc cnh no ct nhau mt im khng phi l


nh ca th. Hnh v nh- vy -c gi l mt biu din phng ca th.
Mt th c th l th phng ngay c khi n th-ng -c v vi nhng

cnh ct nhau, bi v c th v n bng cch khng c cc cnh ct nhau.

V d 6.1.1. th K4 trn hnh 2 vi 2 cnh ct nhau c l th phng

khng?

Gii: K4 l th phng bi v c th v li nh- trn hnh 3 khng c cnh

no ct nhau im khc vi nh.

Hnh 2. th K4

Hnh 3. K 4 v khng c -ng ct nhau

V d 6.1.2. Q3 trn Hnh 4 c l th phng khng?

Hnh 4. th Q3

Hnh 5. Biu din phng ca Q3

Gii: Q3 l th phng v c th v li nh- trn hnh 5 sao cho khng c

cnh no ct nhau.

C th ch ra mt th l phng bng cch ch ra mt biu din phng ca


n. Chng t mt th l khng phng s kh khn hn nhiu. Th-ng ng-i ta
phi da vo mt s tnh cht ca cc th phng xc nh tnh khng phng ca
mt th cho.
6.2. Cng thc Euler

Biu din phng ca mt th chia mt phng thnh cc min k c min v hn.

V d biu din phng ca th trn hnh 6 chia min phng thnh 4 min. Chng -c gn
nhn nh- hnh v.

B
C

Hnh 1. th phng vi 4 min khng giao nhau.


Euler chng minh rng mi biu din phng ca cng mt th phng u chia

mt phng thnh cng mt s min nh- nhau. lm iu ny, ng chng minh mt


cng thc th hin mi quan h gia s min, s nh, s cnh ca mt th phng.

nh l 6.2.1. Cng thc Euler. Cho G l mt n th phng lin thng vi

e cnh v v nh. Gi f l s min trn biu din phng ca G. Khi ta c f = ev+2.

D-i y ta s xt mt vi v d minh ho.

V d 6.2.1. Xt th phng hnh 1. th c s cnh e=7, s nh

v=5, s min f=4. Ta c : f=e-v+2=7-5+2=4.

V d 6.2.2. Chng t th K3,3 trn hnh 2 khng phi l th phng.

Gii: Tht vy, gi s K3,3 l th phng. Do mi chu

trnh ca K3,3 gm t nht 4 cnh, nn mimt ca n phi


-c gii hn bi t nht 4 cnh. Nh- vy tng s cnh
gii hn cc mt t nht phi l 4f. Mt khc trong th

Hnh 2. th K3,3

phng, mi cnh thuc vo nhiu nht 2 chu trnh gii hn cc mt. Nh- vy : 2e
4f. S dng cng thc Euler ta suy ra :
2e 4(e v 2).

Vi th K3,3 hnh 2 ta c : e=9, v=6, tc l 2.9=18 4(9-6+2)=20. Mu


thun ny chng t K3,3 khng phi l th phng.
V d 6.2.2 chnh l bi ton nh - thit

b sinh hot -c m t trong u tit ny : Ba


ngi nh v ba thit b sinh hot khng th ni
vi nhau trn mt mt phng m khng ct
nhau.

Bng cch t-ng t ta cng c th


chng t th K5 hnh 3 cng khng phi l
th phng.

V d 6.2.3. n th phng lin

Hnh 3. th y K5

thng 9 nh, bc ca cc nh t-ng ng bng


2, 2, 2, 3, 3, 3, 4, 4 v 5. Hi th c bao nhiu cnh v biu din phng ca th
ny chia mt phng thnh bao nhiu min?

Gii: th c 9 nh, do vy v=9. Ta c tng s bc ca cc nh chnh l 2

ln s cnh ca th, tc l 2e= 2+2+2+3+3+3+4+4+5=28. Vy s cnh ca th


l e=28/2=14. Theo cng thc Euler, s cc min ca th l:
f = e - v + 2 = 14 - 9 + 2 = 7

6.3. nh l Kuratowski

Chng ta thy cc th K3,3 v K5 khng phi l cc th phng. R

rng, mt th l khng phng nu n cha c hai th ny nh- l cc th con.

Xut pht t nhn xt ny, nh ton hc ng-i Ba Lan Kuratowskii chng minh
mt nh l rt quan trng mang tn ng lin quan n c im ca cc th
phng. pht biu nh l , chng ta cn -a vo mt s khi nim sau.

nh ngha 6.3.1. Xt mt cnh (u,v) no trn th. Php phn chia s

cp cnh (u,v) -c nh ngha l vic loi b cnh ny khi th v thm vo


th mt nh mi w cng hai cnh (u,w), (w,v).

nh ngha 6.3.2. Hai th G v H -c gi l hai th ng phi, nu

chng c th nhn -c t cng mt th bng mt dy cc php phn chia s cp.

V d 6.3.1. Ba th trn hnh 1 l ng phi, v chng c th nhn -c t

th u tin bng cc php phn chia s cp (c th d dng xc nh cc php


phn chia s cp nhn G2 v G3 t G1 ).

G1 a

G2

a
f

G3

b
k

Hnh 1. Cc th ng phi

nh l 6.3.1. nh l Kuratowskii. Mt th l th phng khi v ch khi

n khng cha mt th con ng phi vi K3,3 hoc K5 .

Vic chng minh mi th khng phng u cha mt th con ng phi

vi K3,3 hoc K5 l rt phc tp, chng ta s khng trnh by y. kt thc mc


ny, chng ta xt mt vi v d minh ho cch s dng nh l Kuratowski.

j
i

g f
G

gf
H

c
g
K5

Hnh 2. th v h-ng G, th con H ng phi vi K5 v K5

V d 6.3.2. th G trn hnh 2 c l th phng hay khng?

Gii: H l th con ca th G: H nhn -c t G bng cch xo h, j, k v

tt c cc cnh lin thuc vi cc nh ny. H l ng phi vi K5 v n c th nhn

-c t K5 (vi cc nh a, b, c, g v i) bng mt dy cc php phn chia s cp,


thm vo cc nh d, e v f (c th d dng xy dng dy phn chia s cp ny). Do
th G khng l th phng.

V d 6.3.3. th Petersen trn hnh 3a c l th phng khng?

a
f

j
d

g
h

a)

g
i

b) H

c) K3,3

Hnh 14. (a) th Peterson, (b) th con H ng phi vi K 33


v (c) K

Gii: th con H ca th Petersen nhn -c bng cch b nh b v 3

cnh lin thuc vi n nh- trn Hnh 3b, l ng phi vi K3,3 vi cc tp nh {f,d,j}

v {e, i, h}. Tht vy, H c th nhn -c t K3,3 (hnh 3c) bng mt dy cc phn
chia s cp: xo {d, h}, thm {c, h} v {c, d}; xo {e, f}, thm {a, e} v {a, f}; xo
{i, j}, thm {g, i} v {g, j}. V th th Petersen khng l th phng.
7. T mu th

7.1. M u
phn bit cc min c chung bin gii trn bn , ng-i ta phi t mu

chng bng cc mu tu khc nhau. Mt bi ton thc t -c t ra l: cn c t


nht bao nhiu mu t mu mt bn bt k sao cho cc min k nhau khng
cng mt mu. V d, ta c th thy trn hnh 1, vi bn bn tri cn phi dng t
nht 4 mu, cn vi bn bn cn phi dng t nht 3 mu.

B
A

F
E

C
D

Hnh 1. Hai bn c th t bng s mu ti thiu khc nhau

Ta c th t-ng ng mi bn trn mt phng vi mt th bng cch sau:

mi min bn -c biu din bng mt nh, hai nh trn th -c ni vi


nhau bi mt cnh, nu cc min -c biu din bng hai nh ny c bin gii
chung nhau (hai min ch chung nhau mt im khng -c coi l k nhau). th
nhn -c bng cch nh- vy gi l th i ngu ca bn ang xt. R rng

mi bn trn mt phng u c th i ngu phng. Hnh 2 biu din cc th


i ngu vi cc bn cho trn hnh 1.

B
A

B
A

Hnh 2. Cc th i ngu ca cc bn trn hnh 1

Bng cch xt cc th i ngu, ta c bi ton t mu cc min ca bn


t-ng -ng vi bi ton t mu cc nh ca th sao cho khng c hai nh lin
k nhau c cng mt mu. Chng ta c nh ngha sau.

nh ngha 7.1.1. T mu n th l s gn mu cho cc nh ca n sao

cho khng c hai nh lin k -c gn cng mt mu.

nh ngha 7.1.2. S mu ca mt th l s ti thiu cc mu cn thit

t mu th ny.

Bi ton t mu bn nu trn chnh l bi ton xc nh s mu ln nht

ca cc th phng. Bi ton ny -c nghin cu t 100 nm nay v cu tr li


chnh l mt trong cc nh l ni ting nht trong ton hc.
nh l 7.1.1. nh l Bn mu. S mu ca mt th phng bt k l mt

s khng ln hn 4.

nh l ny ln u tin -c -a ra nh- mt phng on vo nm 1850. V

cui cng n -c hai nh ton hc M l Kenneth Appel v Wolfgang Haken


chng minh vo nm 1976. Tr-c cng c nhiu chng minh sai (m thng
th-ng rt kh tm thy ch sai trong cc chng minh ) -c cng b. ng
thi cng tng c nhiu c gng mt cch v ch tm phn v d bng cch c
v mt bn c s mu ln hn 4.

C l mt chng minh sai ni ting nht trong ton hc l chng minh sai bi

ton bn mu -c cng b nm 1879 bi lut s-, nh ton hc nghip d- Lun-n


tn l Alfred Kempe. Ng-i ta chp nhn cch chng minh ca ng ta cho ti nm
1890 khi Percy Heawood pht hin ra sai lm trong chng minh y. Tuy nhin, cch
lp lun ca Kempe li l c s cho chng minh ca Appel v Haken. iu c bit
l chng minh ca h -c thc hin da trn s phn tch tng tr-ng hp mt cch
cn thn nh my tnh. Appel v Haken ch ra rng nu bi ton bn mu l sai

th s c mt phn v d thuc mt trong gn 2000 loi khc nhau v ch ra khng


c loi no trong s dn ti phn v d c. Chng minh ca h l dng hn
1000 gi my. Cch chng minh ny gy ra nhiu cuc tranh ci v vai tr quan
trng ca my tnh. Chng hn, liu c th sai lm trong ch-ng trnh v iu dn
n kt qu sai khng? L lun ca h c thc s l mt chng minh hay khng, nu
n ph thuc vo thng tin ra t mt my tnh?
xc nh s mu ca th l mt s n, tr-c tin ta phi chng t rng

th c th -c t mu bng n mu. Sau chng ta phi chng t rng khng th


t mu th vi s mu t hn.

b
a

e
g

d
c

b
d

a
c

Hnh 3. Cc n th G v H

g
H

V d 7.1.1. Xc nh s mu ca th G v H cho trn hnh 3?

Gii: V cc nh a, b v c k nhau tng i mt, chng phi -c t bng cc

mu khc nhau, do vy s mu ca G khng th nh hn 3. thy G c th t

bng 3 mu ta gn mu cho a, mu lam cho b v mu lc cho c. Khi d c th


(v phi) t mu v lin k vi b v c. Tip theo e c th (v phi) t mu lc v n
lin k vi cc nh mu v lam. nh f c th (v phi) t mu lam v n lin k
vi cc nh mu v mu lc. Cui cng g c th (v phi) t mu v n lin k
vi cc nh mu lam v mu lc. Vy l ta t mu th G bng ng 3 mu.

th H -c to nn t th G bng cch to thm vo mt cnh ni a vi

g. Mi nh t H bng 3 mu cn phi tun theo l lun dng khi t mu G, tr


giai on cui cng, khi cc nh khc g -c t mu. V g lin k (trong H) vi
cc nh mu , mu lam v mu lc nn ta buc phi dng mu th t-, chng hn
mu nu. V th H c s mu bng 4. Cch t mu cho G v H -c th hin trn
hnh 4. Cng cn l-u l bi ton bn mu ch p dng cho cc th phng. Cc
th khng phng c th c s mu ln tu .

b Lam
a

c Lc

e Lc
g

e Lc

b Lam
d

f Lam

c Lc

Hnh 4. T mu th G v H

Nu

f Lam

V d 7.1.2. Tm s mu ca th Kn .

Gii: C th xy dng cch t mu th Kn vi n mu khc nhau. Mt khc,

v mi cp nh ca th ny u lin k, cho nn khng c hai nh no c th


gn cng mu. V th, s mu ca Kn ng bng n. (Nh li rng Kn l khng phng
nu n 5, v kt qu ny khng mu thun vi nh l 4 mu). Cch t mu th
K5 bng 5 mu -c th hin nh- trn hnh 5.
a

b
e Nu
c Lc
d Vng
Hnh 5. T mu th K5

d Lam

e Lam

f Lam

g Lam

Hnh 6. T mu th K 3,4

V d 7.1.3. Tm s mu ca th phn i y Km, n , trong m v n l cc s


nguyn d-ng.

Gii: C th t tp m nh bng mt mu v tp n nh bng mu khc. V

mi cnh ca Km, n ch ni cc nh t tp m nh ti nh thuc tp n nh nn khng


c hai nh lin k no cng mu. Do vy s mu ca th phn i y Km, n

bng 2. iu th v l, ng-c li, mi th c s mu bng 2 u l th phn i.


Hnh 6 th hin mt cch t mu th K3,4 .

7.2. Mt s ng dng ca bi ton t mu th


Bi ton t mu th c nhiu ng dng

khc nhau. Mt vi ng dng nh- vy s -c xt


thng qua cc v d d-i y.

trong tr-ng i hc sao cho khng c sinh vin


no phi thi ng thi hai mn cng mt lc.

Gii: C th m hnh ho bi ton lp lch

V d 7.2.1. Lp lch thi. Hy lp lch thi

bng th, vi cc nh l cc mn thi, hai nh


c mt cnh ni chng vi nhau nu c sinh vin
phi thi c hai mn -c biu din bng hai nh
ny. Nu coi thi gian thi ca mi mn -c biu

3
4

Hnh 8. th biu din bi


ton lp lch thi

th bng cc mu khc nhau, th vic lp lch thi s t-ng ng vi vic t mu th ny.


V d c 7 mn thi cn xp lch. Gi s cc mn hc -c nh s t 1 n 7, v cc

cp mn thi sau c chung sinh vin: 1 v 2, 1 v 3, 1 v 4, 1 v 7, 2 v 3, 2 v 4, 2 v 5, 2 v


7, 3 v 4, 3 v 7, 4 v 5, 4 v 6, 5 v 7, 6 v 7. th t-ng ng -c cho trn hnh 8. V s
mu ca th ny l 4 (c gi t kim tra li iu ny) nn cn c 4 t thi. Cch t th
bng 4 mu v lch thi -c biu din trn hnh 9.
1
2 Lam

Nu 7

3 Lc

5 Lc

4 Nu

t thi
I
II
III
IV

Mn thi
1.6
2
3,5
4,7

Hnh 9. Dng bi ton t mu lp lch thi


V d 7.2.2. Phn chia tn s. Cc knh truyn hnh t s 2 ti s 13 cn -c phn chia

cho cc i truyn hnh Bc M sao cho khng c hai i pht no cch nhau khng qu 150
dm li dng cng mt knh. Vic chia knh truyn hnh ny c th -c gii quyt bng m
hnh t mu th. C th, ta xy dng th bng cch coi mi i pht l mt nh. Hai

nh -c ni vi nhau bng mt cnh nu chng cch nhau khng qu 150 dm. Vic
phn chia knh t-ng ng vi vic t mu th, trong mi mu biu th mt knh.

V d 7.2.3. Cc thanh ghi ch s. Trong cc b dch hiu nng cao, vic thc hin cc

vng lp -c tng tc khi cc bin dng th-ng xuyn -c l-u tm thi trong cc thanh ghi

ch s ca B x l trung tm (CPU) m khng phi trong b nh thng th-ng. Vi mt vng


lp cho tr-c cn bao nhiu thanh ghi ch s? Bi ton ny c th gii bng m hnh t mu
th. xy dng m hnh ta coi mi nh ca th l mt bin trong vng lp. Gia hai nh c
mt cnh nu cc bin biu th bng cc nh ny phi -c l-u trong cc thanh ghi ch s ti

cng thi im khi thc hin vng lp. Nh- vy s mu ca th chnh l s thanh ghi cn c
v nhng thanh ghi khc nhau -c phn cho cc bin khi cc nh biu th cc bin ny l lin
k trong th.

8. Cy v ng dng
8.1. M u
Mt th lin thng v khng c chu trnh n -c gi l cy. Nm 1957 Arthur

Cayley, mt nh ton hc ng-i Anh, ln u s dng cy xc nh nhng dng khc


nhau ca hp cht ho hc. T cy -c dng gii nhiu bi ton trong cc lnh vc
khc nhau. Trong tin hc cy -c s dng xy dng cc thut ton rt hiu qu nh
v cc phn t trong mt danh sch. Cy cng dng xy dng cc mng my tnh vi chi

ph r nht cho cc -ng in thoi ni cc my phn tn, hoc trong vic to ra cc m c


hiu qu l-u tr v truyn d liu

nh ngha 8.1.1. Cy l mt th v h-ng sao cho gia hai nh bt k lun c

mt -ng i n duy nht ni chng vi nhau.

V d 8.1.1. Trn hnh 1: G1 v G2 l cy. G3 khng l cy v, v d, e, b, a v e, d, a

l 2 -ng i n t e ti a. G4 cng khng l cy v, v d, t a ti b khng c -ng i.

d
c

G1

c
e

G2

G3

G4

Hnh 1. G1 v G2 l cy, G3 v G4 khng l cy.


Trong nhiu ng dng, mt nh c bit ca cy -c gi l gc. Mt khi nh

r gc, ta c th gn cho mi cnh mt h-ng nh- sau. V c -ng i duy nht t gc ti

mi nh ca cy, nn ta nh h-ng mi cnh bng h-ng t gc i ra. Nh- vy cy cng

vi gc sinh ra mt th c h-ng gi l cy c gc. Ta c th chuyn cy khng gc


thnh cy c gc bng cch chn mt nh bt k lm gc. L-u rng vic chn gc khc
nhau s to ra cc cy c gc khc nhau.

V d 8.1.2. Hnh 2 biu din cc cy c gc khc nhau -c to ra t th T

bng cch chn a v sau l c lm gc. Thng th-ng ng-i ta v cy c gc vi


gc pha trn ca th v b mi tn ch h-ng trn cc cnh ca cy c gc v
vic chn gc xc nh h-ng ca cc cnh.

f
d

c
g

e
d

e
f

c
T

e
f

Vi gc a

g
Vi gc c

Hnh 2. Cy v cc cy c gc

Cc thut ng i vi cy c gc -c hiu t-ng t nh- trong sinh vt hc


hoc nh- trong mt ph h. Mt cy T khi xc nh gc, th mi nh v khc vi
gc s ch c duy nht mt nh u sao cho c mt cnh h-ng t u ti v. Khi y nh
u y -c gi l b (cha) ca v v v -c gi l con ca u. Cc nh c cng cha
-c gi l cc nh anh em. Cc nh nm trn -ng i t gc ti mt nh v khc
vi gc -c gi l t tin ca v. Con chu ca nh v l cc nh c v l t tin. Mt
nh ca cy -c gi l l nu n khng c con. Cc nh c con -c gi l nh

trong. Chiu cao ca cy -c nh ngha l -ng i di nht c th c (theo ngha


cha nhiu cnh nht) i t gc ti l ca cy.
Nhiu bi ton thc t c th -c m hnh ha bng cy c gc. D-i y

chng ta s xt mt trong cc bi ton . Nh- chng ta bit, tm kim mt phn t

trong danh sch l mt trong nhng cng vic quan trng nht trong tin hc. Vn
t ra l cn -a ra mt thut ton hiu qu nht tm kim mt phn t trong dy

cc phn t -c sp xp theo mt th t no . iu c th thc hin bng cy


tm kim nh phn.

nh ngha 8.1.2. Cy nh phn l mt cy c gc, trong mi con ca mt

nh hoc l con bn tri hoc l con bn phi, khng c nh no c nhiu hn mt


con bn tri hay nhiu hn mt con bn phi. Cy nh phn -c gi l y , nu
mi nh ca n hoc l khng c con hoc l c ng 2 con.

nh ngha 8.1.3. Cy tm kim nh phn l mt cy nh phn, trong mi

nh -c gn mt kha sao cho mi gi tr kha xc nh ch mt nh v kha ca


nh ln hn kha ca tt c cc nh con bn tri ng thi nh hn kha ca tt c
cc nh con bn phi ca n.

Vic to lp cy tm kim nh phn cho mt danh sch cc phn t c th

-c thc hin bng th tc quy sau y. Bt u l cy ch c mt nh, tc l


gc. Phn t u tin trong danh sch -c dng lm kha ca gc. thm mt
phn t mi ta so snh n vi kha ca cc nh c trn cy, bt u t gc v i
sang tri nu phn t nh hn kha ca nh t-ng ng v nh ny c con bn tri,
hoc i sang bn phi nu phn t ln hn kha ca nh t-ng ng v nh ny c
con bn phi. Khi phn t nh hn kha ca nh t-ng ng v nh ny khng c

con bn tri, ta to mt nh mi cho phn t ny nh- l nh con bn tri ca nh


ang xt. Phn t ang -c xt -c chn lm kha cho nh mi ny. T-ng t,

nu phn t ln hn kha ca nh t-ng ng v nh ny khng c con bn phi, ta


to mt nh mi cho phn t ny nh- l con bn phi vi kha l phn t ang xt.
minh ha th tc ny, ta xt v d sau.
14, 17.

V d 8.1.3. To cy tm kim nh phn cho dy s sau : 11, 8, 7, 10, 12, 16,


Gii: Cy tm kim nh phn cho dy s trn -c cho trn hnh 3.

11
12

8
7

10

16
14

17

Hnh 3. Cy tm kim nh phn cho dy s 11, 8, 7, 10, 12, 16, 14, 17

Mt s tnh cht c tr-ng ca cy -c cho bi nh l sau.

nh l 8.1.1. Cho T l mt th c n nh. Khi cc khng nh sau l

t-ng -ng :

a) T l mt cy.

b) T l th lin thng khng c chu trnh.


c) T l th lin thng c n-1 cnh.

d) T l th khng c chu trnh v c n-1 cnh.

Chng minh: chng minh cc khng nh trn t-ng -ng vi nhau, chng

ta tin hnh chng minh ln l-t theo s sau: a) b) c) d) a).

a) b): Nu T l mt cy th T l th lin thng, bi v gia 2 nh bt k


ca T lun c -ng i ni chng (theo nh ngha cy). ng thi, T khng c chu

trnh. V nu T c chu trnh cha hai nh x v y ca T, th khi c hai -ng i


gia x v y: -ng i th nht chnh l phn ca chu trnh t x ti y, -ng th hai

l phn cn li ca chu trnh nh-ng theo th t ng-c li (mu thun vi nh ngha


cy).
b) c): Gi s T l th lin thng khng c chu trnh. Ta s chng minh T
c n-1 cnh bng ph-ng php quy np theo n.

Nu n=1, th T c 1 nh v 0 cnh. Do vy khng nh trn ng vi n=1. Gi

s khng nh trn l ng vi th n nh lin thng khng c chu trnh. Gi T l


th c n+1 nh lin thng khng c chu trnh. Ta chn trong T mt -ng i P c
di ln nht (cha nhiu cnh ca T nht). Do T khng c chu trnh cho nn P
khng phi l mt chu trnh. Do vy P phi cha t nht mt nh v c bc bng 1.

Xt th T nhn -c t T bng cch loi b nh v v cnh (duy nht) lin thuc


vi v. R rng T l th c n nh lin thng v khng c chu trnh. Theo gi thit
quy np T c ng n-1 cnh. Do T c n cnh. Khng nh trn -c chng minh
hon ton.

c) d): Gi s T l th lin thng c n-1 cnh. Ta cn chng minh T khng

c chu trnh. Tht vy, gi s T c cha chu trnh. Do vic loi b mt cnh no
nm trong chu trnh hon ton khng lm mt tnh lin thng ca th, cho nn ta
c th loi b dn cc cnh, nh-ng vn gia nguyn cc nh, ra khi cc chu trnh
c th c trong T cho ti khi th T nhn -c cui cng l th lin thng v
khng c chu trnh. L-u rng T cng c n nh. Do s dng kt qu chng
minh b) c) ta -c T c n-1 cnh. Th nh-ng iu c ngha l T c nhiu hn
n-1 cnh. Mu thun ny chng t T phi l th khng c chu trnh.

d) a) Gi s T khng c chu trnh v c n-1 cnh. Ta cn chng t T l cy,


tc l T l n th v T lun c mt -ng i n duy nht ni hai nh bt k ca
T. Do T khng c chu trnh cho nn T khng c bt k mt khuyn no. T-ng t, T
khng th cha hai cnh e1 , e2 khc nhau cng lin thuc hai nh v, w v trong

tr-ng hp ng-c li th T s cha chu trnh v, e1 , w, e2 , v (mu thun vi gi


thit T khng c chu trnh!). Nh- vy T l n th.

Gi s T khng lin thng v T1 , T2 , Tk t-ng ng c n 1 , n 2 , n k nh, l

cc thnh phn lin thng ca T. Do T khng lin thng cho nn k > 1. V do T 1 , T2

, Tk lin thng, khng c chu trnh, cho nn theo kt qu chng minh b) c) ta


suy ra T1 , T 2 , T k c t-ng ng n 1 -1, n 2 -1, n k-1 cnh. Mt khc, ta li c s
cnh ca T (l n-1) bng tng s cnh v s nh ca T (l n) bng tng s nh ca
cc thnh phn lin thng, v vy:

n-1 = n 1 -1 + n 2 -1 + + n k-1 < (n 1 + n 2 + + n k ) 1 (do k > 1) = n -1

iu v l ny chng t T l th lin thng. nh l -c chng minh hon ton.


8.2. Cc ph-ng php duyt cy

Cy c gc v -c sp th t th-ng -c dng l-u tr thng tin. truy

nhp d liu l-u tr ti cc nh ca cy cn c cc th tc duyt tt c cc nh ca n. Sau


y chng ta s m t mt s thut ton duyt tt c cc nh ca cy.

H a ch ph dng
Cc th tc duyt tt c cc nh ca cy c gc v -c sp th t u da trn

vic sp th t cc nh con. Trong cc cy c gc v -c sp th t, khi v th c


h-ng ca chng, cc con ca mt nh trong -c th hin theo th t t tri sang phi.

nh ngha 8.2.1. H a ch ph dng ca mt cy c gc v -c sp l mt

cch sp th t ton b cc nh ca cy bng ph-ng php truy hi nh- sau :

1. Gn nhn cho gc bng s nguyn 0. Sau k nh con ca n ( mc 1) t

tri sang phi -c gn cc nhn l 1, 2, 3, ... k.

2. Vi mi nh v mc n c nhn l A, th k v nh con ca n t tri qua phi

-c gn nhn l A.1, A.2, ..., A.k v .

Theo th tc ny, nh v mc n vi n > 1, c nhn l x1 x 2 ...xn , trong

-ng i duy nht t gc ti v s i qua nh th x 1 mc 1, nh th x2 mc 2,


v.v...

Khi cc nh -c gn nhn trong h a ch ph dng nh- trn, ta c th


sp tt c cc nh ca cy theo th t t in cc nhn ca chng. nh c nhn

x1 x 2 ...xn l nh hn nh c nhn y 1 y 2 ...y m nu c mt gi tr ca i, 0 < i < n, sao cho


x1 = y 1 , x2 = y 2 , ... xi-1 = y i-1 v x i < y i hoc nu n < m v x i = y i vi i = 1, 2, ... n.
V d 8.2.1. Chng ta s gn nhn theo a ch ph dng cho tt c cc nh ca

cy trn hnh 1. Th t t in ca cc nhn l :

0 < 1 < 1.1 < 1.2 < 2 < 3 < 3.1.1 < 3.1.2 < 3.1.2.1 < 3.1.2.2 < 3.1.2.3 < 3.1.2.4 < 3.1.3 <
3.2 < 4 < 4.1 < 5 < 5.1 < 5.1.1 < 5.2 < 5.3

1,1 1,2 1,3


3,1,1

3,1,2,1

3,1
3,1,2

3,1,2,2 3,1,2,3

3,1 4,1
3,1,3

5,1

5,2 5,3

5,1,1

3,1,2,4

Hnh 1. H a ch ph dng ca cy c gc
-c sp th t

Cc thut ton duyt cy


Cc th tc ving thm mt cch c h thng tt c cc nh ca mt cy c gc
v -c sp th t -c gi l cc thut ton duyt cy. D-i y s gii thiu ba thut
ton -c s dng th-ng xuyn nht: duyt tin th t, duyt trung th t v duyt hu
th t.
Thut ton 8.2.1. Duyt cy theo kiu tin th t. -c thc hin nh- sau. Gi

s T l mt cy c gc v -c sp th t vi gc r. Nu T ch c duy nht gc r th r l

cch duyt tin th t ca T. Nu khng th gi T1, T2 ... Tn l cc cy con ti r t tri


qua phi ca T. Duyt tin th t s ving thm r u tin. Tip tc duyt T1 theo kiu
tin th t, sau duyt T2 theo kiu tin th t, c nh- vy cho n khi Tn -c duyt
theo kiu tin th t.

C th kim tra -c rng cch duyt kiu tin th t mt cy c gc v -c sp

cho ta mt th t cc nh ht nh- l th t nhn -c theo h a ch ph dng. Hnh 2


biu th cch duyt theo kiu tin th t.

T1

T2

B-c 2:
Thm T1
kiu tin
th t

B-c 3:
Thm T2
kiu tin
th t

B-c 1: Thm r

Tn
B-c n+1:
Thm T1
kiu tin
th t

Hnh 2. Duyt cy kiu tin th t

V d 8.2.2. Xc nh trnh t ving thm cc nh ca cy c gc v -c sp trn

hnh 3 bng cch duyt tin th t

Gii: Cc b-c duyt tin th t cy T

-c biu th trn hnh 4. Chng ta duyt T


theo cch tin th t bng vic ving thm gc

a u tin. Sau l duyt tin th t cy con


c gc b, duyt tin th t cy con c gc c

(n ch gm c) v duyt tin th t cy con c


gc d.
Duyt tin th t cy con c gc b

bt u bng cch lit k b sau l cc nh


ca cy con c gc e theo kiu tin th t v

j
n

l
o

T
d
g

h i
m

sau l cy con c gc f theo kiu tin th t


Hnh 3. Cy c gc v -c sp T
(n ch c f). Lit k theo kiu tin th t ca

cy con gc d bt u bng vic lit k d sau l duyt cy con c gc g theo kiu tin

th t, tip theo l cy con c gc h (n ch c h) v duyt cy con c gc i (n chnh l


i). Do trnh t duyt cc nh ca T theo kiu tin th t l a, b, e, j, k, n, o, p, f, c, d,
g, l, m, h, i.

a
e

n
b

p
f

h i

l
o

Hnh 4. Duyt cy T theo kiu tin th t

l m

Thut ton 8.2.2. Duyt cy theo kiu trung th t. Gi s T l mt cy c gc v

-c sp vi gc r. Nu T ch c r th r l cch duyt trung th t ca T. Nu khng, th

gi T1, T2, ... Tn l cc cy con ti r t tri qua phi ca T. Duyt trung th t s bt u


bng vic duyt T1 theo kiu trung th t, sau ving thm r. Tip tc duyt T2 theo

T1

T2

B-c 1:
Thm T1
kiu
trung th
t

B-c 3:
Thm T2
kiu
trung th
t

B-c 2: Thm r

Tn
B-c n+1:
Thm Tn
kiu trung
th t

Hnh 5. Duyt cy kiu trung th t

kiu trung th t, tip tc duyt T3 theo kiu trung th t, v c tip tc cho n khi Tn
-c duyt theo kiu trung th t.
V d 8.2.3. Th t duyt cc nh ca cy T cho trn hnh 3 theo kiu trung th

t l j, e, n, k, o, p, b, f, a, c, l, g, m, d, h, i.

Thut ton 8.2.3. Duyt cy theo kiu hu th t. Gi s T l mt cy c gc v

-c sp vi gc r. Nu T ch c r th r l cch duyt hu th t ca T. Nu khng, th

gi T1, T2, ..., Tn l cc cy con ti r t tri qua phi ca T. Duyt hu th t s bt u


bng vic duyt T1 theo kiu hu th t, sau duyt T2 theo kiu hu th t v c tip

tc cho n khi Tn -c duyt theo kiu hu th t, v cui cng kt thc bng vic
ving thm r.

B-c n+1: Thm r

T1

T2

B-c 1:
Thm T1
kiu hu
th t

B-c 2:
Thm T2
kiu hu
th t

Tn
B-c n:
Thm Tn
kiu hu
th t

Hnh 6. Duyt cy kiu hu th t

V d 8.2.4. Th t duyt cc nh ca cy T cho trn hnh 3 theo kiu hu th t

l j, n, o, p, k, e, f, b, c, l, m, g, h, i, d, a.

C ba ph-ng php duyt cy nu trn u c tnh quy. lit k cc nh


ca cy c gc v -c sp theo kiu tin, trung hay hu th t, ta v mt -ng cong
xut pht t gc chuyn ng dc theo cc cnh bao quanh cy ang xt, nh- trn hnh
v 7. Khi danh sch cc nh ca cy -c duyt theo kiu tin th t -c xy dng

bng cch lit k mi nh khi -ng cong i qua n. Danh sch cc nh -c duyt
theo kiu trung th t -c xy dng bng cch lit k cc l khi i ngang qua n ln

b
d

c
f

Hnh 7: Cch d nh cc cch duyt cy theo cc kiu


tin, trung, hu th t.

u v lit k cc nh trong khi i ngang qua n ln th hai. V cui cng, danh sch

cc nh -c duyt theo kiu hu th t -c xy dng bng cch lit k cc nh khi


i ngang qua n ln th 2 tr v nh cha ca n.
Ta c th kim tra li theo quy tc ny vi cy c gc trn hnh 7 kiu duyt tin th t

cho ta a, b, d, h, e, i, j, c, f, g, k, kiu duyt trung th t cho ta h, d, b, i, e, j, a, f, c, k, g cn theo


kiu duyt hu th t th ta -c h, d, i, j, e, b, f, k, g, c, a.

8.3. Cy v bi ton sp xp

Bi ton sp xp cc phn t ca mt tp hp xut hin trong rt nhiu lnh


vc. V d nh-: sp xp danh sch th sinh theo h tn nh s bo danh, sp xp
danh sch th sinh theo tng im thi xt trng tuyn
Gi s cn sp xp ton b cc phn t ca mt tp hp. Ban u cc phn t

ca tp hp c th -c sp t theo mt trt t no . Sp xp (sorting) l s sp


t li cc phn t ny vo mt danh sch theo th t tng hoc gim dn. V d, sp
xp danh sch 7, 2, 1, 4, 5, 9 theo th t tng dn s to ra danh sch 1, 2, 3, 4, 5, 7,
9, sp xp danh sch d, h, c, a, f (theo th t t in) s cho danh sch a, c, d, f, h.
D-i y chng ta s trnh by mt vi thut ton sp xp. Ta cng s thy
cy -c dng m t cc thut ton sp xp v phn tch phc tp ca chng

nh- th no.
C kh nhiu thut ton sp xp khc nhau. nh gi hiu qu ca mi

thut ton ng-i ta cn xc nh phc tp ca n. Bng m hnh cy c th tm


-c cn d-i cho tr-ng hp c phc tp ti t nht ca thut ton sp xp.

Xt mt tp gm n phn t. Mi cch sp xp cc phn t ny l mt hon v


ca chng. Do vy c n! cch sp xp chng. Cc thut ton sp xp m chng ta s
nghin cu da trn cc php so snh nh nguyn, tc l mi ln so snh hai phn t
vi nhau. Kt qu ca mi php so snh nh- th s thu hp tp cc cch sp xp c

th. Nh- vy, thut ton sp xp da trn so snh nh nguyn c th -c biu din
bng cy quyt nh nh phn vi cc nh trong l cc php so snh hai phn t, mi
l l mt trong n! hon v ca n phn t.
V d 8.3.1. Trn hnh 1 l biu din cy quyt nh sp xp cc phn t

ca danh sch a, b, c.
phc tp sp xp da trn so snh nh nguyn -c o bng s cc php

so snh -c s dng. S ti a cc php so snh cn dng sp xp mt danh


sch c n phn t chnh l tr-ng hp ti t nht ca thut ton. S ti a cc php so
snh cn dng s bng di ca -ng i di nht trong cy quyt nh biu din
th tc sp xp. Ni cch khc s ti a cc php so snh cn dng s bng chiu

cao ca cy quyt nh. V chiu cao ca cy nh phn vi n! l ti thiu bng


[logn!] nn cn t nht [logn!] php so snh, nh- nh l 8.3.1 sau khng nh.
nh l 8.3.1. Thut ton sp xp da trn so snh nh nguyn i hi t nht
[log n!] php so snh.

C th thy rng [logn!] = O(nlogn). Tht vy, ta c:


log n! > log([n/2])([n/2]) > log n n/4 = (nlogn)/4 vi mi n > 4

Chnh v vy, khng c thut ton sp xp no dng cc php so snh nh- l


mt ph-ng php sp xp c th c phc tp v thi gian trong tr-ng hp ti t
nht li tt hn O(nlogn). Do thut ton sp xp l c hiu qu tt nht c th nu
phc tp thi gian ca n l O(nlogn).
a:b

a>b

a>c

b>c
a>b>c

a:c

b:c

b<c
a>c>b

b<c

b>c

a<c
c>a>b

b:c

a<b

c>b>a

a:c
a>c

b>a>c

a<c
b>c>a

Hnh 1. Cy quyt nh sp xp ba phn t khc nhau

Thut ton 8.3.1. Sp xp kiu ni bt. y l thut ton sp xp n gin nht


nh-ng khng phi l mt trong nhng thut ton c hiu qu nht. Thut ton ny t
danh sch theo th t tng dn bng cch so snh lin tip cc phn t k nhau, i ch
chng cho nhau nu chng ch-a c th t tng dn. tin hnh sp xp kiu ni bt
chng ta thc hin mt thao tc c bn, l s i ch phn t ln hn vi phn t nh
hn i sau, bt u t u danh sch v duyt qua ton b danh sch. Chng ta lp th

tc ny cho ti khi vic sp xp -c hon thnh. Ta hy t-ng t-ng cc phn t -c


t vo mt ct. Trong sp xp kiu ni bt cc phn t nh hn s ni ln trn v
chng i ch vi cc phn t ln hn. Cc phn t ln hn s chm xung y. iu
ny -c minh ha trong v d sau.

V d 8.3.2. Sp xp

3, 2, 4, 1, 5, theo th t tng

dn bng thut ton ni bt.


Gii: Tr-c tin ta so

2
4
1
5

3
4
1
5

3
4
1
5

3
1
4
5

B-c 2

2
3
1
4
5

2
3
1
4
5

2
1
3
4
5

B-c 3

2
1
3
4
5

1
2
3
4
5

B-c 4

1
2

BB-c 1

snh hai phn t 3 v 2. V


3> 2 nn i ch 3 vi 2, ta
-c danh sch 2, 3, 4, 1, 5.
V 3 < 4 ta tip tc s snh

4 vi 1. V 4 > 1 nn i
ch 4 vi 1, ta nhn -c
danh sch 2, 3, 1, 4, 5. V 4
< 5 nn vng duyt th
nht -c hon thnh. Vng
ny m bo phn t ln

nht 5, -c t vo ng v
tr ca n.
Vng duyt th hai
bt u bng vic so snh 2
v 3. V chng ng th t
cn sp xp nn ta so snh 3

v 1. V 3 > 1 nn i ch
chng cho nhau ta -c
danh sch 2, 1, 3, 4, 5. V 3
< 4 nn cc s ny

ng th t. Khng cn so
snh tip v cc phn t 4 v
5 ng v tr ca
chng.

3
4

: i ch
: cp ng th
t

Cc s trong khung
m bo th t ng
Hnh 2. Cc b-c sp xp theo kiu ni bt

Vng th t- ch cn cho php so snh gia 1 v 2. V 1< 2 nn chng


ng th t. Thut ton sp xp kiu ni bt kt thc.

Cc b-c ca thut ton ny -c th hin trn hnh 2.


By gi ta xt mc hiu qu ca sp xp kiu ni bt. V trong vng th i

cn phi thc hin (n-i) php so snh, nn tng hp cc php so snh trong sp xp
kiu ni bt l :
(n-1) + (n-2) + ... + 2 + 1 = n(n-1)/2.

Do , sp xp kiu ni bt dng n(n-1)/2 php so snh sp th t mt


danh sch n phn t. V th thut ton sp xp ni bt c phc tp trong tr-ng
hp xu nht l O(n 2 ). Mt khc, vi mi s d-ng thc c, ta lun c n(n-1)/2 >
c.nlogn, vi n ln, ta suy ra sp xp kiu ni bt khng t -c phc tp thi

gian O(nlogn) trong tr-ng hp xu nht.


Tip theo, chng ta xem xt mt trong nhiu thut ton sp xp c phc tp
trong tr-ng hp xu nht t -c gi tr ti -u, tc l O(nlogn) php so snh sp
xp n phn t: sp xp kiu ha nhp.
Thut ton 8.3.2. Sp xp kiu ho nhp. -c thc hin bng cch phn i
lin tip cc danh sch thnh hai danh sch con c di bng nhau (hoc hn km
nhau mt phn t) cho ti khi mi danh sch con ch gm mt phn t. Dy danh
sch con ny c th biu din bng cy nh phn cn i. Th tc tip tc bng cch
ho nhp ln l-t cc cp danh sch c th t tng dn cho ti khi ton b danh
sch ban u -c sp theo th t tng dn. Dy danh sch ho nhp -c biu din
bng cy nh phn cn i. Ta xt mt v d minh ho cho thut ton ny.

824

8
8

82

2
2

82

82469
4

8 2 4 6 9 7 10 1 5
69

824

9
69

24689

7 10 1

7
7

7 10

1
1

7 10

7 10 1

7 10 1 5
1

5
7 10 1 5

53

3
53

123456789
10 sch 8, 2, 4, 6, 9, 7, 10, 1, 5, 3 .
Hnh 3. Sp xp kiu ho nhp danh
V d 8.3.3. Chng ta s sp xp danh sch 8, 2, 4, 6, 9, 7, 10, 1, 5, 3 theo

thut ton sp xp kiu ho nhp. Theo thut ton ny tr-c tin ta chia i lin tip

danh sch cho thnh hai danh sch con. Dy cc danh sch con ng vi v d ny
-c biu din bng cy nh phn cn i c chiu cao bng 4 (xem na trn ca
hnh 3).

Vic sp xp -c tin hnh bng cch ho nhp ln l-t cc cp danh sch.


b-c u tin, hai phn t -c ho vo danh sch theo th t tng dn. Sau s ln
l-t ho nhp cc cp danh sch cho ti khi ton b danh sch -c sp xp theo th
t tng dn. Dy cc danh snh -c ho nhp theo th t tng dn -c biu din
bng cy nh phn cn i chiu cao bng 4 (Cy ny -c trnh din ln ng-c
na d-i ca hnh 3).

sp xp theo kiu ho nhp, ta cn c mt thut ton hiu qu ho nhp


hai danh sch c th t thnh mt danh sch c th t ln hn. By gi ta s m t
thut ton nh- vy.
V d 8.3.4. Chng ta s m t cch ho nhp danh sch c th t 2, 3, 5, 6 v
1, 4. Bng 1 minh ho cc b-c chng ta s dng.
Tr-c tin so snh hai phn t nh nht trong hai danh sch l 2 v 1. V 1
nh hn nn t n vo u ca danh sch ho nhp v xo n khi danh sch th
hai. Cui giai on ny danh sch u l 2, 3, 5, 6 v danh sch th hai l 4 cn danh
sch ho nhp l 1, 2.
Tip theo so snh 2 v 4, hai phn t nh nht ca hai danh sch. V 2 l nh

hn nn gp 2 vo danh sch ho nhp v xo n khi danh sch th nht. Cui giai


on ny danh sch u l 3, 5, 6 v danh sch th hai l 4 cn danh sch ho nhp
l 1, 2 .
Tip tc so snh 3 v 4 l hai phn t nh nht ca hai danh sch. V 3 nh

hn nn thm 3 vo danh sch ho nhp, ri xo n khi danh sch 1. Cui giai on


ny danh sch u l 5, 6 v danh sch th hai l 4 cn danh sch ho nhp l 1, 2, 3.

By gi tip tc so snh 4 v 5 l hai phn t nh nht ca hai danh sch. V


4 nh hn nn thm 4 vo danh sch ho nhp, ri xo n khi danh sch th 2. Cui

giai on ny danh sch u l 5, 6 danh sch th hai rng cn danh sch ho nhp
l 1, 2, 3.
Cui cng v danh sch th hai rng nn tt c cc phn t ca danh sch 1
-c ni vo cui ca danh sch ho nhp theo th t m chng c danh sch 1.
Kt qu chng ta nhn -c danh sch -c sp theo th t tng dn 1, 2, 3, 4, 5, 6.
Bng 1. Ho nhp hai danh sch 2, 3, 5, 6 v 1, 4

Danh sch 1

Danh sch 2

Danh

2 3 5 6

1 4

2 3 5 6
3 5 6

4
4

nhp

1 2
1 2 3

sch

ho So snh
1< 2
2< 4
3< 4

5 6

5 6

4
4

1 2 3 4

1 2 3 4 5 6

4< 5

Thut ton 8.3.3. Ha nhp hai danh sch c th t. Mt cch tng qut, vic

ho nhp hai danh sch c th t L 1 v L 2 vo mt danh sch c th t L -c thc


hin nh- sau: L -c khi to l mt danh sch rng. So snh hai phn t nh nht
ca hai danh sch. t phn t nh hn trong hai phn t ny vo cui danh sch L,

v xo n khi danh sch m n c mt. Tip theo, nu mt trong hai danh sch
L 1 v L 2 l rng th ni danh sch kia (danh sch khng rng) vo L v th tc ho
nhp kt thc. Nu c L 1 v L 2 u khng rng th lp li qu trnh trn.
By gi chng ta s nh gi s php so snh cn dng trong th tc ho nhp
hai danh sch c th t, mt th tc c bn ca thut ton sp xp ho nhp. Mi ln
c mt php so snh phn t ca L 1 vi mt phn t ca L 2 th li c mt phn t
-c ghi vo danh sch ho nhp L. Tuy nhin khi L 1 hoc L 2 l rng th khng cn
phi so snh na. K hiu m v n t-ng ng l cc phn t ca L 1 v L 2 . R rng

thut ton 8.3.3 c hiu lc km nht khi c m + n - 2 php so snh -c thc


hin, m mi danh sch vn cn mt phn t. Ta cn phi lm mt so snh na th s
c mt danh sch rng. V th thut ton 2 dng khng qu m + n - 1 php so snh.
By gi chng ta c th phn tch phc tp ca thut ton sp xp kiu

ho nhp. Thay cho vic nghin cu bi ton tng qut chng ta gi s s phn t
ca danh sch l lu tha ca 2, tc l n = 2 m. iu lm cho vic phn tch bt
cng knh hn, nh-ng khi n 2 m, dng nhng thay i khc nhau cng s cho mt
nh gi nh- th.

u tin ta chia danh sch lm hai danh sch con mc 1 ca cy nh phn,


mi danh sch c 2 m-1 phn t. Tip tc chia hai danh sch con ny thnh bn danh
sch con mc 2, s c 2 k+1 danh sch mc k - 1, mi danh sch c 2 m-k+1 phn t.
Cc danh sch mc k - 1 -c phn chia thnh 2 k danh sch mc k, mi danh

sch c 2 m-k phn t. Kt thc qu trnh phn chia, mc m ta c 2 m danh sch, mi


danh sch c ng mt phn t.
Chng ta bt u ho nhp mi cp ca 2 m danh sch c 1 phn t thnh 2 m-1
danh sch mc (m-1) mi danh sch c 2 phn t. Vic ho nhp mi cp cn ng

1 php so snh.
Th tc tip tc sao cho mc k (k= m, m - 1, m - 2, ... 3, 2, 1), 2 k danh sch,

mi danh sch c 2 m - k phn t mc k-1. Khi ta cn tt c 2 k-1 php ho nhp hai


danh sch, mi danh sch c 2 m-k phn t. Theo phn tch trn, mi php ho nhp

cn nhiu nht 2 m-k + 2 m - k - 1 = 2 k-1 ( 2 m-k+1 - 1) php so snh. Ly tng tt c cc


nh gi ny ta s nhn -c s cc php so snh cn thit cho thut ton ho nhp,
nhiu nht l

2 k 1 (2 m k 1 1)
m

k 1

2 m (2 k 1 1)
m

k 1

k 1

= m 2 - ( 2 - 1)
= nlogn -n + 1 , v m = logn v n = 2 m .
m

Vy thut ton ho nhp t -c nh gi O-ln tt hn v s cc php so


snh cn thit. iu ny -c pht biu bng nh l sau.
nh l 8.3.2. S cc php so snh cn thit trong thut ton sp xp kiu ho
nhp mt danh sch n phn t l O(nlogn).

Mt thut ton sp xp na, cng rt c hiu qu, l thut ton sp xp


nhanh (quick sort) s -c m t trong phn bi tp.
8.4. Cy khung

Trong phn ny chng ta nghin cu tip mt ng dng ca cy trong vic

gii mt s bi ton thc t khc nh-: xc nh s lin kt ti thiu m bo s


lin kt ca tt c cc my tnh trong mt mng my tnh, xc nh s tuyn -ng

ti thiu cn thit m bo giao thng gia cc thnh ph Cc bi ton nh- vy


c th gii bng cch xy dng mt th con c mt s ti thiu cc cnh v cha
tt c cc nh ca th m t mng my tnh hoc mng giao thng gia cc thnh
ph.
nh ngha 8.4.1. Mt cy T -c gi l cy khung ca th G nu n l
mt th con ca G v cha tt c cc nh ca G.
V d 8.4.1. Tm cy khung a
b
ca th G trn hnh 1.

Gii: th G lin thng,


nh-ng khng l mt cy v n cha
chu trnh n. Xo cnh {a, e}.
iu loi -c mt chu trnh,
th con nhn -c vn cn lin
thng v cha tt c cc nh G.

Hnh 1. th n G

Tip theo xo cnh {e, f } s loi


-c mt chu trnh na. Cui cng xo cnh {c, g} s thu -c mt n th

khng c chu trnh. th ny l cy khung v n l cy v cha tt c cc nh ca


G. Dy cc cnh b xo i khi to ra cy khung -c minh ho trn hnh 2.

xo cnh {a,e}

xo cnh {e,f}

xo cnh {c,g}

Hnh 2. To ra cy khung ca G bng cch xo cc cnh to ra chu trnh


n

g
b

Hnh 3. Cc cy khung ca G.

Cy trn hnh 2 khng phi l cy khung duy nht ca G. V d, mi mt


trong bn cy trn hnh 3 u l cy khung ca G.
T nh ngha cy khung ta c nh l sau
nh l 8.4.1. th G c cy khung khi v ch khi G l th lin thng.
Chng minh : Tr-c tin gi s th G c cy khung T. Do T l th con

lin thng cha tt c cc nh ca th G, cho nn G l lin thng.

By gi ta gi s G l th lin thng. Nu G khng phi l mt cy th n


phi c chu trnh n. Ta xo i mt cnh ca mt trong cc chu trnh n ny,

th nhn -c cha mt s cnh t hn nh-ng vn cn cha tt c cc nh ca G v


vn lin thng. Cng ging nh- trn ta li xo i mt cnh ca chu trnh n khc.
Lp li qu trnh ny cho n khi khng cn chu trnh n. iu ny l c th v ch

c mt s hu hn cc cnh trong th. Qu trnh kt thc khi khng cn chu trnh

j
i

b
Hnh 4. th G

n trong th nhn -c. Kt qu thu -c l mt cy, v th vn cn lin


thng khi xo i cc cnh. Cy ny l cy khung v n cha tt c cc nh ca G.
Cch chng minh nh l 8.4.1 cho ta mt thut ton tm cy khung bng
cch xo i cc cnh khi cc chu trnh n. Thut ton ny l khng hiu qu v n

i hi phi nhn bit -c cc chu trnh n. Thay cho vic xy dng cy khung
bng cch loi b cc cnh, c th xy dng -c cy khung bng cch ln l-t ghp
cc cnh. Chng ta s gii thiu hai thut ton da trn nguyn tc ny. Cy khung
-c xy dng l mt cy c gc.
Thut ton 8.4.1. Xy dng cy khung bng ph-ng php tm kim theo chiu
su.

Input: Cho th lin thng G vi tp cc nh -c sp xp theo mt trnh

t no v1 , v2 ,, vn .
Output: Cy khung T ca th G.
1.

[Khi to] t w:=v1 . Gi T l th cha mt nh v1 v khng c

2.

[Chn cnh] Chn cnh (w, vk) vi k nh nht c th -c sao cho khi

3.
4.

5.

cnh no. Coi nh v1 l gc ca cy T.

thm cnh vo T khng to thnh chu trnh trong T. Nu cnh nhvy khng tn ti th ti b-c 4.

[Thm cnh] Thm cnh (w, vk ) cng nh vk vo T ; t w := vk v


quay tr li b-c 2.

[Kt thc?] Nu w:=v1 , kt thc thut ton (T l cy khung cn tm).

[Quay lui] Gi s x l b ca w (trong cy T). t w :=x, quay li b-c


2.

V d 8.4.2. Dng thut ton tm kim theo chiu su, tm cy khung ca

th G cho trn hnh 4 .


Gii: Cc b-c dng trong thut ton tm kim theo chiu su xy dng
cy khung ca G -c biu th trn hnh 5. Chng ta xut pht t nh a. Cc nh

tip theo tun t -c thm vo T l b, c, d, e. Khng thm -c nh no t nh e.


Li li d. Khng thm -c nh no t d, v th li ti c. T c c th thm nh g.

Sau li li v c v tip tc li v a. T a li c th thm cc nh f, h, i, j. Tip


, ta phi li li v i v thm -c nh k. T k li v tip n a ln na v kt thc
vic xy dng T.

b
g

g
k

Hnh 5. Kt qu tm kim -u tin chiu su ca G

rng.

Thut ton 8.4.2. Xy dng cy khung bng ph-ng php tm kim theo chiu
Input: Cho th lin thng G vi tp cc nh -c sp xp theo mt trnh

t no v1 , v2 ,, vn .
Output: Cy khung T ca th G.
1.

[Khi to] t S:={v1 } . Gi T l th cha mt nh v1 v khng c

2.

[Thm cnh] Vi mi nh x S theo th t sp xp cc nh trong G

3.

cnh no. Coi nh v1 l gc ca cy T.

ta ln l-t thm cnh (x,y) v nh y theo th t sao cho vic thm cnh
vo T khng to thnh chu trnh trong T.

[Thay i S] Nu tt c cc nh ca G -c ghp vo T th kt

thc thut ton (T l cy khung cn tm). Tr-ng hp ng-c li, thay S


bng tp cc con ca S (trong cy T). Quay li b-c 2.

D-i y l mt v d minh ho thut ton xy dng cy khung bng ph-ng


e
b
c
php tm kim theo chiu rng.
l
V d 8.4.3. Dng thut ton tm kim theo
chiu rng, tm cy khung ca th hnh 6.

Gii : Cc b-c ca th tc tm kim -u


g
tin chiu rng -c biu din hnh 7. Ta chn
nh a lm gc ca cy, S:={a}. Sau ta thm
cc cnh lin thuc v tt c cc nh lin k vi
a, tc l cc cnh t e ti b, d, f v i -c ghp
vo T. Tip theo thay S={a} bng cc con ca S
(trong T). Tc l S :={b, d, f, i}. V th cc cnh

a
b

a
d

j
k

Hnh 6. th G

i
c
g

b
e
j

d
h

f
j

i b

k a

c
l

h
m

Hnh 7. Tm kim -u tin chiu rng ca G.

t b ti c v e -c ghp vo T ; cng nh- th vi cc cnh t d ti h, t f ti j v g,

v t i ti k. Thay S bng cc con ca S trong T, tc l S :={c, e, g, h, j, k}. Tip theo


ghp cc cnh t cc nh ny ni vi cc nh cn ch-a thuc vo cy. Tc l ghp
thm cc cnh t g ti l v t k ti m.
8.5. Cy khung nh nht

Trn th G hnh 1 ch ra cho ta 6 a im dn c- a, b, c, d, e, f v chi ph

xy dng cc -ng giao thng ni chng vi nhau. Vn t ra l cn thit k


mt mng l-i -ng giao thng cho php i t bt k im no ti tt c cc im
cn li vi chi ph nh nht. D dng thy rng li gii bi ton ny s l mt cy
khung T ca th G sao cho tng trng s cc cnh ca T l nh nht.

a
3

c
6

4
2

3
6

2
f

Hnh 1. th G

Mt lp rt rng cc bi ton c th gii bng cch tm cy khung trong mt


th c trng s sao cho tng trng s ca cc cnh ca cy l nh nht t-ng t
nh- trn.
nh ngha 8.5.1. Cy khung nh nht trong mt th lin thng, c trng

s l mt cy khung c tng trng s trn cc cnh ca n l nh nht.


D-i y ta s gii thiu hai thut ton xy dng cy khung nh nht. C hai

u -c tin hnh bng cch ghp cc cnh c trng s nh nht trong s cc cnh
c mt tnh cht no m ch-a -c dng. Nhng thut ton ny l nhng v d v

cc thut ton tham lam. Thut ton tham lam l mt th tc thc hin mt la chn
ti -u mi giai on. Ti -u ho mi giai on ca thut ton khng m bo to
ra li gii ti -u ton cc. Nh-ng hai thut ton gii thiu trong mc ny xy
dng cy khung nh nht l cc thut ton tham lam to ra cc li gii ti -u.
Thut ton 8.5.1. Thut ton Prim. Do Robert Prim -a ra vo nm 1957, mc
d t-ng c bn ca n c t sm hn rt nhiu. Thut ton bt u bng vic

chn mt cnh bt k c trng s nh nht, t n vo cy khung. Ln l-t ghp vo


cy cc cnh c trng s ti thiu lin thuc vi mt nh ca cy v khng to ra
chu trnh trong cy. Thut ton s dng khi n - 1 cnh -c ghp vo cy.
L-u rng vic chn mt cnh ghp vo cy trong mi giai on ca thut

ton l khng xc nh khi c nhiu hn mt cnh cng trng s v tho mn nhng


tiu chun no . Do vy chng ta cn sp xp cc cnh theo mt th t no
vic chn mt cnh -c xc nh. Khi chng ta s khng phi lo lng g v iu

ny. Cng cn l-u l c th c nhiu hn mt cy khung nh nht ng vi mt


th lin thng v c trng s. V d sau minh ho cch dng thut ton Prim.
V d 8.5.1. Dng thut ton Prim thit k mt mng -ng giao thng c
chi ph ti thiu ni cc im dn c- -c biu din trn hnh 1 sao cho c th i t
bt k im dn c- no ti cc im cn li.
Gii: Chng ta s gii bi ton B-c chn Cnh
1
2
3
4
5

ny bng cch tm cy khung nh


nht trong th trn hnh 1. Thut
ton Prim -c tin hnh bng cch
chn cnh u tin l cnh c trng

s nh nht v ln l-t ghp thm


mt cnh c trng s nh nht trong

Ph tn

{c, d}
{c, a}
{a, e}
{e, f}
{a, b}
Tng cng

2
12

1
2
3
4

Hnh 2. Thut ton Prim xy dng cy khung nh


nht i vi th c trng s hnh 1

s nhng cnh ni vi mt nh ca
cy v khng to thnh mt chu trnh (xem hnh 2). Cc cnh -c t m trn hnh
3 l cy khung nh nht nhn -c bng thut ton Prim.

c
6

4
2

3
6

2
f

Hnh 3. Cy khung nh nht ca th hnh 1

Thut ton 8.5.2. Thut ton Kruskal. Do Joseph Kruskal pht minh vo nm

1956, mc d t-ng c bn ca n cng -c bit t sm hn nhiu. Thut ton


Kruskal bt u bng vic chn cnh c trng s nh nht ca th. Ln l-t ghp

thm vo cnh c trng s ti thiu v khng to thnh chu trnh vi cc cnh


-c chn. Thut ton dng sau khi n - 1 cnh -c chn.
S khc nhau gia thut ton Prim v thut ton Kruskal l ch: Trong thut
ton Prim ta chn cc cnh c trng s ti thiu, lin thuc vi cc nh thuc cy

v khng to ra chu trnh. Trong khi theo thut ton Kruskal s l chn cc cnh
c trng s ti thiu m khng nht thit phi lin thuc vi cc nh ca cy
chn v khng to ra chu trnh. Ch rng cng nh- trong thut ton Prim nu cc
cnh khng -c sp th t c th c nhiu cch chn trong mi b-c ca thut ton
ny. Do cho th tc xc nh cn sp xp cc cnh theo mt trt t no . V
d sau y s minh ho cch dng thut ton Kruskal.
Ph tn
V d 8.5.2. Nu kt qu B-c chn Cnh
1
{c, d}
1
2
{c, a}
2
3
{e, f}
2
4
{a, e}
3
5
{a, b}
4
Tng cng
12
Hnh 4. Thut ton Kruscal xy dng cy khung
nh nht i vi th c trng s hnh 1

thc hin thut ton Kruskal tm

cy khung nh nht ca th trn


hnh 1.
Gii: Cy khung nh nht
cho hnh 3, trnh t chn cc cnh
trong mi b-c ca thut ton
Kruskal -c th hin trn hnh 4.
9. Mng. Lung trn mng

Bi ton lung cc i trong mng l mt trong s nhng bi ton ti -u trn

th tm -c nhng ng dng rng ri trong thc t cng nh- nhng ng dng th v


trong l thuyt t hp. N -c xut vo u nhng nm 1950 v gn lin vi tn tui
hai nh ton hc M l Ford v Funkerson.
9.1. Cc khi nim:

Xt th c h-ng biu din mt h thng -ng ng dn du (hnh 1). Du -c

bm t tu ch du (nh a), qua cc trm bm trung chuyn (cc nh b, c, d, e) ti b cha


(nh z). Cc cung c h-ng ca th biu din cc -ng ng dn v cho bit h-ng di

chuyn ca lung du trong ng dn. Cc s trn cc cung l kh nng thng qua (c th hiu l
tit din) ca cc ng dn du t-ng ng. Vn l ch cn xc nh lung du ln nht c

th bm t tu ch du vo
b cha v bm bng cch
no. th c h-ng trn
hnh 1 l mt v d v mng
vn ti (gi tt l mng).

Bi ton nh- vy

c th xut hin trong rt


nhiu ng dng thc t

5
d

Hnh 1. Mng ng dn du G

khc. Chng hn khi cn xc nh c-ng ln nht ca dng vn ti gia 2 nt ca


mt bn giao thng. Li gii ca bi ton lung cc i s ch cho ta cc on -ng
ng xe nht v chng to thnh ch hp t-ng ng vi dng giao thng xt theo 2
nt -c chn.

nh ngha 9.1.1. Mng l n th c h-ng, c trng s G = (V, E) tho mn cc

iu kin:

a) C duy nht mt nh a khng c cung i vo gi l im pht


b) C duy nht mt nh t khng c cung i ra gi l im thu
c) Trng s ca mi cung e = (v, w) E l mt s khng m c(e) = c(v,w), -c gi l
kh nng thng qua ca cung e.

d) th v h-ng nhn -c t G bng cch b qua h-ng ca cc cung l th lin


thng.

Trong mc ny chng ta s lun k hiu im pht l a v im thu l s.


nh ngha 9.1.2. Gi s G = (V, E) l mt mng vi kh nng thng qua ca cc cung

k hiu l c(v,w). Ta gi lung f trong mng G = (V, E) l nh x f : E R+ gn cho mi cung e =


(v, w) E mt s thc khng m f (e) = f(v,w), gi l lung trn cung e, tho mn cc iu kin sau:
1) Lung trn mi cung e E khng v-t qu kh nng thng qua ca n:
0 = f(e) c(e)
2) Vi mi nh v khng phi l nh thu hoc nh pht, tng lung trn cc cung i vo nh

v (lung vo nh v) bng tng lung trn cc cung i ra khi nh (lung ra khi nh v):

f ( w, v)

f (v, w)

(iu kin cn bng lung trn mi nh ca mng)

Sau ny, trong cc tng kiu nh- trn, nu khng c l-u g khc, ta c th coi

tng -c ly trn ton b tp cc nh ca mng. Vi nhng tr-ng hp (w,v) hoc


(v,w) khng phi l cung trn mng ta coi cc gi tr f(v,w), f(w,v) l bng 0.
V d 9.1.1. Cc gi tr:
f ( a, b ) 2, f (b, c ) 2, f (c, z ) 3
f ( a, d ) 3, f ( dc ) 1, f ( d , e) 2, f (e, z ) 2

to thnh mt lung trn mng hnh 1. D dng thy, v d, lung vo nh d l


f(a,d)=3 cng bng lung ra khi nh d l f(d,c)+f(d,e)=1+2=3.

Trn hnh 2 chng ta v li lung cho trong v d 9.1.1 trn mng G ca hnh 1.
Cc cp s x,y vit trn mi cung t-ng ng l kh nng thng qua v lung trn cung
y. Cch biu din ny s -c s dng trong sut nhng phn tip theo ca mc ny.

2,

3,

c
4,

2,

5,
d

2,

4,

Hnh 2. Lung trn mng ng dn du G

Ta c th thy trong v d trn, lung ra khi nh pht a l f(a,b)+f(a,d)=2+3=5

cng chnh l gi tr ca lung i vo nh thu z: f(c,z)+f(e,z)=3+2=5. nh l sau y


cho thy iu ny cng ng cho lung trn bt k mng no.

nh l 9.1.1. Cho f l mt lung trn mt mng no . Khi y ta lun c lung

ra khi nh pht bng lung i vo nh thu. Tc l

f ( a , v ) f (v , z ) .
v

Chng minh: K hiu V l tp cc nh, E l tp cc cung ca mng. Ta c:

f (v, w) f (w, v) ,

wV

vV

wV

vV

bi v cc tng kp biu thc trn thc ra u l

f (e) . Do vy:
eE

0 f (v, w) f (w, v)
wV vV
vV

f (v, z ) f ( z , v) f (v, a ) f (a, v) f (v, w) f ( w, v)


vV
vV
vV
vV
vV
wV , vV

w a , z

f ( v, z ) f ( a , v )
vV

vV

Trong cc bin i trn, chng ta s dng nh ngha 9.1.1:


f (v , a ) 0 f ( z , v ), v V ;
f ( v, w) f (v, w), w V \{a, z} .

vV

vV

Do nh l trn, ta c th nh ngha:

nh ngha 9.1.2. Gi s f l mt lung trn mng G. Khi i l-ng:


Val ( f ) f (a, v) f (v, z )
vV

vV

-c gi l gi tr ca lung f trn mng.

9.2 Thut ton tm lung cc i trong mng

Chng ta s xem xt mt thut ton gii bi ton sau: Cho mng G = (V, E) vi
nh pht a v nh thu z. Trong s tt c cc lung c th c ca mng G hy tm lung
f* sao cho gi tr lung val(f*) l ln nht. Lung nh- vy ta s gi l lung cc i trong
mng. t-ng chnh ca thut ton l: xut pht t mt dng ban u no (v d
dng ng nht bng 0 trn mi cung) tng b-c ta tm cch tng dn gi tr ca lung
cho ti khi khng th tng -c na.

trnh by thut ton chng ta cn -a vo mt vi thut ng. Gi P l mt


-ng i v h-ng t a ti z (hiu theo ngha -ng i trn th v h-ng t-ng ng
vi G) v gi s:
P (v0 , v1, ..., vn ), v 0 =a, v n =z.

Tt c mi -ng i trong mc ny u -c hiu l -ng i v h-ng nh- trn.

nh ngha 9.2.1. Nu cung e trong P c h-ng trong G trng vi h-ng t


vi 1 vi th ta ni rng e l cung thun trn -ng P (gi tt l cung thun); trong

tr-ng hp ng-c li ta ni rng e l cung nghch trn -ng i P (gi tt l cung


nghch).

R rng l, nu ta tm -c -ng i P t nh pht a ti nh thu z sao cho tt c

cc cung trn P u l cung thun v lung trn mi cung u nh hn kh nng thng


qua ca cung, th ta c th tng gi tr ca lung hin c.

V d 9.2.1. Xt -ng i t a ti z trn hnh 1. Ta c th tng lung trong mng

ln 1.

3,1

b
2,1

3,2

4,1

b
2,2

1a)

4,2

c
1b)

Hnh 1. Lung trn mng 1a) c th tng 1 thnh lung 1b)

Cng c th c kh nng tng lung trn cc -ng t nh pht ti nh thu

ngay c trong tr-ng hp -ng i c cung thun v cung nghch. Tht vy, gi s P l
-ng i t nh a ti z v x l mt nh nm trn P ( x a , x z ). C 4 kh nng nh
h-ng cc cung e1 v e2 lin thuc vi x (hnh 2).

a
a
a
a

e1
e

e1
e

e1
e
e1
e

x
x
x
x

2a)

2b)

2c)

2d)

Hnh 2. Cc kh nng nh h-ng cung lin thuc vi x


Trong tr-ng hp 2a) c 2 cung u l cung thun, do vy nu ta tng lung trn
mi cung e1, e2 ln mt i l-ng th lung i vo nh x vn bng lung ra khi x
(iu kin cn bng lung ti nh x khng b ph v). Trong tr-ng hp 2b) nu ta tng
lung trn e2 ln mt i l-ng , th ta cn gim lung trn e1 cng mt i l-ng

lung i vo x vn bng lung i ra khi x. Tr-ng hp 2c) t-ng t nh- 2b) ngoi
tr vic ta tng lung trn e1 ln mt i l-ng v gim lung trn e2 i mt i l-ng
. Trong tr-ng hp 2d) chng ta gim lung trn c hai cung ni trn mt i l-ng

. Cc thay i trong mi tr-ng hp trn vn cho ta kt qu l mt lung trn mng.

Tt nhin, thc hin -c cc thay i ta cn c lung trn trn cung thun nh


hn kh nng thng qua ca cung v lung trn cung nghch phi ln hn 0.

V d 9.2.2. Xt -ng i t a ti z trn hnh 3a). Cc cung (a,b), (c,d), (d,z) l

cc cung thun, cn (c,b) l cung nghch. Chng ta tng lung trn cc cung thun ln 1,
gim lung trn cung nghch (c,b) i 1. Kt qu ta -c lung mi c gi tr ln hn
lung c l 1 n v nh- -c ch ra trn hnh 3b).

3,1
a
3,2

5,1

3,2
c

5,2

3,3

4,1
z
4,2

3a)

3b)

Hnh 3. Minh ha kh nng tng lung trn mng

T cc tho lun v cc v d trn ta c th chng minh nh l sau:

nh l 9.2.1. Gi s P l mt -ng i t a ti z v lung f cho tr-c trn th

G tha mn cc iu kin sau:

a) Vi mi cung thun (v,w) trn P ta c f (v, w) c(v, w);


b) Vi mi cung nghch (v,w) trn P ta c 0 f (v, w).

Gi l gi tr nh nht trong cc s -c nh ngha nh- sau:


c(v, w) f (v, w) i vi cc cung thun (v,w) trn P ;
f (v, w) i vi cc cung nghch (v,w) trn P .

Xy dng lung f* trn mng G theo qui tc sau:


f (u , v ) , nu (u,v)P l cung

f *(u , v ) f (u , v ) , thun
f (u , v ),

nu (u,v)P l cung

Khi , f* l lung trong mng G v val(f*)= val(f)+.

nh l 9.2.1 l c s xy dng thut ton lp sau y tm lung cc i


trong mng.
Thut ton 9.2.1. Thut ton Ford Fulkerson tm lung cc i trn mng.

Thut ton ny cn -c gi l thut ton gn nhn.

Input: Cho G = (V, E) l mt mng vi nh pht a, nh thu z, kh nng thng


qua ca cc cung k hiu l c(v,w) v cc nh a v0 , v1 , v2 ,..., vn z .
Output: Lung cc i f.

1. [Khi to lung] t f (u, v) : 0 vi mi cung (u,v) c trong mng.


2. [Gn nhn cho nh pht] Gn nhn (, ) cho nh a.

3. [nh thu z -c gn nhn?] Nu nh thu z -c gn nhn, th chuyn ti


b-c 6.

4. [Chn nh c nhn tip theo] Chn nh c nhn nh-ng ch-a -c xem xt


vi vi ch s i nh nht. Nu khng tn ti nh no nh- vy, th kt thc thut
ton (lung f thu -c l lung cc i); trong tr-ng hp ng-c li, t v : vi .

5. [Gn nhn cho cc nh lin k] Gi s ( , ) l nhn ca v. Kim tra tt c cc


cung c dng (v,u), (u,v) [theo th t (v, v0 ), (v0 , v), (v, v1 ), (v1 , v),... ], trong u l
nh ch-a -c gn nhn. Vi mi cung dng (v,u), nu f (v, u ) c (v, u ) ta gn

cho u nhn v , min , c (v, u ) f (v , u ) ; nu f (v, u ) c(v, u ) th u khng -c


gn nhn. Vi mi cung dng (u,v), nu f (v, u ) 0 ta gn cho u nhn
v, min , f (v, u ) ; nu f (v, u) 0 th u khng -c gn nhn. Chuyn ti

b-c 3.
6. [Xc nh li lung] Gi s ( , ) l nhn ca z. K hiu u0 z , u1 . Nu

nhn ca ui l ( ', ') th t ui 1 '. Tip tc qu trnh cho ti khi


uk a. Khi

P : a uk , uk 1 ,..., u1 , u0 z

l mt -ng i t a ti z. Thay i lung trn P theo quy tc sau: nu e l cung


thun trn P, ta tng lung ca e ln mt i l-ng ; nu e l cung ngch trn

P, ta gim lung ca e i mt gi tr . Xa nhn ca tt c cc nh v quay li


b-c 2.

Ta kt thc mc ny bi v d minh ha cho thut ton Ford - Fulkerson sau y.


V d 9.2.3. Tm lung cc i trn mng cho hnh 1.

Gii: Ta bt u t lung khng (hnh 4). Sau 3 b-c tng lung ta thu -c

lung cc i. Kt qu thc hin thut ton trn mng cho hnh 1 -c th hin trn
cc hnh 4-6.

3,
a

(a,3)

(, )

2,

(b,2)

4,

2,

(c,2)

5,0
(a,5)

(d,2

2,

4,

Hnh 4. Lung ban u v nhn cc nh

3,
a

2,

(a,1

(,

4,
2,

5,

(a,3

(e,2

2,

(d,2
e

4,

Hnh 5. Tng lung ln 2 v nhn cc nh

3,
a

2,

(a,1

(,

4,
2,

5,
d

(a,3

2,

4,

Hnh 6. Tng lung ln 3 v nhn cc nh

Bi tp.

1. Mt th c 100 nh, mi nh u c bc 50. Hy tnh s cnh ca n


2. th K3 c bao nhiu th con khng ng cu ?

Trong cc bi tp 3 - 5, cc cp th cho c ng cu hay khng?

3.

U1

U2

U3

U6

U5

4.

U4

U8

U7

u1

v1

5.

v4

v6

v5

v1

u1

v2

u2

u8
u7

u4
u5

v6

v5

u3
u6

v3

v7

u4

v8
v4

v4

v2

v8

u3

u5

v8

v1

u7
u6

v7

v5

v3

u2

u8

v6

v2

v7

v3

th y m - phn Kn1, n2,...nm c cc nh -c phn thnh m tp con trong


mi tp n 1 , n 2 , .... n m, phn t v cc nh ni vi nhau nu v ch nu chng thuc
cc tp con khc nhau .
6. V th sau:
a) K1,2,3

b) K2,2,2

c) K1,2,3,4

7. th y m - phn Kn1, n2,...nm c bao nhiu nh, bao nhiu cnh?


8. Hy chng minh hoc bc b rng trong mt n th hu hn c t nht hai

nh, lun tn ti hai nh cng bc.


Clic trong mt n th v h-ng l mt th con y khng nm trong
bt c th con y no rng hn n. Trong cc bi tp 9-11 hy tm tt c cc
clic trong cc th cho.

9.

e
11.

g
h

f
b

k
j
h

e
i

Tp tri ca cc nh trong mt n th l tp gm cc nh sao cho mi nh

khc u ni vi t nht mt trong cc nh ca tp ny. Tp nh tri vi s phn t


nh nht -c gi l tp tri ti thiu. Trong cc bi tp 12 - 14 hy tm tp tri ti
thiu cho mi th cho.

12
.

13.

a
c

e
c

d
f

14

e
i

15. Cho G1 v H1 l ng cu v G2 v H2 l ng cu. Hy chng minh hoc bc b


khng nh rng G1 G2 v H1 H2 l ng cu.

16. Ch ra rng mi mt trong cc tnh cht sau y l mt bt bin m cc th


n ng cu hoc l c hoc l khng c tnh cht ny.
a) Lin thng;

b) Tn ti chu trnh Hamilton

e) C n nh c lp ;

f) th phn i.

c) Tn ti chu trnh Euler ; d) C s t ct C

17. C bao nhiu n h ln thng, phn i khng ng cu v c 4 nh ?


18. C bao nhiu n th lin thng, khng ng cu vi 5 nh v
1. Khng c nh no bc ln hn 2?
2. C s mu bng 4?

3. L th khng phng ?

nh h-ng mt th v h-ng l vic gn mt chiu no cho cc cnh


ca n sao cho th c h-ng nhn -c l lin thng mnh. Nu c th nh

h-ng -c mt th v h-ng th th ny gi l th c th nh h-ng -c.


Trong cc bi tp t 19 - 21 hy xc nh xem cc th c th nh h-ng -c
khng ?

19.

20.

21.

a
d

th xoay l th c h-ng sao cho nu u v v l cc nh ca th th c

ng mt cnh (u, v) hoc (v, u) ni hai nh ny.

22. C bao nhiu th xoay khc nhau vi n nh ?

23. Tng cc bc vo v bc ra ca mt nh trong mt th xoay bng bao nhiu ?


24. Hy ch ra mi th xoay u c -ng i Hamilton .

25. Gi s G l mt a th lin thng vi 2k nh bc l. Chng t rng tn ti k

th con nhn G nh- l hp ca chng, trong mi th con c -ng i Euler


v khng c hai th con no c mt cnh chung.

(Gi : Php k cnh vo th ni cc cp nh bc l v dng chu trnh Euler cho


th ln hn ny).

26. Gi s G l n th vi n nh. Chiu rng ca G k hiu B (G) l s nh nht


ca max {i - j ai v aj l k nhau }, i vi tt c cc hon v a1 , a2 , ....an , ca
cc nh. iu ny c ngha l chiu rng l s nh nht i vi tt c cc danh sch
cc nh trong cc s ln nht ca hiu cc ch s ng vi cc nh k nhau.
Hy tm chiu rng ca cc th sau:
a) K5

b) K1,3

c) K2,3

d) K3,3

e) Q3

f) C5

27. Khong cch gia hai nh phn bit v1 v v2 ca mt n th lin thng l


di (s cc cnh) ca -ng i ngn nht gia v1 v v2 . Bn knh ca th l s nh
nht i vi tt c cc nh v1 ca khong cch cc i t v ti cc nh khc. -ng

knh ca th l khong cch cc i gia hai nh phn bit. Hy tm bn knh v


-ng ca cc th.
a) K6

b) K4,5

c) Q3

d) C6

28. a) Chng t rng nu -ng knh ca n th G t nht bng 4 khi -ng


knh ca phn b ca n khng ln hn 2.

b) Chng t rng nu -ng knh ca n th G t nht bng 3 khi -ng knh


ca phn b ca n khng ln hn 3.

29. Gi s mt a th c 2m nh bc l. Chng t rng mi chu trnh cha tt c


cc cnh ca th s cha t nht m cnh hn mt ln.

30. Tm -ng i ngn nht t a ti z trong cc th sau:.


a.

c.

2 3 2
4
2
1

3
4

z
4

b.

2
6

3
4

6
4
2
2
1

p
1

q
r

3
8

31. Tm di ca -ng i ngn nht gia cc cp nh sau y ca th trong bi


tp 30.
a) a v d
c) c v f

b) a v f

d) b v z

Tp cc nh trong mt th -c gi l c lp nu khng c hai nh no

trong tp ny -c ni vi nhau. S c lp ca mt th l s cc i cc nh
ca tp c lp ng vi th .

32. Tm s c lp ca
a) Kn

b) Cn

c) Qn

d) Km n

33. Ch ra rng s nh trong mt n th l nh hn hay bng tch ca s c lp


v s mu ca th .

Mt tnh cht no vn gi nguyn khi thm mt cnh vo mt n th

(khng thm nh) -c gi l n iu tng, v mt tch cht no vn gi


nguyn khi b mt cnh khi mt n th (khng xo nh) -c gi l n iu
gim .

34. Vi mi th c tnh cht sau hay xem xt n l n iu tng hay n iu gim .

a) th G lin thng

b) th G khng lin thng

c) th G c chu trnh Euler.

d) th G c chu trnh Hamilton


e) th G l phng

f) th G c s mu bng 4

g) th G c bn knh bng 3

h) th G c -ng knh bng 3

35. Chng minh rng tnh cht P ca th l n iu tng nu v ch nu tnh cht


Q ca th l n iu gim, trong Q l tnh cht ph nh ca P.
36. Hy xy dng th i ngu vi mi bn sau:

B
C D

A B C D

F
C

B
D

37. Tnh s mu ca cc th nu trong bi tp 36.

38. Tnh s mu ca th Wn .
39. Chng minh rng mt n th c chu trnh v c mt s l cc nh khng th

t bng hai mu.

40. Hy lp lch thi cc mn Gii tch, i s, Tin hc, Ngoi ng, Vt l vi s t

nht cc t thi, nu khng c sinh vin no thi c hai mn Gii tch v Tin hc, Gii tch v
i s, Ngoi ng v Tin hc, Gii tch v Vt l, nh-ng c sinh vin thi trong mi t hp
khc ca cc mn.

41. Su i truyn hnh cch nhau nh- cho trong bng d-i y. Hi phi

cn bao nhiu knh khc nhau pht sng, nu hai i khng th dng cng mt
knh khi chng cch nhau khng qu 150 dm?

1
2

85

100

175

200

175

100

160

250

50

200

-125

175

100

125
100

175

85

3
4

200

50

100

200

250

100

160

210

220

220

100

210

160

100

T mu cnh th l gn cc mu cho cc cnh sao cho cc cnh lin thuc

vi nhau mt nh chung -c gn mu khc nhau. S mu cnh ca mt th l


s nh nht cc mu cn dng t mu cc cnh ca n.
42. Hy tm s mu cnh ca cc th.

a) Kn

b) Km,n

c) Cn

d) W n

43. By bin xut hin trong vng lp ca mt ch-ng trnh. Cc bin v cc b-c

trong chng cn phi l-u l: t: cc b-c t 1 ti 6; u: b-c 2; v: cc b-c t 2 ti 4; w:

cc b-c 1,3 v 5; x: cc b-c 1 v 6; y: cc b-c t 3 ti 6; v z: cc b-c 4 v 5. Cn bao


nhiu thanh ghi ch s khc nhau l-u cc bin trn khi thc hin vng lp ny.

Trong cc bi tp 44-46 hy xy dng h a ch ph dng cho cc cy c

gc -c sp cho. Sau dng chng sp xp cc nh ca cy theo th t t


in ca cc nhn.

44.

45.

46.

47. Gi s a ch ca nh v trong cy T l 3.4.5.2.4.


a) v mc no ?
b) Tm a ch ca cha ca v.

c) v c ti thiu my anh em ?
d) T c t nht bao nhiu nh ?

e) Hy tm cc a ch khc phi c.
48. Gi s nh c a ch ln nht trong cy T c a ch 2.3.4.3.1. C th xc nh
-c s nh trong T hay khng ?
49. Cc l ca cy c gc v -c sp c th c danh sch a ch ph dng nh- sau

-c khng ? Nu c, hy xy dng cy c gc .
a). 1.1.1, 1.1.2., 1.2, 2.1.1.1., 2.1.2, 2.1.3, 2.2, 3.1.1, 3.1.2.1, 3.1.2.2,
3.2

b) 1.1, 1.2.1, 1.2.2, 1.2.3, 2.1, 2.2.1, 2.3.2, 2.4.2.1, 2.4.2.2, 3.1, 3.2.1, 3.2.2

c) 1.1, 1.2.1, 1.2.2, 1.2.2.1, 1.3, 1.4, 2, 3.1, 3.2, 4.1.1.1


Trong cc bi tp t 50 -52 hy xc nh th t m cc nh ca cy c gc

-c ving thm nu ta duyt n theo kin tin th t.


50.

51.

c
e

52.

c
e

d
g

f g
j

m n o

e
l

k
p

c
f

l m
r

g
n

h
o
s

d
j

i
p

53. Xc nh th t m cc nh ca cy c gc trong bi tp 50-52 -c


ving thm nu ta duyt n theo kiu trung th t.
54. Xc nh th t m cc nh ca cy c gc trong bi tp 50-52 -c
ving thm nu ta duyt n theo kiu hu th t.
55. Hy xy dng cy c gc -c sp sao cho khi duyt n theo kiu tin th
t ta -c a, b, f, c, g, h, i, d, e, j, k, l trong a c 4 con, c c 3 con, j c 2 coan, b

v c u c 1 con cn tt c cc nh khc u l l.
56. Hy chng t rng cy c gc -c sp li xc nh duy nht khi cho
danh sch cc nh ca n sinh ra bng cch duyt tin th t v s con ca mi
nh l -c cho tr-c.

57. Hy chng t rng cy c gc -c sp l xc nh duy nht khi cho danh


sch cc nh ca n sinh ra bng cch duyt hu th t v s con ca mi nh l
-c cho tr-c .
58. Dng thut ton sp xp kiu ni bt hy sp xp danh sch 3, 1, 5, 7, 4.
Hy ch r cc danh sch nhn -c mi b-c.
59. Dng thut ton sp xp kiu ni bt hy sp xp danh sch d, f, m, k, a,
b. Hy ch r cc danh sch nhn -c mi b-c.
60. Dng thut ton sp xp kiu ho nhp hy sp xp danh sch 4, 3, 2, 5,
1, 8, 7, 6. Hy ch r tt c cc b-c -c s dng trong thut ton.
61. Dng thut ton sp xp kiu ho nhp hy sp xp danh sch b, d, a, f,
g, h, z, p, o, k. Hy ch r tt c cc b-c -c s dng trong thut ton.
62. Cn bao nhiu php so snh ho nhp cc danh sch sau y bng
thut ton 2?
1, 3, 5, 7, 9 ; 2, 4, 6, 8, 10

1, 2, 3, 4, 5, ; 6, 7, 8, 9, 10
5. 6. 7. 8; 2. 3. 4. 9. 10
Sp xp kiu chn lc bt u bng vic tm phn t nh nht trong danh

sch. Phn t ny -c chuyn ln u danh sch. Sau ta li tm phn t nh

nht trong cc phn t cn li ri t n v tr th hai. Th tc ny -c lp li


cho ti khi ton b danh sch -c sp xp .
63. Sp xp danh sch sau y bng thut ton sp xp kiu chn lc:
a) 3, 5, 4, 1, 2
b) 5, 4, 3, 2, 1
c) 1, 2, 3, 4, 5.
Sp xp nhanh l mt thut ton c hiu qu tt. sp xp a 1 , a 2 ... a n

thut ton ny bt u bng vic ly phn t u tin a 1 v to hai danh sch con,
danh sch u cha cc phn t nh hn a 1 theo th t xut hin ca chng, danh

sch th hai cha cc phn t ln hn a 1 theo th t ca chng. Khi a 1 -c t


cui ca danh sch u. Th tc ny -c lp li mt cch quy cho mi danh
sch con cho ti khi mi danh sch cha ch mt phn t theo th t xut hin ca
chng.

64. Sp xp danh sch 3, 5, 7, 8, 1, 9, 2, 4, 6 bng thut ton sp xp nhanh.


Trong cc bi tp 65 - 67 dng k thut tm kim theo chiu rng v chiu

su hy xc nh cy khung cho cc th n cho. Chn a lm gc ca cy v


gi s rng cc nh -c sp theo th t t in .

65.

h
g

67.

j
g

66.

Trong cc bi tp 68-70 hy dng cc thut ton Prim v Kruskal tm cy


khung nh nht ca cc th cho.
68.

69.

a
4
c

2
3

1
e
1

3
2

b
3

a
2

d
6

b
5

7
8

6
e

3
4

4
3
1
4

4
2

70.
2
a

d
2

1
1

c 1

e
2
f 3 g 2
2
3
4
h
3 i
3
j 4 k 3
3
4
3
l
2 m 2 n 2 o 3
p
2

71. Hy tm mt n th lin thng c trng s vi mt s ti thiu cc cnh sao cho


n c nhiu hn mt cy khung nh nht.

Cy khung cc i ca mt th v h-ng, lin thng, c trng s l cy khung c


trng s ln nht.

72. Hy xut mt thut ton t-ng t thut ton Prim xy dng cy khung cc i ca
mt th lin thng c trng s.

73. Hy xut mt thut ton t-ng t thut ton Kruskal xy dng cy khung cc i
ca mt th lin thng c trng s.

74. Hy tm cy khung cc i cho th c trng s trong 68-70.

75. S dng lung tm -c trong v d 9.2.3, tm lung cc i ca mng trong


tr-ng hp kh nng thng qua ca cung (d,e) l 4.
76. Tm lung cc i trong cc mng sau :

a.

b
5

3, 1
5, 2

6
2

4, 2

4, 3

e
g

4, 2
4

b.
b
8

10
11

a
9

2
4

12
9

12
10

10
10

14

8
6

12

14

6
6

10
(1

10
(1

12
10

Ch-ng IV
i s Boole v mch t hp
Trong ch-ng ny chng ta s cp mt s khi nim c bn ca mch t hp

v i s Boole, ng thi ch ra mi lin h mt thit gia cc hm Boole v cc mch


t hp. Chng ta s ch ra cch dng cc hm Boole phn tch v thit k cc mch

thc hin cc nhim v khc nhau. Mt trong cc ph-ng php ti -u ho biu thc
Boole - ph-ng php Quine-McClusky cng s -c gii thiu ch-ng ny. Cn phi
nhn mnh rng th tc ti -u ho biu thc Boole ng vai tr ht sc quan trng
thit k cc mch t hp t hiu qu cao trong thc t.
1. Khi nim v mch t hp

1.1. Khi nim


Cc mch in trong cc thit b in t ni chung v trong my tnh in t ni

ring u c hai trng thi khc nhau ca cc u vo cc u ra -c k hiu l 0 v 1


(bit 0 v bit 1). Hai trng thi c th to ra bng cc chuyn mch c th hai v tr

m hoc ng. Chng cng c th to ra bng cc dng c quang hc c th sng hoc


ti. Nm 1938 C.E.Shannon chng t rng c th dng cc quy tc c bn ca logic
do G.Boole -a ra trong cun sch mang tn Cc quy lut ca t- duy c s ca i s
Boole - thit k cc mch in. Cc mch m chng ta xt y c u ra ch ph

thuc vo cc t hp u vo m khng ph thuc vo trng thi hin thi ca mch.


Ni mt cch khc, cc mch ny khng c kh nng nh, chng -c gi l cc mch
t hp.
Cc mch t hp c th -c xy dng trn c s s dng ba loi phn t/mch

c bn, cn -c gi l cc cng logic. l cng AND (cng nhn), cng OR (cng


cng) v cng NOT (b o).
nh ngha 1.1.1. Cng AND nhn cc gi tr vo l cc bit x1, x2 l cc bit v

cho u ra l mt bit -c k hiu l x1 x2 (tch Boole ca x1 v x2, i khi ta cng


vit l x1.x2 hoc n gin l x1x2) c gi tr -c nh ngha nh- sau:
1, nu x1 1 v x2 1
x1 x2
0, trong cc tr-ng hp cn li.

nh ngha 1.1.2. Cng OR nhn cc gi tr vo l cc bit x1, x2 l cc bit v cho

u ra l mt bit -c k hiu l x1 x2 (tng Boole ca x1 v x2, i khi ta cng vit


l x1+x2) c gi tr -c nh ngha nh- sau:

1, nu x1 1 hoc x2 1
x1 x2
0, trong cc tr-ng hp cn li.

nh ngha 1.1.3. Cng NOT (b o) nhn gi tr vo l bit x v cho u ra l


mt bit k hiu l x (ph nh/phn b Boole ca x) c gi tr -c nh ngha nh- sau:
0 nu x 1
x
1 neu x 0.

Biu din ca cc cng AND, OR, NOT -c cho trong hnh 1.


x1
x2

x1 x2

x1 x2

x1
x2

Hnh 1. Cc phn t lgic c bn


Ng-i ta c th dng cc bng logic lit k tt c cc gi tr u vo cng cc
u ra t-ng ng c th c ca mch t hp.
NOT:

V d 1.1.1. Cc bng d-i y l cc bng logic ca cc mch c s AND, OR,

x1 x2 x1x2 x1 x2
1

Cc mch t hp phc tp c th -c xy dng bng cch dng t hp cc b


o, cc cng AND v OR. Khi lp t hp cc mch, mt s cng c th dng chung

u vo. iu ny c th thy mt trong hai cch v mch d-i y. Mt cch dng


cc phn nhnh ch tt c cc cng dng chung mt u vo cho. Cn cch th hai

ch u vo ny mt cch ring bit i vi mi cng. Cng cn l-u rng u ra ca


mt cng c th -c dng lm u vo i vi mt hoc nhiu phn t mch khc.
V d 1.1.2. Hnh 2 minh ha hai cch v mch cho cng u ra l z c th -c

vit nh- sau:

z ( x y ) ( x y) .

x
y

xy
x

x
y

xy

x
y

xy

x
y

Hnh 2. Hai cch v cng mt mch t hp

Biu thc kiu nh- biu thc trn -c gi l biu thc Boole m ta s nh
ngha d-i y.
1.2. Biu thc Boole
Mt bin x -c gi l mt bin Boole nu n ch nhn mt trong hai gi tr
lgic k hiu l 0 hoc 1. Biu thc Boole i vi cc bin Boole x1 , x2 ,..., xn -c nh
ngha mt cch quy nh- sau.

nh ngha 1.2.1. Cc k hiu 0,1 v cc bin Boole x1 , x2 ,..., xn l cc biu thc

Boole. Nu X 1 , X 2 l cc biu thc Boole no , th

a) ( X 1 )

b) X 1 X 1

b) X 1 X 2

c) X 1 X 2

cng l cc biu thc Boole.

Nu X l biu thc Boole vi cc bin x1 , x2 ,..., xn th ta cng s vit l


X ( x1 , x2 ,..., xn ) . Trong cc biu thc Boole, nu khng c cc du ngoc xc nh trnh

t cc php ton, ta lun quy -c -c thc hin tr-c .

V d 1.1.2 ch ra cho ta cch biu din mch t hp vi mt u ra bng biu


thc Boole. V d d-i y s cho ta bit cch thit k mt mch t hp t-ng ng vi
mt biu thc Boole cho tr-c.
V d 1.2.1. Tm mch t hp t-ng ng vi biu thc Boole sau y:
( x1 ( x 2 x 3 )) x 2

v vit bng logic t-ng ng vi mch thu -c.


Chng ta bt u bng biu thc x 2 x 3 nm trong ngoc trong cng ca biu

thc. Biu thc ny t-ng ng vi mch t hp trn hnh 3. u ra ca mch ny cng

vi x1 qua cng AND cho ta mch t hp hnh 4. Cui cng u ra ca mch t hp


trn hnh 4 cng vi x2 qua cng OR cho chng ta mch t hp cn tm trn hnh 5.
x2

x2
x2

x3

x3

Hnh 3.

x1
x2
x3

x2
x2

x3

Hnh 4.

x1 (x2 x3 )

x1
x2

x1 (x2 x3 )

x2
x2

x3

x3

(x1 (x2 x3 )) x2

Hnh 5.
Bng logic ca mch t hp l:

x1 x2 x3 ( x1 ( x 2 x 3 )) x 2
1

2. Cc tnh cht ca mch t hp


2.1. Cc tnh cht
Trong mc tr-c chng ta nh ngha cc php ton
Gi s x, y, z l cc bin Boole, khi ta c cc tnh cht sau:

trn cc bin Boole.

a) Lut kt hp: ( x y ) z x ( y z ); ( x y ) z x ( y z )
b) Lut giao hon: x y y x; x y y x

c) Lut phn phi: x ( y z ) ( x y ) ( x z ); x ( y z ) ( x y ) ( x z )

d) Lut ng nht: a 0 a; a 1 a
e) Lut b: x x 1; x x 0
f) Lut lu ng: x x x; x x x
g) Lut De Morgan: ( x y ) x y ; ( x y ) x y
C th kim tra trc tip cc tnh cht trn bng cch s dng bng logic so

snh gi tr ca hai v ng thc t-ng ng. D-i y l mt minh ho dng bng logic
chng minh tnh cht c):

x y z

x ( y z)

( x y) ( x z)

1 1 1

1 1 0

1 0 1

1 0 0

0 1 1

0 1 0

0 0 1

0 0 0

Nhn xt 2.1.1. Ta c th thy rng cc tnh cht a) - g) u xut hin theo

tng cp. Mi quan h gia hai ng thc trong mi cp lin quan n mt khi nim
-c gi l i ngu. i ngu ca mt biu thc Boole nhn -c bng cch i ch

cc php ton v cng nh- cc s 0 v 1 c mt trong biu thc y cho nhau. V


d, i ngu ca cc biu thc x ( y 0) v ( x 1) ( y 0 z ) t-ng ng l
x ( y 1) v ( x 0) ( y 1 z ) . C th chng minh -c rng, mt hng ng thc

gia cc biu thc Boole vn cn ng nu ta ly i ngu c hai v ca n (xem bi tp


11 cui ch-ng). Kt qu ny -c gi l nguyn l i ngu - th-ng -c dng
nhn -c cc ng thc mi.

2.2. Mch t hp t-ng -ng


Hai biu thc Boole trn cng cc bin Boole x1, x2,..., xn -c gi l bng
nhau nu gi tr ca chng trn mi b gi tr c th c ca cc bin l nh- nhau. Nhvy, cc tnh cht nu trong mc trn thc cht l cc hng ng thc Boole. Cc mch
t hp t-ng ng vi cc biu thc Boole bng nhau -c gi l cc mch t hp t-ng

-ng. Ni cch khc, hai mch t hp -c gi l t-ng -ng nu hai mch vi


cc u vo nh- nhau s cho u ra cng nh- nhau.
V d 2.2.1. Mch t hp hnh 1a) c biu thc Boole t-ng ng l a b , cn
mch t hp hnh 1b) c biu thc Boole t-ng ng l a b . Hai biu thc Boole
bng nhau, do vy hai mch t hp hnh 1 l cc mch t hp t-ng -ng.

a
b

a)

y1

a
y1
b

b)

Hnh 1. Hai mch t hp t-ng -ng

V d 2.2.2. C th kim tra cc biu thc Boole sau bng nhau


x1 ( x 2 x3 ); (x1 x2 ) x3 .

Do vy cc mch t hp t-ng ng vi chng trn hnh 2 l cc mch t hp


t-ng -ng.

x1

y1

x2
x3
x1

x2

y1

x3

Hnh 2. Hai mch t hp t-ng -ng

V d 2.2.3. C th thy rng x y x y , do vy cng OR c th -c biu

din qua cc cng AND v NOT nh- trn hnh 3.


x
y

xy

Hnh 3. Biu din cng OR qua cc cng AND v NOT

Nhn xt 2.2.1. V d 2.2.3 cho thy ch cn hai cng AND v NOT ta c th

biu din mt mch t hp bt k, hn th na, c th chng minh rng khng th ch


dng mt cng AND hoc mt cng NOT biu din cc mch t hp. iu ny lin
quan n mt khi nim quan trng l tnh y ca mt h cng. Cc bi tp 1213 cui ch-ng s ch ra cho chng ta mt vi h cng y khc.
3. Hm Boole v vn tng hp mch
3.1. i s Boole

Cc tnh cht m chng ta nu trn i vi cc biu thc Boole cng c th


chuyn thnh cc kt qu t-ng ng cho cc mnh hoc cc tp hp. V th s rt c
ch, nu chng ta nh ngha i s Boole mt cch tng qut. Khi y, nu mt cu trc
no -c xc nh l mt i s Boole, th mi kt qu -c thit lp cho i s
Boole tng qut cng s -c p dng cho cu trc c bit y.
Trong mc ny chng ta s nh ngha i s Boole bng cch ch ra nhng tnh
cht m cc php ton cn phi tho mn.

nh ngha 3.1.1. i s Boole l mt tp B c hai phn t khc nhau k hiu l 0 v

1 vi hai php ton hai ngi +, . v mt php ton mt ngi trn B sao cho vi mi
x,y,z thuc B ta lun c:

a) Lut giao hon: x+y=y+x; x.y=y.x


b) Lut kt hp: (x+y)+z=x+(y+z); (x.y).z=x.(y.z)
c) Lut phn phi: x.(y+z)=(x.y)+(x.z); x+(y.z)=(x+y).(x+z)
d) Lut ng nht: x+0=x; x.1=x
e) Lut b: x+x =1; x.x =0
T nh ngha trn ta thy tp B={0,1} vi cc php ton AND, OR v NOT; tp
hp cc mnh n bin vi cc php ton logic ; tp tt c cc tp con ca tp v
tr vi cc php ton hp, giao v ton t ly phn b u l cc i s Boole.

Dng cc lut -c nu trong nh ngha 3.1.1 ta c th chng minh -c nhiu

tnh cht khc cng ng vi mi i s Boole.

nh l 3.1.1. Phn t x trong nh ngha 3.1.1 ca mt i s Boole bt k l duy

nht. C th, nu x+y=1 v x.y=0 th y=x.


Chng minh: Tht vy, ta c
y = y.1

[Theo nh ngha 3.1.1 d)]

= y(x+x)

[Theo nh ngha 3.1.1 e)]

= y.x+y.x

[Theo nh ngha 3.1.1 c)]

= x.y+y.x

[Theo nh ngha 3.1.1 b)]

= 0 + y.x

[Theo gi thit]

= x.x +y.x

[Theo nh ngha 3.1.1 e)]

= x.x +x.y

[Theo nh ngha 3.1.1 b)]

= x.(x+y)

[Theo nh ngha 3.1.1 c)]

= x.1

[Theo gi thit]

= x

[Theo nh ngha 3.1.1 d)].

Do nh l trn, ta c th nh ngha:

nh ngha 3.1.2. Phn t x trong mt i s Boole bt k -c gi l phn t b

ca x.

T-ng t nh- nh l 3.1.1 ta c th chng minh nh l sau:

nh l 3.1.2. Gi s B=(S, +,., , 0, 1) l mt i s Boole. Khi x, y S ta

lun c:

f) x+x=x; x.x=x, x S (lut lu ng)

g) (x+y) = x.y; (x.y)=x+y, x, y S (lut De Morgan)


h) x + 1 = x; x.0=0

i) x + x.y = x; x(x+y)= x, x, y S
j) (x) =x, x S
k) 1 =0; 0 =1.
3.2. Hm Boole v vn tng hp mch
Gi s X(x1,x2,...,xn) l mt biu thc Boole. Khi hm s f(x1,x2,...,xn) =
X(x1,x2,...,xn) -c gi l hm s Boole. K hiu B={0,1}, ta c hm Boole
f(x1,x2,...,xn) l mt nh x t Bn vo B. Ng-c li, mi hm n bin f(x1,x2,...,xn) t Bn
vo B bt k u c th biu din d-i dng mt biu thc Boole. Khi thit k mt mch

th-ng ng-i ta cho bit tr-c u ra ca mch t-ng ng vi cc b gi tr u vo.


Chnh v vy, bi ton xc nh biu thc t-ng ng biu din hm Boole trn c s cho
tr-c cc gi tr ca hm ng vai tr rt quan trng trong vic thit k cc mch. D-i
y, thng qua mt v d, chng ta s xem xt mt ph-ng php tm biu thc Boole
biu din mt hm Boole.
V d 3.2.1. Cho hm Boole c gi tr -c cho trong bng. Hy xc nh biu

thc Boole t-ng ng ca hm .

x1 x2 x3 f(x1,x2,x3)
1

Gii: Xt dng th nht ca bng v t hp

x1 x 2 x3 (1)

Ta c nhn xt l nu x1 x 2 x 3 1 nh- trong dng th nht ca bng th gi

tr ca biu thc (1) s bng 1. Cc gi tr ca xi -c cho trong bt k dng no khc


u lm cho (1) bng 0. T-ng t, i vi dng th t- ca bng ta xy dng t hp
x1 x 2 x 3 (2).

Gi tr ca biu thc (2) cho gi tr 1 ch khi cc xi c gi tr cho dng th t-

ca bng, cn vi cc gi tr ca cc xi cho bi cc dng khc ca bng u lm cho gi


tr biu thc (2) bng 0.
Th tc xy dng cc biu thc (1), (2) y kh r rng. Xt dng R bt k ca
bng c gi tr u ra bng 1. Xy dng t hp x1 x 2 x3 v thay th mi xi trong t

hp bng x i nu gi tr ca bin xi trong dng R bng 0. T hp thu -c s nhn gi


tr 1 khi v ch khi xi nhn cc gi tr t-ng ng cho trong dng R ca bng. Nh- vy,
vi dng th 6 ca bng ta nhn -c t hp:
x 1 x 2 x 3 (3)

Tip theo, ly tng OR ca (1)-(3) ta nhn -c biu thc Boole:


( x1 x 2 x 3 ) ( x1 x 2 x 3 ) ( x 1 x 2 x 3 ) (4)

D dng kim tra li -c l biu thc trn ch nhn gi tr bng 1 khi v ch khi

x1,x2,x3 nhn cc gi tr trn cc dng ca bng t-ng ng vi gi tr f(x1,x2,x3) = 1,


cn khi x1,x2,x3 nhn cc gi tr t-ng ng vi nhng dng f(x1,x2,x3) = 1 th biu
thc (4) cng nhn gi tr 0. Do vy (4) chnh l biu thc Boole t-ng ng ca hm f
cn tm.
f ( x1 , x 2 , x3 ) ( x1 x 2 x 3 ) ( x1 x 2 x 3 ) ( x 1 x 2 x 3 ) .

V d 3.2.1 minh ho mt ph-ng php xy dng biu thc Boole biu din mt

hm Boole c cc gi tr cho. Mi mt t hp gi tr ca cc bin lm cho hm c gi


tr 1 s dn ti mt tch Boole ca cc bin hoc cc phn b ca chng.

nh ngha 3.2.1. Mt bin Boole hoc phn b ca n -c gi l mt tc bin.


Tch Boole y1 y 2 ... y n trong y i x i hoc y i x i vi x1, x2,...,xn l cc bin
Boole -c gi l mt tiu hng (minterm). Do , tiu hng l tch ca n tc bin.

Mt tiu hng c gi tr 1 i vi mt v ch mt t hp gi tr ca cc bin ca


n. Chnh xc hn, tiu hng y1 y 2 ... y n bng 1 khi v ch khi mi yi=1 v iu
ny ch xy ra khi v ch khi xi 1 nu y i x i v xi 0 khi y i x i .

Bng cch ly tng Boole ca cc tiu hng phn bit chng ta c th lp -c


biu thc Boole vi tp cc gi tr -c cho tr-c. c bit, tng Boole ca cc tiu
hng c gi tr 1 khi v ch khi mt trong cc tiu hng c gi tr bng 1. Tiu hng
c gi tr 0 i vi mi t hp gi tr cn li ca cc bin. Do , vi mt hm Boole
cho ta c th lp mt tng Boole cc tiu hng c gi tr 1 khi hm c gi tr 1 v c
gi tr 0 khi hm c gi tr 0. Cc tiu hng ca tng Boole ny t-ng ng vi cc t

hp gi tr lm cho hm c gi tr 1. Tng Boole cc tiu hng biu din hm -c gi l


khai trin tng cc tch Boole hay dng tuyn chun tc (dng chun tuyn) ca hm
Boole.
V d 3.2.2. Gi s cn phi tm dng chun tuyn ca mt hm -c cho bi

biu thc Boole sau:

f ( x1 , x 2 , x 3 ) ( x1 x 2 ) x3
Chng ta c th xy dng bng logic ca f ri sau bng ph-ng php trnh

by trn xy dng dng tuyn chun tc t-ng ng. Tuy nhin, ta cng c th s
dng cc tnh cht ca cc php ton AND, OR, NOT v nh ngha dng tuyn chun
tc gii bi ton t ra. Tht vy, tr-c tin, ta p dng lut phn phi -a biu
thc ca f v dng sau:

( x1 x 2 ) x 3 ( x1 x3 ) ( x2 x3 )
Mc d biu thc thu -c l tng ca cc tch Boole, song y ch-a phi l

dng tuyn chun tc, v mi tiu hng c mt trong biu thc trn khng cha tt c cc
bin x1,x2,x3. Tuy nhin, ta c th bin i biu thc nh- sau:

( x1 x3 ) ( x2 x3 ) ( x1 x3 1) ( x2 x3 1)
( x1 x3 ( x2 x 2 )) ( x2 x3 ( x1 x1 ))

( x1 x2 x3 ) ( x1 x 2 x3 ) ( x1 x2 x3 ) ( x1 x2 x3 )
( x1 x2 x3 ) ( x1 x 2 x3 ) ( x1 x2 x3 )

Biu thc cui cng chnh l dng tuyn chun tc ca hm f cho.

Nhn xt 3.2.1. S dng nguyn l i ngu (xem nhn xt 2.1.1) ta cng c th

tm -c biu din ca mt hm Boole bt k d-i dng tch ca cc tng Boole. Biu


din y -c gi khai trin tch cc tng Boole hay l dng hi chun tc (dng chun
hi) ca hm Boole. Mt ph-ng php tm dng hi chun tc ca mt hm Boole -c
cho trong bi tp 25 cui ch-ng ny.
4. Mt vi ng dng
4.1.

B cng

Trong phn ny chng ta s thit k cc mch logic thc hin php cng hai s
nguyn d-ng t cc khai trin nh phn ca chng. S mch s -c xy dng t
mt s mch thnh phn. dn gin trong cch vit, chng ta s thay cc k hiu ,
bng cc du . v +.

Tr-c ht chng ta xy dng mt mch -c dng tm gi tr x+y vi x, y l cc

bit. u vo mch ny s l x v y v gi tr ca chng u bng 0 hoc 1. u ra s


gm 2 bit s v c, trong s l bit tng v c l bit nh. Mch ny -c gi l mch nhiu
u ra v n c ln hn mt u ra (cc hm v cc biu thc Boole m chng ta xt
phn trn t-ng ng vi cc mch ch c mt u ra). Mch m chng ta ang thit k
-c gi l b na cng, v n cng hai bit m khng xt n s nh t php cng tr-c.
Bng cc gi tr u vo v u ra ca b na cng c dng nh- sau:
u vo v u ra ca b na cng
u vo

u ra

T bng trn ta thy c x. y , s x. y x. y ( x y ).( x. y ) . Do mch cho trn

hnh 1 s tnh tng s v bit nh c t cc bit x v y.

x
y

Hnh 1. B na cng

By gi chng ta s xy dng b cng y tnh tng v bit nh khi hai bit


-c cng cng vi s nh. u vo i vi b cng y l cc bit x, y v s nh ci,
u ra l bit tng s v bit nh mi ci+1.
u vo v u ra ca b cng y
u vo

u ra

ci

ci+1

Hai u ra ca b cng y , tc l bit tng s v bit nh ci+1 -c biu din

bi cc dng tuyn chun tc nh- sau:

s x. y.ci x. y.c i x. y.c i x. y.ci

ci 1 x. y.ci x. y.c i x. y.ci x. y.ci

Dng cc cng AND, OR, NOT t-ng ng ta c th thit k -c b cng y .

Tuy nhin, thay v thit k b cng y t cc phn t c bn, ta s dng cc b na cng


to cc u ra mong mun nh- -c ch ra trong hnh v d-i y.

x
y

z
c

Hnh 2. B cng y

4.2.

Cc tiu ho cc mch. Ph-ng php Quine-McCluskey

4.2.1. Cc tiu ho cc mch. Trong cc mc trn ta thy cng mt hm s c th

c nhiu cng thc biu din khc nhau. T-ng t nh- vy, mt mch t hp c th c
nhiu thit k vi cc phn t khc nhau. Hiu qu ca mt mch t hp ph thuc rt
nhiu vo s l-ng cc cng v s b
tr cc cng . Qu trnh thit k
mt mch -c bt u bng vic xy
dng bng logic ch r cc gi tr u

x
y
z

ra t-ng ng vi mi mt t hp u x
vo. S dng cc bng logic d, ta
lun c th dng khai trin tng cc
tch ca mch (dng tuyn chun tc)
tm ra tp cc cng logic thc hin
mch . Tuy nhin, khai trin y c

xyz
xyz+xy z

x y z

z
x

Hnh 1.
xz

th cha cc s hng nhiu hn mc z


cn thit v bng cc bin i t-ng -ng chng ta c th rt gn dng tuyn chun tc
ca mch. V d, mt mch t hp c dng tuyn chun tc l x. y. z x. y.z c th rt
gn nh- sau:

x. y. z x. y. z x. y.( z z ) x. y.1 x. y

Biu thc cui cng cng l mt biu din ca mch cho nh-ng vi s l-ng
php ton t hn. Hai thc hin khc nhau ca mch -c cho trn hnh 1.

R rng l mch th hai n gin hn mch th nht, n ch dng mt cng,


trong khi mch th nht phi dng ti ba cng v mt b o.
V d nu trn cho thy cn thit phi tm cch biu din mch bng biu thc

n gin nht c th -c. y cng chnh l ni dung c bn ca bi ton cc tiu ho


mch t hp. Mc ch ca cc ph-ng php cc tiu ho mch t hp l to tng Boole
cc tch Boole cha mt s nh nht tch cc tc bin sao cho cc tch ny li cha mt
s t nht cc tc bin trong s tt c nhng tng cc tch Boole biu din cng mt
mch. D-i y ta s xt mt trong s cc ph-ng php y ph-ng php QuineMcCluskey do W.V.Quine v E.J.McCluskey pht trin vo nhng nm 1950.

4.2.2. Ph-ng php Quine-McCluskey. Ph-ng php ny gm hai th tc chnh.

Th tc th nht xc nh cc s hng l ng vin -a vo khai trin cc tiu nh- mt


tng cc tch Boole. Th tc th hai xc nh trong s cc ng vin , cc s hng no
l thc s dng -c. Chng ta s xem xt ph-ng php ny thng qua mt v d minh
ho.

C th, chng ta s dng ph-ng php Quine-McCluskey tm biu thc cc


tiu t-ng -ng vi tng cc tch Boole sau:
x. y. z x. y. z x. y.z x. y.z x. y.z

Ta s biu din cc tiu hng trong tng trn bng cc xu bit. Bit u tin s l
1 nu xut hin x v l 0 nu xut hin x . T-ng t, bit th hai (bit th 3) s l 1 nu
xut hin y (z) v l 0 nu xut hin y(z ) . Sau chng ta nhm cc s hng theo s
l-ng cc tr 1 trong cc xu bit t-ng ng. Thng tin ny -c ghi trong bng 1.
Bng 1
Tiu hng

Xu bit

S cc s 1

x.y.z

111

x. y. z

101

x. y. z

011

x. y. z

001

x. y. z

000

Cc tiu hng c th -c t hp li l nhng s hng ch khc nhau mt tc


bin. Do , hai s hng c th t hp -c s ch khc nhau mt con s 1 trong cc xu
bit biu din cc s hng . Khi hai tiu hng -c t hp thnh mt tch, tch ny s
cha hai tc bin. Tch c hai tc bin-c biu din bng mt du gch ngang ch
bin khng xut hin. V d, tiu hng x. y.z v x. y.z -c biu din bng cc xu bit
101 v 001 c th -c t hp thnh y.z -c biu din bng xu 01. Tt c cc cp
tiu hng t hp -c v tch to thnh t cc t hp -c cho trong bng 2. Tip
theo, tt c cc cp tch c hai tc bin c th t hp -c s -c t hp thnh s hng
c mt tc bin. Hai tch nh- vy c th t hp -c nu chng cha tc bin ca cng

hai bin. Cc tc bin ny ch khc nhau i vi mt trong hai bin . Ni theo ngn
ng cc xu biu din cc tch , th hai xu phi c gch ngang cng mt v tr v
ch khc nhau mt trong hai v tr cn li. V d, chng ta c th t hp cc tch
y.z , y.z -c biu din bi cc xu 11, -01 thnh z vi biu din t-ng ng bi xu 1.
Tt c nhng t hp c th -c to theo cch -c cho trong bng 2.
Bng 2
B-c 1
S hng

bit

Xu

hng

B-c 2
S

bit

Xu

x. y. z

111

(1,2)

x. z

1-1

x. y. z

101

(1,3)

y.z

-11

x. y. z

011

(2,4)

y.z

-01

x. y. z

001

(3,4)

x.z

0-1

x. y. z

000

(4,5)

x. y

00-

hng
(1,2,3,4)

S
z

Trong bng 2 cng ch ra nhng s hng -c dng to ra cc tch c s

tc bin nh hn song khng nht thit phi c mt trong biu thc cc tiu. Th tc
tip theo ca ph-ng php l nhn dng tp cc tiu cc tch cn thit c mt trong biu
din cn tm. Cng vic ny -c bt u vi tt c cc tch ch-a -c dng xy
dng cc tch c s tc bin t hn (trong v d ang xt l z , x. y ). Tip theo, ta lp
bng 3, trong c mt dng dnh cho mi ng vin -c to ra bng cch t hp

bit

Xu
--1

cc s hng gc (ban u) v mt ct dnh cho mi s hng gc. Ta nh du x


t-ng ng trong bng, nu s hng gc trong khai trin tng cc tch -c dng
to tch ng vin . Trong tr-ng hp ny ta ni tch ng vin ph tiu hng gc.
Chng ta cn phi bao hm t nht mt tch ph mi mt tiu hng gc. Do , bt c
khi no ch c mt du x trong mt ct ca bng, th tch t-ng ng vi hng c x s
cn phi -c s dng trong biu din cn tm. T bng 4 ta thy c z ln x. y u l cn
thit. Do vy p s cui cng s l z x. y .
Bng 3
z

x. y. z

x. y.z

x. y.z

x. y.z

x. y

x. y.z

By gi chng ta cng c kt li trnh t cc b-c cn thc hin theo ph-ng


php Quine-McCluskey rt gn mt khai trin tng cc tch:
1. Biu din mi tiu hng n bit bng mt xu bit c di n vi s 1 v tr th i
nu xi xut hin v vi s 0 nu x i xut hin.
2. Nhm cc xu bit theo s cc s 1 c mt trong chng.
3. Xc nh tt c cc tch n-1 bin c th to thnh bng cch ly tng Boole cc
tch trong khai trin , Cc tiu hng c th t hp -c biu din bng cc xu
bit ch khc nhau mt v tr. Biu din cc tch n-1 bin ny bng cc xu bit c
s 1 v tr th i nu c xi , hoc s 0 nu v tr c x i hoc l mt du
gch ngang nu khng c khng c mt tc bin no lin quan n xi trong
tch.
4. Xc nh tt c cc tch n-2 bin c th -c to thnh bng cch ly tng Boole
ca cc tch n-1 bin tm -c b-c tr-c. Cc tch n-1 bin c th t hp
-c biu din bng cc xu bit c du gch ngang cng v tr v khc nhau
ch mt v tr trong s cc v tr cn li.

5. Tip tc tin hnh t hp cc tch Boole thnh cc tch c s bin t hn theo


cch t-ng t b-c 4 cho ti khi no khng th rt gn -c na.
6. Tm tt c cc tch Boole xut hin nh-ng khng -c dng lp tch Boole vi

s tc bin bt i 1.
7. Lp bng xc nh mi tch thu -c ph nhng tiu hng gc ban u no
vi iu kin mi tiu hng phi -c ph t nht bi mt tch. Trn c s xc
nh tp nh nht cc tch Boole sao cho tng ca chng biu din hm cho
ban u.
kt thc, ta xt thm mt v d minh ho sau:
V d 4.2.1. Tm biu thc cc tiu t-ng -ng vi tng cc tch Boole sau:
v.x. y.z v.x. y.z v.x. y.z v.x. y.z v.x. y.z v.x. y.z v.x. y.z v.x. y.z

Kt qu thc hin cc b-c ca ph-ng php Quine-McCluskey -c cho trong


cc bng 4-6.
Bng 4
Tiu hng

Xu bit

S cc s 1

v.x.y.z

1111

v. x. y. z

0111

v. x. y. z

1011

v. x. y. z

1101

1110

v.x. y.z

0011

v.x. y.z

1010

v. x. y. z

0001

v. x. y. z

Bng 5
B-c 1
S hng

B-c 2

Xu bit

S hng

Xu bit

S hng

v.x. y.z

1111

(1,2)

x. y. z

-111

(1,2,3,6)

v.x. y.z

0111

(1,3)

v. y.z

1-11

(1,3,5,7)

v. x. y.z

1011

(1,4)

v. x.z

11-1

v. x. y.z

1101

(1,5)

v.x. y

111-

v. x. y.z

1110

(2,6)

v. y.z

0-11

v.x. y.z

0011

(3,6)

x. y. z

-011

v. x. y.z

1010

(3,7)

v. x. y

101-

0001

(6,8)

v.x.z

00-1

v. x. y.z

Xu bit

y.z

v.y 11
1-

-1-

Bng 6

v.x. y.z
v.y

y.z

v. x.z

v. x. y. z

v. x. y.z

v. x. y. z

x
x

v. x. y. z

v. x. y.z

v. x. y.z

v. x. y. z

x
x

x
x

v.x.z

T bng 6 ta suy ra biu thc cc tiu cn tm l:


v.x.z v.x.z v. y y.z

Bi tp.

Vit biu thc Boole biu din cc mch t hp sau:


1.

x1
x2

2.

3.

x1

x1

x2

x2

x3

4.
x1
x2

x3

5.
x1
x2
x3
x4

Cho x1=1, x2=1, x3=0, x4=1, tnh gi tr cc biu thc Boole sau:
6. ( x1 ( x 2 ( x1 x 2 ))) (( x1 x 2 ) ( x1 x 3 )
7. ((( x1 x 2 ) ( x 3 x 4 )) (( x1 x3 ) ( x 2 x3 ))) ( x1 x 4 )
Kim tra cc ng thc sau:
8. x1 ( x 2 x 3 ) ( x1 x 2 ) ( x1 x 3 )
9. ( x1 x 2 ) ( x 3 x 4 ) ( x 3 x1 ) ( x 3 x 2 ) ( x 4 x1 ) ( x 4 x 2 )
10. Gi s X ( x1 , x2 ,..., xn ) l mt biu thc Boole bt k trn cc bin x1 , x2 ,..., xn .

Chng minh rng X * ( x1 , x2 ,..., xn ) X ( x1 , x 2 ,..., x n ) (du * ch biu thc i ngu).


11. Cho X ( x1 , x2 ,..., xn ) v Y ( x1 , x2 ,..., xn ) l hai biu thc Boole trn cc bin
x1 , x2 ,..., xn v X Y . S dng kt qu bi tp 10, chng minh rng X * Y * .
12. Hy chng t cng AND c th biu din qua cc cng OR v NOT.
13. Cng NAND l mt mch logic nhn u vo l 2 bit x1 v x2, cho u ra k kiu l
0 nu x1 x2 =1;

x1 x2
1 trong cc tr-ng hp cn li.

Cng NAND -c biu din nh- hnh bn. Hy x


chng t ch dng cc cng NAND c th thit k cc y
mch t-ng -ng vi cc cng AND, OR, NOT.

x1 x2

14. Gi s S={1,2,3,6} v cc php ton .,+, trn S -c nh ngha nh- sau:


x+y=USCNN(x,y); x.y=BSCNN(x,y); x=6/x.
Chng t (S,.,+,,1,6) l mt i s Boole.
Tm cc dng tuyn chun tc ca cc hm sau v v cch mch t hp t-ng

ng ca chng.
15.

x1

x2

f(x,y)

Hm vi gi tr cho trong bng trn -c gi l hm XOR v k hiu l x y .

Biu din hm XOR qua cc cng AND, OR, NOT -c cho trong hnh v d-i y.
x
y

x y

16.
x

f(x,y,z)

f(x,y,z)

17.

18.
x

f(x,y,z,t)

S dng cc tnh cht i s Boole tm dng tuyn chun tc ca cc hm sau


(d-i y thay v a b ta s vit n gin l ab ):

19. f ( x , y ) ( x y )( x y )
20. f ( x , y , z ) ( xy xz )( x yz )
21. f ( x, y, z ) x ( y ( x y xz ))
22. f ( x, y, z ) ( xy xz )( xyz yz )( x yz x y x yz xyz )
23. f ( w, x, y, z ) (wx yz x yz )( xyz yz )( x yz x. y x yz xyz )
24. Chng minh rng, tng Boole y1 y2 ... yn trong yi xi hoc yi x i c gi

tr 0 i vi ch mt t hp gi tr cc bin, c th l khi xi 0 nu yi xi v xi 1
nu yi x i . Tng Boole ny -c gi l mt i hng (maxterm).

25. Chng minh rng mt hm Boole c th -c biu din nh- tch cc i hng (dng
hi chun tc ca hm). (Gi : -a mt i hng vo tch ny i vi mi t hp gi tr
ca cc bin lm cho hm nhn gi tr 0).

26. Tm dng hi chun tc ca cc hm cho trong bi tp 15-18.

27. Thit k cc mch thc hin s b phiu theo a s cho 3 v cho 5 thnh vin.
28. Thit k h thng mch n -c iu khin bi 2 cng tc sao cho n sng khi c hai cng
tc cng m hoc cng ng, n tt trong cc tr-ng hp cn li.
Tm cc khai trin cc tiu ca cc biu thc sau:
29. xyz xyz xyz xy z
30. x. y. z.t x. y.z.t x. y.z.t x. y.z.t x. y. z.t
31. x. y. z.t x. y.z.t x. y.z.t x. y.z.t x. y.z.t x. y.z.t x. y.z.t x. y. z.t x. y. z.t
32. Tm cc mch n gin hn c cng u ra i vi cc mch sau:
x
y
z

xyz
xyz +xyz

x
y
z
x
y
z
x
y
z
x
y
z
x
y
z

y
z

xyz

xyz

x y z

x
y
z

xy z +xyz+ xyz+ xyz


xyz

x y z

Ch-ng V.
Automat, vn phm v ngn ng hnh thc
Trong ch-ng ny chng ta s nghin cu ba loi cu trc ri rc, l cc my hu hn

trng thi, cc vn phm v my Turing. Cc loi my hu hn trng thi -c dng trong vic
m hnh ho; cc vn phm -c dng cu to cc t ca mt ngn ng v xc nh mt t c

trong mt ngn ng hay khng; my Turing -c dng nhn dng cc tp hp ni chung v


on nhn cc ngn ng ni ring. y l cc cu trc c lin quan cht ch vi nhau v ng
vai tr quan trng trong cc m hnh tnh ton ca my tnh hin i.
1. Mch tun t v my hu hn trng thi
phn trn, chng ta nghin cu v i s Boole v cc s mch th hin cc biu

thc trn i s Boole. M hnh y mi phn nh -c s ph thuc ca thng tin u ra vi

thng tin u vo. N cn khc xa vi cc h thng x l thng tin trong thc t. i vi cc h

thng x l thng tin trong thc t, thng tin u ra khng ch ph thuc vo thng tin u vo
m cn ph thuc vo kt qu cc b-c x l trung gian. phn nh cc h thng x l thng
tin trong thc t, ng-i ta th-ng s dng cc m hnh mch in c gn thm b nh hoc b
x l thng tin. H thng mch c kt ni vi b nh hoc b x l thng tin gi l h thng
mch tun t hay h thng mch lin tip (sequential circuit).
1.1. Mch tun t
nh ngha 1.1.1. Mt cch trc quan chng ta c th m t h thng mch tun t nh-

hnh 1. C th coi h thng mch tun t gm:

- Mt knh vo X ( knh nhn thng tin vo)


- Mt knh ra Y ( knh pht tn hiu ra)
- Mt b x l thng tin S
x

B ch bin thng tin


v bin i trng thi
S

Hnh 1. S mch tun t


Chng ta qui -c qu trnh hot ng ca h thng mch lin tip l ri rc. Ngha l vic

nhn tn hiu vo, pht tn hiu ra v vic thay i trng thi ca b x l thng tin xy ra lin
tip ti nhng thi im khc nhau. Ni mt cch khc chng ta c th phn chia qu trnh hot

ng ca h thng theo thi gian sao cho ti mi thi im trong dy thi im t1, t2 ,.., tn, vic

nhn tn hiu vo, pht tn hiu ra v thay i trng thi ca b x l thng tin xy ra ng thi.
Mt h thng hot ng theo nguyn l trn gi l h thng hot ng theo qu trnh ri rc.

Nu k hiu t l thi im ang xem xt, tn hiu vo t knh x thi im t l xt , tn

hiu pht ra ti thi im t knh ra y l yt v trng thi ca b x l thng tin ti thi im t

l st . Khi c th din t qu trnh hot ng ri rc ca h thng mch tun t theo cng

thc ton hc sau:

st+1=(st, xt );
yt=(st, xt );
y k hiu ch nguyn tc ph thuc ca trng thi st+1 i vi st, xt ; k hiu ch nguyn

tc ph thuc ca tn hiu yt i vi trng thi st v tn hiu vo xt .

V d 1.1.1. B cng hai s nh phn x=0xnxn-1 ....x0 v y=0ynyn-1 ....y0 l mt h thng mch lin
tip. B cng ny hot ng theo nguyn tc cng cc bt t-ng ng theo th t vo t phi sang
tri , tng nhn -c l dy cc bt zn+1 zn zn-1.... z1 z0 . Hnh 2 m t trng thi lm vic ca b

cng khi n=3 ti cc thi im t0, t1, t2 vi x=010 ; y=011 v z=101


x0 = 0
y0 = 1
x1 = 0
y1 = 1
x2= 0
y2= 0

B cng

z0= 1

B cng

z1 = 0

t0 = 0

t1 = 1

a)

b)

B cng
c)
t2 = 2
Hnh 2.

1.2. My hu hn trng thi


My hu hn trng thi l mt h thng mch tun t -c nh ngha hnh thc nh- sau.
nh ngha 1.2.1. My hu hn trng thi M l b 6 k hiu M=(S, X, Y, , , s0):
- S l tp hu hn, gi l tp cc trng thi
- X l tp hu hn gi l bng cc k hiu vo hay bng vo
- Y l tp hu hn gi l bng cc k hiu ra hay bng ra
- l nh x; : S x X S; -c gi l hm chuyn trng thi

- l nh x; : S x X Y; -c gi l hm ra
- s0 S l trng thi bt u ca my trng thi
V d 1.2.1. B M=(S, X, Y, , , s0) cho sau y l mt my hu hn trng thi:

S=q0,q1; X=a, b; y=0,1; s0=q0 ; hm chuyn trng thi v hm ra -c cho bng:


+ Bng xc nh hm chuyn trng thi
X/S

q0

q1

q0

q1

q1

q1

Theo bng trn ta c (a,q0)= q0; (a,q1)= q1 ; (b,q0 )= q1; (b,q1)= q1.
+ Bng xc nh hm ra
X/S

q0

q1

Theo bng trn ta c (a,q0)=0; (a,q1)=1; (b,q0 )=1; (b,q1)=0;


tin cho vic xem xt mt cch trc quan my hu hn trng thi, ng-i ta -a ra cc

ph-ng php biu din sau:

Ph-ng php th
Trong ph-ng php th, ng-i ta biu din mi my hu hn trng thi vi th c

h-ng, trong mi trng thi ca tp S ng vi mt nt trong th, cung c h-ng i t nt p

n nt q v c nhn a/ nu (p,a) =q v (p,a)= . Nt ng trng thi bt u ca my s


c mi tn khng i ra t nt no tr vo n .

V d 1.2.2. My hu hn trng thi trong v d 1.2.1 c th biu din bi th hnh 3.

a/1

a/0
start

q0

b/1

q1

b/0
Hnh 3. Biu din th ca my hu hn trng thi
Ph-ng php bng
Trong ph-ng php bng hay cn gi l ph-ng php ma trn biu din my hu hn

trng thi ng-i ta s dng 2 ma trn gm m dng v n ct, trong m l s trng thi ca S, n
l s k t vo ca bng vo X. Ma trn th nht dng xc nh hm chuyn trng thi: phn
t hng i, ct j ghi trng thi qk nu (qi,aj) = qk. Ma trn th hai dng xc nh hm ra :
phn t hng i, ct j ghi kY nu (qi,aj) = k (xem v d 1.2.1).

2. Automat hu hn
Automat hu hn l mt tr-ng hp c bit ca my hu hn trng thi, tuy vy n li l

cng c rt quan trng nghin cu ngn ng thut ton v s l thng tin tun t.
2.1. Khi nim v nh ngha

nh ngha 2.1.1. Automat hu hn l b 5 k hiu M = (S, , , q0, F), trong :


- S l tp hu hn khc rng, gi l tp cc trng thi.
- l tp hu hn khc rng, gi l tp k hiu vo (gi tt l bng vo).
- l nh x : S x S, gi l hm chuyn trng thi.
- q0 l trng thi bt u: q0S .
- F l tp con ca S, FS gi l tp trng thi kt thc.
nh x (p, a)=q, vi p, qS, a l php chuyn t trng thi p sang trng thi q khi

automat nhn k t vo a.

Ch chng ta s m rng hm chuyn t S x vo S thnh hm t S x * vo S nh-

sau: (q , )=q ; l k t rng v (q,xa)=((q, x),a) a, x*, qS;

T nh ngha ca automat hu hn, chng ta thy rng automat hu hn l tr-ng hp

ring ca myhu hn trng thi khi m tp k hiu u ra Y= v hm ra (q,a)= vi mi


qS, aX.

Khc vi my hu hn trng thi, trong nh ngha automat hu hn ng-i ta c bit

quan tm n tp trng thi kt thc FS, tp trng thi F cn -c gi l tp trng thi chp
nhn -c (accept) ca automat.

2.2. Biu din automat hu hn


T-ng t nh- my hu hn trng thi, chng ta c th s dng ph-ng php th hoc

ph-ng php bng biu din automat, ch l-u mt iu l vi automat hu hn chng ta cn


nh du cc trng thi kt thc. D-i y chng ta s s dng vng trn kp ch trng thi
kt thc.

V d 2.2.1. Xt automat M = (S, , , q0 , F) vi S=q0 ,q1, q2; =a, b; F=q2 v hm

chuyn trng thi -c cho bng sau :

S\

q0

q0

q1

q1

q0

q2

q2

q0

q2

Dng th ca automat M cho hnh 1.


a
q0

b
a

b
q1

q2

a
Hnh 1. Biu din th ca automat hu hn
2.3. Ngn ng on nhn bi automat
Tr-c ht chng ta nh ngha xu -c gi l on nhn bi Automat hu hn M
nh ngha 2.3.1. Cho automat hu hn M = (S,,,q0,F). Gi s =x1 x2..... xn l mt xu

trn trn , (xi i=1,2...n ). Ta ni rng xu on nhn -c bi automat hu hn M, nu tn

ti dy trng thi qk0,qk1 , qk2,..., qkn -1 , vi qkj S tho mn iu kin :

- qk 0=q0;
- (xi, qki -1 )= qki vi i=1,2,...,n
- qkn F.
Dy (qk0,qk1, qk2,..., qkn - 1) -c gi l dy trng thi on nhn xu trn M.
Hnh 2 minh ho cho nh ngha trn:
W =X 1 X2 X3 .Xn
q0

q1

q2 ..qn -1

qn

Hnh 2.
nh ngha 2.3.2. Cho automat hu hn M = (S,,,q0,F), tp tt c cc xu on nhn

-c bi automat M -c gi l ngn ng on nhn bi automat hu hn M.

Chng ta k hiu ngn ng on nhn bi automat M l: T(M) = w (q0 ,w)F, w*


nh ngha trn coi ngn ng on nhn bi automat M l tp tt c cc xu vo x *

sao cho khi automat trng thi ban u q0, nhn xu vo x th automat c th chuyn dch v

trng thi kt thc thuc tp F.

nh ngha 2.3.3. Cho hai automat hu hn M = (S,,,q0,F) v M = (S,,,q0,F). Ta

ni rng M v M l hai automat t-ng -ng nu T(M)= T(M).

nh ngha trn coi hai automat cng on nhn mt ngn ng l t-ng -ng.
V d 2.3.1. Cho automat hu hn M = (S, ,,q0,F):
S={q0,q1,q2,q3}; ={0,1}; F={q0}
Hm chuyn xc nh nh- sau:
(q0,0)=q2

(q0,1)=q1

(q1,0)=q3

(q1,1)=q0

(q2,0)=q0

(q2,1)=q3

(q3,0)=q1

(q3,1)=q2

S m t automat M -c cho trn hnh 3. C th ch ra xu =0011 l xu on nhn

bi M.

start

q0
0

q2

1
1

q1
0

q3

Hnh 3.
2.4. Automat khng tt nh (nondeterministic automat)
nh ngha 2.4.1. Automat hu hn M = (S, , , q0, F), -c gi l automat tt nh nu

a, q S , (a,q) 1 (*)

Automat tt nh cn c tn gi l automat n nh. Automat khng tho mn iu kin

(*) gi l automat khng tt nh (nondeterministic automat) hay khng n nh. Nu M l


automat khng n nh khi (x,q) ni chung s l mt tp con ca S.

Ch rng vi automat hu hn khng tt nh M=(S,,,q0,F), ngn ng on nhn bi

M -c xc nh l tp :

N(M) = {x(q0,x)F, x *}
Chng ta thy rng i vi automat tt nh, vi mi k t vo v t mt trng thi n xc

nh khng qu mt trng thi chuyn tip theo. Ng-c li, i vi automat khng tt nh, vi
mi k t vo v t mt trng thi, trng thi chuyn tip ni chung l mt tp con ca S.
i vi automat khng tt nh, nh x c th m rng trn S x * nh- sau:

p(q,x) (p,a) vi

(q,)={q} ;(q,xa)=

x*; p,qS.

V d 2.4.1. Xt automat khng tt nh M sau


M = (S, , , q0 , F) ; S = {q0 ,q1,q2,q3,q4}; = {0,1};F = { q2,q4}
(q0 ,0)={q0,q3}; (q3 ,0)={q4}; (q0 ,1)={q0 ,q1}; (q3 ,1) = #
(q1 ,0)=# ;(q4 ,0)= {q4};(q1 ,1) = {q2 }; (q4 ,1)= {q4}; (q2 ,0)= {q2};
( q2 ,1)= {q2}; ( q2 ,0)= {q2}. Ch rng k hiu # ch rng khng xc nh.

Biu din M -c cho hnh 1. Ta c th thy t w=00110011 on nhn -c bi M

khng tt nh.

0,1

0
0

q0

q3

q4

1
q1
q2

1
0,1
Hnh 1.

2.5. Quan h gia automat tt nh v khng tt nh


Vn t ra y l liu ngn ng on nhn bi automat tt nh v khng tt nh c

khc nhau hay khng? Lin quan n iu ny chng ta c nh l sau:

nh l 2.5.1. Gi s N(M) l ngn ng on nhn bi automat khng tt nh M, khi

s tn ti automat M tt nh on nhn N(M), hay ni cch khc N(M)=T(M).

Chng minh: Gi s cho M = (S, , , q0, F) l automat khng tt nh, ta xy dng

automat M' = (S', , ', q'0, F') nh- sau:

S' l tp tt c cc tp con ca S (S= 2S )


q'0 = {q0}, q'0 l tp con ca S c mt phn t q0
F' ={USUF}; F' l mi tp con ca S giao vi F khc rng.
nh x ': S' x S' -c xy dng bng cch da vo nh- sau:
'(U,a)= sU (s,a) vi U S', a
R rng vi cch xy dng ' nh- trn th M' l automat tt nh. By gi ta chng minh

M' on nhn N(M). Ni mt cch khc ta cn chng minh:

'({q},x)= (q,x) vi mi qS, x. (1)


Chng ta s chng minh qui np theo di xu x*. Nu x c di bng 0,x=0,

ngha l x=e, ta c: '({q},x)= '({q},e)={q}=q=(q,e) . R rng vi x mx=0 th (1) hin

nhin ng.

Gi s cng thc (1) ng vi mi xu x m xn, n 0. Ta xt xu x di n+1. Vi

gi thit ny ta c th vit dng x=wa vi w=n, a=1 khi ta c:


'({q},x)='({q},wa)='('({q},w),a)
Theo nh ngha ca ' ta c:

'('({q},w),a)=Uq''(q,w) (q',a)=Uq'(q,w)(q',a)=(q,wa)=(q,x)
T (*) ta suy ra:
xN(M)(q0,x)F'({q0},x)F'(q0,x)F'xT(M')
V d 2.5.1. Cho M = (S, , , q0 , F) l automat khng tt nh: S=q0,q1; =0,1

;F=q1; (q0,0)=q0,q1;(q0,1)=q1;(q1,0)=; (q1,1)=q0,q1.

Chng ta c th xy dng automat tt nh M' on nhn T(M) nh- sau :


M'=<S',', ', q'0 ,F'>
S'=[q0], [q1],[q0 ,q1 ] ,;
'=0,1;
q'0=[q0];
' -c xc nh nh- sau:
(q0,0)= q0,q1 do '([q0],0)=[q0,q1];
T-ng t ta c :
'([q0],1)=[q1] ;'([q1],0)= ; '([q1],1)=[q0,q1] ;
'(,0)='(,1)= ;
'([q0,q1 ],0)=[q0,q1] v (q0,q1,0)=(q0,0) (q1,0) =q0 ,q1
'([q0,q1],1)=[q0,q1] v (q0 ,q1 ,1)=(q0,1) (q1,1)=q0,q1
F=[q1],[q0,q1]
Nh- vy, automat tt nh v automat khng tt nh l t-ng -ng nhau v mt ngn

ng. Tuy nhin v ph-ng din cu trc v vit ch-ng trnh dch th automat khng tt nh
phc tp hn nhiu so vi automat tt nh, v vy ng-i ta cn bin i automat khng tt nh
v automat tt nh. D-i y chng ta s tm hiu mt thut ton bin i automat khng tt
nh v automat tt nh.

Tr-c ht ta -a ra mt vi nh ngha. Gi s automat hu hn M= <S,,,q0,F> l

automat khng tt nh.

nh ngha 2.5.1. Ta gi tp ED(s) l tp cc trng thi m M c th t n -c t trng

thi sS khi automat c vo t rng :

ED(s) = {qS(s, ) = q}
nh ngha 2.5.2. Ta gi tp ED(T) l tp cc trng thi m M c th t n -c t mt

trng thi sT vi TS khi automat c vo t rng :

ED(T) = {qS(s,) = q; sT; TS }


nh ngha 2.5.3. Ta gi tp Move(T, a) l tp tt c cc trng thi m M c th t n

-c t mt trng thi sT vi TS khi automat c vo t a.

Move(T,a)=)={qS(s,a)=q; sT; TS; a }


Thut ton bin i automat -c xy dng da trn t-ng sau: Gi s automat tt nh

cn phi xc nh l D, khi ta cn xy dng tp trng thi ca automat D, k hiu l Dstate v


bng chuyn trng thi Dtran. Mi trng thi ca automat tt nh D s ng vi mt tp cc
trng thi ca automat khng tt nh M m M c th t n -c sau khi c mt k t ca
t vo. Thot u D c trng thi bt u l ED(q0) v trng thi kt thc ca D l tp trng thi

c cha mt trng thi kt thc ca M. Qu trnh tnh ED(T) l qu trnh tm cc nt c th n


-c trn th t tp cc nt xut pht cho tr-c.

Thut ton 2.5.1. Thut ton bin i automat.


Input: Automat khng tt nh M= <S,,,q0,F>
Output: Automat tt nh D
1. Tm -closure(q0) -a vo bng trng thi Dstate, coi n l trng thi ch-a nh du.
2. Nu trong Dstate c trng thi T ch-a nh du, th nh du xt cho T v vi mi
a tnhU=ED(move(T,a)). Nu U l trng thi mi ch-a c trong Dstate th -a U

vo Dstate v -a vo bng hm chuyn Dtran[T,a]:=U.

3. Lp li b-c 2 nu Dstate cn trng thi ch-a nh du xt.

Hnh 1 l s khi ca thut ton nu trn.


T:=ED(q0)
Dstate:=T
nh du T vi mi a
U:=ED(Move(T, a)
U Dstate
n

Dstate:=U Dtran[T, a]:=U


n

Dstate c T
ch-a nh du

Stop

Hnh 1.
Ch rng thut ton trn chc chn s kt thc sau mt s b-c hu hn no , bi v

xt cho cng th ta dng cc phn t trong Dstate nh du cc tp con ca cc trng thi

automat khng tt nh, m s tp con ca mt tp hu hn khng v-t qu 2S vi s l s trng


thi ca automat khng tt nh.

V d 2.5.2. Xt automat hu hn M= <S,,,q0,F> khng tt nh, vi S =

{0,1,2,3,4,5,6,7,8,9,10}; q0 = {0}; F = {10}; ={a,b}; hm cho trong hnh 2.

C th kim tra li rng tp ngn ng on nhn bi M l tp N(M)=(ab)*abb


E

Start

1
E

E
E

a
b

6
E

Hnh 2.
Tnh A =ED(0) = {0,1,2,4,7}; -a A vo Dstate;
Tnh Move(A,a) = {3.8};

10

b
start

B
a

b
b

C
a

D
a

E
a

Tnh B=ED(Move(A,a))=closure({3.8})={1,2,3,4,6,7,8}; -a B vo Dstate.


V do bng chuyn trng thi ca D c gi tr Dtran(A,a)=B.
Trong s cc trng thi tp A, c 4 trng thi chuyn sang trng thi 5 khi nhn thy b, do

ch c mt trng thi t A n C khi nhn thy b. Vi C -c xc nh :


C =ED({5}) = {1,2,,4,5,6,7}; v vy Dtran[A,b]=C.
Tip tc lm t-ng t vi cc trng thi ch-a xt trong bng Dtran ta c :
A = {0,1,2,4,7}
B = {1,2,3,4,6,7,8}
C = {1,2,4,5,6,7}
D = {1,2,4,5,6,7,9}
E = {1,2,4,5,6,7,10}

y A l trng thi bt u v E l trng thi kt thc ca automat tt nh. Bng d-i

y l bng chuyn trng thi ca automat v hnh 3 l th biu din automat ny.
Trng thi

Hnh 3.
kt thc mc ny, chng ta xem xt mt thut ton kim tra xu on nhn bi mt

automat cho.

Thut ton 2.5.2. Thut ton kim tra xu on nhn bi automat


d hnh dung chng ta s xem xt thut ton kim tra xem mt xu vo c on nhn

bi automat hu hn hay khng thng qua automat M cho bi s hnh 4.


b
E

a
a

Hnh 4.
Input: S n l di xu ; xu =x1 x2 x3 ... xn ; vi xia,b.
Output: In ra Accept nu T(M) hoc No Accept trong tr-ng hp ng-c li.
1. t state:=E ; i:=1;
2. Nu state=E th chuyn ti b-c 3, ng-c li state=f, chuyn ti b-c 4.
3. Nu xi=a th t state:=f; nu xi=b th t state:=E. Chuyn n b-c 5.
4. Nu xi=a th t state:=E; nu nu xi=b th t state:=f.
5. Nu i<n th i:=i+1 quay li b-c 2, ng-c li chuyn ti b-c 6.
6. Nu state=f in thng bo accept; nu state=E in thng bo no accept. Kt
thc thut ton.

3. Vn phm v ngn ng
3.1. Cc khi nim v nh ngha
Ta tng nghe, tng ni nhiu v ngn ng nh- ngn ng ting Anh, ting Vit, ngn ng

lp trnh Pascal, ngn ng thut ton v.v... Tuy nhin, t ai c th tr li chnh xc ngn ng l g?
C bao nhiu loi ngn ng? V iu quan trng hn l lm th no c ngn ng?

T lu cc nh ngn ng hc nghin cu v ngn ng, song vic nghin cu ca h

thin v cc kha cnh x hi, lch s ca ngn ng nh- qu trnh hnh thnh ngn ng, cc c
tr-ng ngn ng ca cc dn tc v.v... T khi my tnh ra i, ng-i ta c nhu cu phi to ra
ngn ng din t thut ton, phi dch t ngn ng thut ton ny sang ngn ng thut ton
khc v.v... Khi buc ng-i ta phi gii quyt mt lot cc vn -c t ra nh-: Ngn ng
l g? Lm th no c ngn ng? Ngn ng thut ton cn phi c c tr-ng g?...

Quan tm nghin cu, gii quyt nhng vn nu trn l i t-ng nghin cu ca ngn

ng hnh thc. Phn ny s gip chng ta b-c u lm quen vi mt s khi nim, thut ng v
ph-ng php tip cn gii quyt vn ca cc b mn khoa hc nu trn.

nh ngha 3.1.1. Bng ch ci hay b ch ci l mt tp bt k gm mt s hu hn cc

phn t gi l cc ch ci hoc cc k t.

V d 3.1.1. Cc tp sau u l cc bng ch ci: Tp hp cc ch ci La tinh L= a, b,

c... A, B...Z l mt bng ch ci; tp 0,1; tp X= , , ,... , U= 0,1 ... 9.

ch mt ch ci thuc hay khng thuc bng ch ci ta dng k hiu hoc .


V d 3.1.2. Xt cc bng ch ci nu trong v d 3.1.1. Khi X, X, 9U, aX,

15U, aL.

nh ngha 3.1.2. Xu hay t trn bng ch ci l mt dy bt k cc phn t ca bng

ch ci. di ca xu l s k t c trong xu, di ca xu x k hiu l x.


Ch :
Thut ng xu hoc t cn ng ngha vi chui k t.

Mt xu rng l xu khng c ch ci no, di ca xu rng bng khng, xu rng


-c k hiu l . C th coi xu rng l xu -c to lp t bng ch ci bt k.

Cho V l mt bng ch ci, chng ta k hiu V* l tp tt c cc xu sinh ra t V k c


xu rng, ta k hiu V+ l tp tt c cc xu sinh ra t V tr xu rng: V* =V+{}

V d 3.1.3. Nu X=0,1 ... 9, th X+=0,1,19, 105, 11990...... . Nu Y=a,b,c}, th

Y+=aabcc, aaaaa, cab,... l cc xu trn bng ch ci Y. Nu V = 0,1, V*=

,0,1,00,01,10,11,000... , V+ = 0,1,00,01,10,11,000... .

nh ngha 3.1.3. Cho bng ch ci V, ta gi tp con bt k ca tp V* l ngn ng trn

bng ch ci V.

V d 3.1.4. Gi s V = a. Xt tp L = ai aV, iN; y k hiu ai thay cho vit k

t a i ln. Khi L l ngn ng trn bng ch ci V (N l tp cc s nguyn).

Vi nh ngha trn, r rng ngn ng L cng l mt tp hp, ta k hiu L l s cc

xu hay s phn t ca L, nu L l mt s hu hn ta ni rng L l ngn ng hu hn, ng-c

li L l v hn.

V ngn ng L l mt tp hp, cho nn cc php tnh trn tp hp nh- php hp, php

giao, php ly phn b... trong l thuyt tp hp u c th p dng trn cc ngn ng ca cng
mt bng ch ci. Chng hn cho V l bng ch ci, L1, L2 l hai ngn ng trn V th

L1 L2 = xxL1 hoc xL2}.


nh ngha 3.1.4. Cho L, L1 l ngn ng trn bng ch ci V. Chng ta nh ngha php

nhn ghp gia L, L1 nh- sau:

G=L L1=ww=uv; uL, vL1


Chng ta s s dng mt s k hiu sau: L0 = ( k hiu t rng); tp Ln = Ln-1L,

n=1,2,...; php lp L+ = L1L2L3......= nN Ln ; L*= nN Ln .


3.2. Vn phm v ngn ng

Mt cng c m t hu hn ngn ng l vn phm. y l cng c c nh ngha ton

hc cht ch, -c cc nh ton hc nghin cu k v tr thnh mt thnh phn quan trng


trong l thuyt ngn ng.

nh ngha 3.2.1. Vn phm G l b gm 4 thnh phn G= (N,T,P, k). Trong :


N - l mt tp hu hn ; N -c gi l tp cc bin hay tp cc nonterminal
T - l tp hu hn ;T -c gi l tp cc k hiu cui hay tp cc terminal; y hai tp
N v T khng c chung phn t: NT=

P - l tp con ca tp (NT)+ x (NT)* (P - -c gi l tp cc qui tc sn xut hay tp


cc qui tc dn xut, hoc tp cc qui tc sinh. Ni mt cch khc P l mt nh x t tp
(NT)+ sang tp (NT)*

P: (NT)+ (NT)*
Gi s p (NT)+, q(NT)* v q l nh ca p qua nh x P, khi ta k hiu p q v

gi n mt qui tc dn xut ca G.

Nu c mt dy cc qui tc dn xut trong G dng: p1 p2, p2 p3, ..., pn-1 pn ta k

hiu ngn gn l p1* pn ; pi(NT)*

k - l phn t c bit ca N (kN ) -c gi l k t bt u.


Ch :
T y tr i trnh nhm ln chng ta qui -c cc phn t thuc N -c k hiu l
cc ch hoa A,B,C..., cc phn t thuc T ta k hiu l cc ch th-ng a,b,c..., cc xu
gm cc k hiu ca T k hiu l x,y,u,v,w,... cc xu thuc (NT)* ta k hiu l ,, ...
Nu khng xy ra nhm ln th mi qui tc p q -c hiu l mt dn xut trong G.
Nu cn thit trnh nhm ln ta k hiu p * G q.

nh ngha 3.2.2. Cho vn phm G = (N,T,P, k). Ta gi tp L(G)= wwT* v k*w

l ngn ng sinh bi vn phm G.

nh ngha trn cho php chng ta c th coi ngn ng sinh bi mt vn phm G cho

tr-c l tp tt c cc xu sao cho n l kt qu ca mt s ln s dng cc qui tc dn xut


trong G bt u t bin k.

V d 3.2.1. Cho G = (N,T,P,k) vi N= k ; T= a ; P= k aaa, k aaak . Khi

L(G)= a3n nN.

V d 3.2.2. Cho vn phm G = (N,T,P,k) vi N= k ; T = 0,1; P= k 0k1, k

01. Ta c nhn xt mi dy dn xut c th trong G phi c dng:


k 0k1 00k11 ... 0i k1i
T y suy ra L(G) = 0n.1n} n 1.

V d 3.2.3. Chng ta bit tn gi hay bin n gin trong Pascal l mt dy gm cc

ch ci hoc ch s bt u bng ch ci. V d d-i y to ra tp L(G) l tp cc bin n


gin ca Pascal vi bng ch ci hn ch.
Xt

G = <N,T,P,k> :

N = Cci>,<Cs>,<s>,<t>,<tngi> (cc t trong cp < > -c coi l mt k t

hay mt phn t ca N).


T = a,b,c,0,1
k = <tngi>
P:

1) <Cci > a
2) <Cci > b
3) < Cci > c
4) <Cs> 0
5) <Cs> 1
6) <s> <Cs>
7) <s> <Cs> <s>
8) <t > <Cci >
9) <t > <Cci > < t >
10) <tngi> <t> <t><s><t><s><t>

nh ngha 3.2.3. Cho hai vn phm G=(N,T,P,k), G=(N,T,P,k) ta ni rng G t-ng

-ng vi G k hiu GG nu L(G)=L(G). Ni cch khc hai vn phm -c gi l t-ng

-ng nu tp ngn ng sinh ra bi chng trng nhau.


3.3. Phn loi vn phm v ngn ng

trn chng ta nh ngha vn phm, mt cng c sinh ra ngn ng . Tip theo chng

ta s xem xt cc vn lin quan n cu hi c bao nhiu loi ngn ng ? Quan h gia cc loi

nh- th no? Lin quan n cc cu hi ny c nhiu nh khoa hc -a ra cc cch phn loi khc
nhau, song cch phn loi -c coi l ph hp v thun tin cho mc ch nghin cu l thuyt ln
ng dng l cch phn loi ca nh ton hc Chomsky. Chomsky ngh mt h thng phn loi

cc vn phm da vo c tr-ng ca cc qui tc sn xut hay cc lut sinh ca vn phm . Cch phn
loi ca Chomsky -c s dng nh- l cch phn loi ch yu trong l thuyt ngn ng hnh thc.

Theo cch phn loi ca nh ton hc Chomsky c tt c 4 loi vn phm v t-ng ng vi n c 4


loi ngn ng.

Vn phm loi 0
nh ngha 3.3.1. Vn phm m cc qui tc dn xut ca n khng c bt k mt iu kin

rng buc no gi l vn phm loi 0.

Ngn ng sinh bi vn phm loi 0 gi l ngn ng loi 0. Ta k hiu G0 l vn phm loi

0, khi L(G0) hay ngn gn L0 t-ng ng s l ngn ng loi 0.


Vn phm loi 1

nh ngha 3.3.2. Vn phm G -c gi l vn phm loi 1 nu cc qui tc dn xut ca n

c dng: P ; trong ,(N T) + v

Vn phm loi 1 cn c tn gi l vn phm cm ng cnh (context sensitive grammar).


Ngn ng sinh bi vn phm loi 1 gi l ngn ng loi 1. K hiu G1 l vn phm loi 1,

khi L(G1) hay ngn gn L1 t-ng ng s l ngn ng loi 1.


Vn phm loi 2

nh ngha 3.3.3. Vn phm G -c gi l vn phm loi 2 nu cc qui tc dn xut ca n

c dng: P ;trong N, (N T)+ .

Vn phm loi 2 cn c tn gi l vn phm phi ng cnh vit tt l cfg (context free

grammar). Ngn ng sinh bi vn phm loi 2 gi l ngn ng loi 2. K hiu G2 l vn phm


loi 2, khi L(G2) hay ngn gn L2 t-ng ng s l ngn ng loi 2.
Vn phm loi 3

nh ngha 3.3.4. Vn phm G -c gi l vn phm loi 3 nu cc qui tc dn xut ca

n c dng: A aB hoc A a; trong A,BN, aT.

Vn phm loi 3 cn c tn gi l vn phm chnh qui (regular grammar). Vn phm

chnh qui nu trn cn -c gi l vn phm tuyn tnh tri. Trong tr-ng hp cc qui tc sn
xut ca vn phm G c dng A Ba hoc A a th G -c gi l vn phm tuyn tnh phi.

Ngn ng sinh bi vn phm loi 3 gi l ngn ng loi 3. K hiu G3 l vn phm loi 3,

khi L(G3) hay ngn gn L3 t-ng ng s l ngn ng loi 3.

Ch : T cch phn loi trn chng ta c nhn xt: vn phm loi 3 cng l vn phm

loi 2, vn phm loi 2 cng l vn phm loi 1, vn phm loi 1 cng l vn phm loi 0. T
y suy ra nhn xt trn cng ng vi cc loi ngn ng. C th minh ho quan h gia cc loi
vn phm v ngn ng nh- hnh 1.
G0

G1

G2

L0
G3

Hnh 1
Chng ta xt mt s v d v vn phm.
V d 3.3.1. Cho G = (N,T,P, k):
N=S,A,B
V=a,b
P:

S aB

A bAA

S bA

B b

A a

B bS

A aS

B aB

Khi G l vn phm loi 2 hay vn phm phi ng cnh.


V d 3.3.2. Cho vn phm G = (N,T,P,k) :
N= k, B, C ,T = a, b, c
P gm cc qui tc dn xut:

L1

L2

L3

1) k akBC

5) bB bb

2) k aBC

6) bC bc

3) CB BC

7) cC cc

4) aB ab
Khi G l vn phm loi 1 hay vn phm cm ng cnh.
V d 3.3.3. Cho G = (k, A, B, 0,1, P, k):
P:

k 0A

B 1B

k 1B

B 1

A 0A

B 0

A 0k

k 0

A 1B
Khi G l vn phm loi 3, vn phm chnh quy.
V d 3.3.4. Xt vn phm G m L(G) l tp ccs nguyn, G = (N, T, P, k):
N=<digit>,<signed integer>,<unsigned integer>, <integer>
T=0,1,2,3,4,5,6,7,8,9,+,-; k=<integer>
P:
<digit> 0; <digit> 1;......., <digit> 9;
<integer> <signed integer> ;
<integer> <unsigned integer>
<signed integer> + <unsigned integer>
<signed integer> - <unsigned integer>
<unsigned integer> <digit>
<unsigned integer> <digit><unsigned integer>
3.4. Mt s tnh cht ca ngn ng
Bn cht ca ngn ng l tp hp, v vy ngn ng mang y cc tnh cht ca tp hp.

Mt khc, ngn ng li -c sinh bi vn phm v vy chng ta cn phi xem xt thm mt s


tnh cht ca ngn ng ngoi cc tnh cht ca tp hp.

Tnh cht 3.4.1. Loi ca ngn ng ng vi php hp. Ni cch khc, hp ca hai ngn

ng cng loi l ngn ng cng loi.

Tnh cht 3.4.2. Loi ca ngn ng ng vi php lp.


Tnh cht 2 khng nh rng php lp ca mt ngn ng l ngn ng cng loi. Mt cch

hnh thc nu M=L.L...L=Li th M l ngn ng cng loi vi L.

Chng ta c th kim tra tnh ng n ca cc tnh cht trn. lm v d chng ta

kim tra tnh cht 3.4.1.

Gi s cho 2 vn phm cng loi G1=<N1,T1,P1,k1>, G2=<N2,T2,P2,k2>, vn phm G1, G 2

t-ng ng sinh ra ngn ng L1,L2 . Gi s L=L1L2 ta cn chng t L l ngn ng -c sinh bi


vn phm G cng loi vi vn phm G1, G 2. Khng gim tng qut ta c th gi thit T1=T2=T.
By gi ta xy dng vn phm G = <N,T,P,k> tho mn 2 iu kin:
- G cng loi vi G1, G 2
- G sinh ra L m L=L1L2
Chng ta ly N=N1N2k (k khng thuc N1,N2); P = P1P2kk1 , kk2.
Vi cch xy dng G nh- trn r rng G cng loi vi G1, G 2. By gi ta cn chng t:
L=L(G)=L1 L2.
Gi s wL, theo nh ngha k* w trong G, t cch xy dng P, b-c u tin trong G

ch c th: kk1*w hoc kk2 *w. Nh-ng t k1*w hoc k2*w ta suy ra wL1L2.

Ng-c li nu wL1L2, khi wL1 hoc w L2 . Theo nh ngha suy ra k1*w hoc

k2 *w. Nu wL1 c ngha l k1*w suy ra k k1*w hay wL, t-ng t nu wL2 c
ngha l k2*w . Suy ra kk2*w hay wL.

Tnh cht 3.4.3. Cho G = <N,T,P,k> khng phi vn phm loi 0, c k hiu ban u k

v phi ca qui tc dn xut. Khi tn ti vn phm G cng loi v t-ng -ng vi G khng
c k hiu bt u v phi ca qui tc dn xut.

Chng minh: Gi s cho vn phm G = <N,T,P,k>, theo gi thit G c th l vn phm

loi 1 loi 2, hoc loi 3, ta xy dng G = <N, T,P,k> nh- sau:


N=N k; y k l k hiu mi ch-a c trong N v T
P = PkkP, (NT)+

Ni cch khc P gm tt c cc qui tc ca G c b sung thm cc qui tc dng: k

nu trong P c qui tc dng k .

Vi cch xy dng trn r rng trong G khng c qui tc no m k hiu bt u k xut

hin v phi. By gi ta cn chng minh G v G t-ng -ng nhau, ngha l L(G)=L(G).

Gi s wL(G) khi theo nh ngha ta c dn xut k*w, tch b-c u tin ca dn

ny ta c k*w; vi (NT)+. Theo cch xy dng G v trong G c qui tc k nn


trong G c qui tc k, khi ton b dn k*w l trong G v on dn xut *w
trong G cng l trong G, suy ra wL(G) hay L(G)L(G).

Ng-c li gi s wL(G) khi theo nh ngha ta c k*w. n y lp lun t-ng t

nh- trn ta c L(G)L(G). Suy ra L(G)=L(G).

Ch : Tnh cht 3 cho php ta coi cc vn phm loi 1, 2, 3 khng c k hiu bt u

v phi ca cc qui tc dn xut. Do nu t rng L(G) th trong P phi c qui tc k.

Tnh cht 3.4.4. Cho G =<N,T,P,k> khng phi vn phm loi 0, khi L(G)\{} hoc

L(G){} l ngn ng cng loi vi vn phm G.

Chng minh: Tnh cht 3.4.4 d dng -c chng minh nh tnh cht 3.4.3, v nu vn

phm khc loi 0 c sinh ra t rng th tp qui tc dn xut P c cha qui tc k v vy tp

ngn ng L(G){} hoc L(G)\{} s t-ng ng vi vic loi b hay b sung qui tc k ra
vo tp P v iu ny r rng khng lm thay i loi ca ngn ng...
3.5. Tnh qui ngn ng.
nh ngha 3.5.1. Chng ta ni rng vn phm G l qui nu tn ti thut ton xc nh

mt t w cho tr-c c thuc L(G) hay khng?

Tr-c khi khng nh v tnh qui ca ngn ng chng ta c vi nhn xt sau:


Gi s G = <N,T,P,k> l cm ng cnh, t tnh cht ca ngn ng nu trn ta suy ra xu

rng thuc L(G) khi v ch khi tp P c qui tc k. Nh- vy chng ta cn phi kim

tra qui tc k c trong P hay khng? Bng cch loi qui tc k khi P ta c vn
phm cm ng cnh mi G = <N,T,P,k>. Vn phm G sinh ra tp L(G)\ , mi
dn xut trong G tho mn iu kin ca vn phm cm ng cnh. Ngha l trong mi

qui tc dn xut ca G di v phi ln hn hoc bng v tri.

Xt vn phm G trong nhn xt 1, gi s V=NT, V=n v p l s no , khi trong


V c nhiu nht np t c di p m thi.

Xt vn phm G trong nhn xt 1, gi s V=NT, V=n v w l xu khc rng thuc


L(G). Ta suy ra k* w. Gi s dn xut c dng:

k12...m *); y m=w.

T nhn xt 1 nu trn ta suy ra :


1 2 ... m
Gi s rng tn ti i sao cho cc xu i, i+1,..., i+j trong dy dn xut trn c di nh-

nhau v bng s p no . Nu j np khi suy ra trong dy dn xut *) c t nht hai xu

ging nhau. Do nhn xt 2 ta c th b i t nht mt b-c trong dy dn xut trn. Nu r=s


vi r<s, khi dn xut *) c th vit li ngn hn l:

K1...rs+1...m=w.
Nhn xt 3 cho ta nhn xt trc gic rng nu c mt dn xut ca w th n s c dn

xut khng qu di. Nhn xt ny l c s cho nh l sau:

nh l 3.5.1. Cho G=<N,T,P,k> l cm ng cnh khi G l vn phm qui.


Chng minh: T nhn xt trn, ta gi s L(G) khng cha t rng, khi ta c quyn

gi thit P khng cha qui tc k. Gi s w thuc L(G), wV+ v w=n. Ta cn ch ra thut


ton xc nh w c thuc L(G) hay khng?
Ta nh ngha tp Tm nh- sau:
Tm={V+; n; dn xut k* c khng qu m b-c}
R rng T0 ={k}. D dng thy rng c th tm Tm qua Tm-1:
Tm=Tm -1{ ; Tm-1; n} *)
Nu k* v n th phi thuc vo mt Tm no , tri li nu k khng dn xut

-c ra th hoc >n hoc khng thuc bt k Tm no.

T cch xy dng Tm trn suy ra Tm l dy khng gim Tm-1Tm vi mi m1, dy ny

b chn nn s tn ti Tm m Tm=Tm+1...=V. Nu w khng thuc Tm khi w khng thuc L(G)

v -ng nhin nu w thuc Tm th K*w.

Gi s V= NT v V = q, khi trong V+ s xu c di nh hn hoc bng n l

q+q2 +q3+.....+qn .... (1+q)n+1. R rng s t ny l hu hn trong V v vy ta c th so w vi cc


cc t c trong V. Ngha l tn ti thut ton xc nh xem w c thuc L(G) hay khng.
V d 3.5.1. Cho G= <N,T,P,S >:
N={S ,B, C}; T={a,b,c}
P:

S aSBC;

S aBC ;

CB BC; aB ab;

bB bb;

bCbc ;

cCcc;

Xt t w=abac trong L(G); theo cch xy dng Tm trong nh l trn ta c :


T0={S};
T1={S, aSBC,aBC};
T2={S, aSBC, aBC, aBC};
T3={S, aSBC, aBC, aBC, abc };
T4=T3=V
V w=abac khng thuc V nn w khng thuc L(G).
4. Automat hu hn v ngn ng chnh qui
Trong mc ny chng ta xem xt mi quan h ca automat hu hn v ngn ng chnh

qui. Chng ta s chng t s t-ng -ng ca hai quan im, quan im sinh v quan im on
nhn ngn ng.

4.1. Quan h gia automat hu hn v ngn ng chnh qui


nh l 4.1.1. Nu G = (N,T,P,k) l vn phm loi 3 th tn ti automat hu hn M sao

cho T(M) = L(G).

Chng minh: Khng gim tng qut ta gi s G l vn phm tuyn tnh tri. Chng ta xy

dng automat M = (S, , , q0, F) da vo vn phm G nh- sau:


Ly = T
q0=k

S = N{A} A l trng thi b sung, A ch-a c trong N.


F = {k,A} nu k e l qui tc c trong P
F = {A} nu k e khng c trong P.
Ngoi ra, nu ke l qui tc ca P th khng nh h-ng n kt qu chng ta gi thit k

khng xut hin trong v phi cc lut sinh khc. Hm -c nh ngha nh- sau:
(B,a) cha C B aC P
(B,a) cha A B a P B,CN v (A,a)= cho mi aT
By gi ta chng t L(M) = L(G) :
L(M) L(G)?
Ly xL(G), gi s x=a1 ... an . Khi ta c dn xut sau trong G:

ka1A1...a1a2 ...an-2 An-2 a1a2 ... an-1B a1 ... an-1an , vi Ai,BN, aiT.
Theo cch xc nh hm ta suy ra:
(k,a)A1, (A1,a2)A2,..., (An-2,an-1)B, (B,an)A
V A F suy ra L(G)L(M)
L(G) L(M)?
Gi s x=x1x2...xn , xT(M), ngha l (q0,x)F suy ra phi tn ti dy k,A1,A2,...An-1,A

sao cho:

(k,a)A1, (A1,a2)A2,..., (An-1,an)A


Theo cch xy dng M, ta suy ra trong P ca G phi c cc qui tc sau:
ke
A1 a2A2
....................
An-2 an-1 An-1
An-1 an
T y suy ra k*x, iu ny c ngha l xL(G).
Chng ta ch rng nu T(M), khi kF. Suy ra trong P c qui tc k , ngha l

L(G).

V d 4.1.1. Xt automat G = {(k,A,B), {0,1}, P,k} c P l:


k 0A

B 1B

k 1B

B 1

A 0A

B 0

A 0k

k 0

A 1B
Chng ta c th xy dng automat hu hn on nhn cc xu ca L(G):
M = <S,,, q0,F)>
S = { k,A,B }{C}, q0={k}, F={k,C}
= {0,1}

Khi Automat c th xc nh theo th hnh 1.


0
K
0

1
C

Hnh 1.
nh l 4.1.2. Cho automat M = (S, , , q0, F) khi tn ti vn phm G loi 3 sao cho

L(G) = T(M).

Chng minh: Khng gim tng qut gi s rng M l automat n nh M = (S, , , q0,

F). Ging nh- trong nh l 4.1.1 ta xy dng G da vo M nh- sau:


G = (N,T,P,k)
Ly T =
N=S
k = q0
Tp qui tc P xc nh nh- sau:
1, B aCP nu (B,a) = C
2, B aP nu (B,a) = C v CF

Qu trnh chng minh nh l ny -c tin hnh t-ng t nh- nh l trn.


V d 4.1.2. Cho G = (N,T,P,k) :
N = {A,B,k}, N = {0,1}
P : kB1, k1, AB1, BA0, A1
Ta xy dng automat M=(S,,,q0,F) xc nh nh- sau:
S = N{C} = {A,B,C,k}
q0 = {k}
F = {k,C}
:

(k,1) = {A,B} , (A,1) = {B,C}

(B,0) = {A}, (C,0) = , (C,1) =


Bn c hy t kim tra t w=10101 c thuc L(G) v T(M) khng?
4.2. Mt s tnh cht ca ngn ng loi 3
Tnh cht 4.2.1. Ngn ng loi 3 khp kn vi php hp.
Chng minh: Gi s L1, L2 l ngn ng loi 3 t-ng ng sinh bi G1=<N1,T1,P1,k1>;

G2=<N2, T2, P2, k2> y L1=L(G1); L2=L(G2). Chng ta xy dng vn phm G3 nh- sau :

G = <N, T, P, k>; trong N=N1N2{k}, k ch-a c trong N1, N2 ,T1,T2; T =T1T2


P = P1P2 {k k1P1 or k2P2 }
D dng thy rng wL(G) khi v ch khi wL(G1) hoc wL(G2). Do vy
L(G) = L(G1)L(G2)
Ch rng trong tr-ng hp L(G1) hoc L(G2) cha t rng ta thm vo G qui tc k.
Tnh cht 4.2.2. Ngn ng loi 3 khp kn vi php ly phn b
Chng minh: Gi s A = <S, , , q0, F> on nhn tp T(A)*, gi s 1 l tp cha

trong . Ta xy dng automat A' = <S', 1, ', q0, F'> on nhn tp 1*\T(A) nh- sau:
S' = {S}{d}; F' = {K\F}{d};
' xc nh nh- sau:
'(q,a) = (q,a) nu qS, a
'(q,a) = d nu qS, a1\
'(d,a) = d nu a1

Vi cch xy dng A' nh- trn r rng A' l m rng ca A trn tp 1 bng cch thm

vo trng thi d v c th kim tra -c rng A' on nhn 1*\T(A).


Tnh cht 4.2.3. Tt c cc xu hu hn u l ngn ng loi 3

Chng minh: Xt xu hu hn x = a1a2...an . Chng ta xy dng automat A gm n+2 trng

thi gm: q0, q1 , q2,..., qn v p, trong q0 l trng thi u, qn l trng thi kt thc. A hot ng
nh- sau khi n nhn ra k hiu thuc X, n s dch chuyn n trng thi c ch s cao hn, nu

A khng nhn ra k hiu khng thuc X n s chuyn sang trng thi p l trng thi "by". C
th hm chuyn ca A nh- sau:

(qi-1,ai) = qi vi i = 1,2,...n

(qi-1,a) = p; vi a ai v i = 1, 2,...n
(qn ,a) = (p,a) = p vi mi a
D dng kim tra -c rng A on nhn xu x.
Trong tr-ng hp x l xu rng chng ta c th xy dng A = <S,,, q0,F>, trong
S={q0,p}; F={q0}; ( q0,)=q0;( q0,a)=( p,a)=p vi a
Tnh cht 4.2.4. Lp cc ngn ng loi 3 ng vi php nhn ghp
Chng minh: Gi s M1 = (S1, 1, 1 , q1, F1) on nhn L1 = T(M1); M2=(S2,2,2,q2,F2)

on nhn L2 = T(M2), ta cn chng minh L1L2 = {uvuL1; vL2} l ngn ng loi 3.

Khng gim tng qut, ta gi s S1,S2 giao nhau bng rng v 1=2=. Ta xy dng

automat M=(S,,,q1 ,F) nh- sau:

S = S1S2; F = F2 nu t rng khng thuc L2 , ng-c li F = F1F2


Hm chuyn xy dng nh- sau:
(q,a) = {1(q,a)} vi qS1\F1, a.
(q,a) = {1(q,a), 2(q,a)}vi qF1
(q,a) = {2(q,a)} vi qS2; a.
D dng thy rng qui tc th nht cho php M hot ng nh- M1 nu q ch-a trng thi

kt thc ca M1. Qui tc th hai cho php x l tnh hung khi q l trng thi kt thc ca M1 v
bt u t on nhn bi M2 . Qui tc th ba m phng s hot ng ca M2 trn M.
Tnh cht 4.2.5: Lp cc ngn ng loi 3 ng vi php ly bao ng
Chng minh: Gi s M = (S,,,q0,F) l automat on nhn L ta gi L* = n=0 Ln l php

ly bao ng ca L vi L0={}; Li = Li-1L. Chng ta xy dng automat M' nh- sau:


M' = (S', , ', q0', F'), trong
S' = S{q'0}; F' = F{q'0 }
Hm chuyn ' xy dng nh- sau:
'(q'0,a) = {(q0 ,a),q0} nu (q0,a) F.
'(q'0,a) = (q0,a) nu (q0,a)F.
'(q,a) = {(q,a),q0} nu (q,a)F; qS
'(q,a) = (q,a) nu (q,a)F; qS

Qui tc th nht i vi trng thi q'0 chp nhn t rng . Nu q0 khng thuc F th trng

thi q'0 s ging q0.

Gi s xL* khi hoc x = hoc x = x1 x2x3...xn vi xiL, i=1,2...,n.


R rng M' on nhn t rng . Gi s x i L suy ra (q, xi)F, do (q 0 ,x i) v

(q0 ,x i) u cha q 0 v mt trng thi p no trong F, cng do vy (q0 ,xi) phi cha
mt trng thi no trong F suy ra xT(M).

Ng-c li xT(M) khi x = a1a2a3...an , suy ra phi tn ti mt dy cc trng thi

q1 ,q2 ,...,qn sao cho (q0,a1) cha q1 v (qi,ai+1) cha qi+1 vi i = 1,2,...,n v qn F. Nh- vy vi
mi i hoc cha qi+1 = q0 v (qi,ai+1)F hoc (qi,ai+1) = qi+1. T y suy ra c th vit (q0,
ai)F vi i = 1,2,...n ngha l aiL.

4.3. Mt s tnh cht ca vn phm phi ng cnh


D-i y chng ta s b-c u lm quen vi lp ngn ng phi ng cnh, lp ngn ng

quan trng v rt gn vi cc ngn ng lp trnh. d hnh dung tr-c khi i vo xem xt vn


phm phi ng cnh, d-i y chng ta s xt v d v vic ng dng vn phm phi ng cnh
trong vic to ra cc thnh phn ca ngn ng trnh Pascal.

V d 4.3.1. Khi cng b ngn ng chun Pascal, ng-i ta s dng cch ghi ca Bakur-

Naur (dng Bakur-Naur) gi tt l cch ghi BNF. Cch ghi ny s dng k hiu '::=', c l
-c nh ngha l. Bin trong Pascal -c m t nh- sau:
<Ch ci>:: = a b .... z A B ... z
<Ch s>:: = 0 1 2 .... 9
<S nguyn>: = <Ch s> <s nguyn> <ch s>
<T> : : =<ch ci > <t > <ch ci>
<Tn gi>::= <T> <Tn gi > <T> <tn gi> <s>
<Bin n gin>::=<Tn gi>

D dng thy rng cch ghi BNF l mt cch ghi ngn gn cc qui tc dn xut ca vn

phm. Ta c th chuyn i t-ng -ng gia hai cch ghi ny. Chng hn vi cch ghi BNF
trn ta c th chuyn v cch ghi dng qui tc dn xut ca vn phm G nh- sau:
T={a,b...,z,A,B,...,Z,0,1....,9}
N = {<Ch ci>, <Ch s>, <S nguyn>, <T>, <Tn gi>,<Bin n gin>} (mi
phn t dng < ... > ca tp N -c hiu l mt k t).

k= <Bin n gin>
Chng ta xc nh P l tp cc qui tc -c chuyn t cng thc siu ng:

<Ch ci>

<s nguyn> <Ch s>

<Ch ci>

<s nguyn> <s nguyn> <ch s>

<Ch ci>

<Ch ci>

<Ch ci>

...........................
<Ch ci>

<Ch s>

<Ch s>

<Ch s>

<ch ci>

<t>

<t> <t> <ch ci>


<tn gi>

<t>

<tn gi> <tn gi> <t>


<tn gi> <tn gi> <s>
<Bin n gin> <tn gi>

...........................
<Ch s>

Vi cch xy dng trn ta nhn -c vn phm G=(T,N,P,k), r rng G l vn phm phi

ng cnh, ngn ng sinh ra bi vn phm G l mt tp v hn cc xu bt u bng ch ci :


L(G)={cc xu bt u bng ch ci}.

Gi s cho vn phm G phi ng cnh = (N, T, P, k). Chng ta d dng nhn thy rng tp

cc bin thuc N v tp cc qui tc dn xut P l hai tp quyt nh to ra ngn ng. Khi vit
ch-ng trnh dch, s phc tp, cng knh, tnh hiu qu ca ch-ng trnh dch ph thuc rt

nhiu vo hai tp ny. Vn t ra y lm sao thu nh tp N, tp P m khng lm nh

h-ng n tp ngn ng L(G) do vn phm G sinh ra. V l do ny, vic rt gn tp N v tp P


-c coi l c s l thuyt quan trng ca vn phm phi ng cnh.
Rt gn vn phm phi ng cnh
nh ngha 4.3.1. Cho vn phm phi ng cnh G = (N, T, p, k). Phn t x N T -c

gi l k hiu khng t n -c trong G nu x khng xut hin trong bt k dy dn xut no


xut pht t k trong G.

Ta gi cc k hiu khng tham gia sinh ra ngn ng l cc k hiu tha, khi cc k hiu

khng t ti -c s l tp con trong tp cc k hiu tha.

nh l 4.3.1. Tn ti thut ton loi b cc k t khng t n -c trong vn phm phi

ng cnh G.

Chng minh: Gi s cho vn phm phi ng cnh G = (N, T, p, k), thut ton -c xy

dng nh- sau:

Chng ta xy dng tp V theo cc b-c:


B-c 1: vi

i=0 ly Vo = {k}

B-c 2: vi i>0 ly Vi = x A x p A Vi-1 Vi-1


B-c 3: Nu Vi Vi-1 th i = i+1 quay li b-c 2, ng-c li lm theo b-c 4
B-c 4: ly V= Vi = Vi-1 v dng.
Thut ton ton trn s dng sau mt s b-c hu hn v dy {Vi} l dy khng gim v

b chn bi tp N T do Vi N T vi mi i.
Ta c: N' = V N ; T' = V T.

Ta gi P' l tp tt c cc qui tc P m ch cha cc k hiu thuc Vi khi :


G' = (N', T', P', k)
Cng vic cn li l kim tra tnh t-ng -ng gia 2 vn phm. R rng L(G') L(G) v

N' N, T' T v P' P. Ng-c li xL(G) suy ra k *x, dy dn xut ny nm trong P,

nh-ng theo cch xy dng P' dn xut ny phi gm cc qui tc trong p, do vy xL(G')
L(G') = L(G).

v1

B
B1

v0

A1
B2

v2

A2
B3

A4
A3
B4

B5

C th minh ho cch xy dng tp V ca thut ton trong nh l trn theo hnh trn.
V d 4.3.1. Ta xt mt v d minh ho cho thut ton ca nh l 3.

Cho G = (N, T, P, k)
N = { k, , , }
T = { x, y, z}
P:

kx
ky

y
x
y
x

Vo = {k}
Vi = {, , x, y} {k}
V2 = V1 {k} = V1

N' = V2 N = {k, , }
T' = V2 T = { x, y}
P':

k x
ky

x
y

Khi L(G) = L(G').


nh ngha 4.3.2. Cho vn phm phi ng cnh G = (N, T, p, k) vn phm G -c gi l

khng rng nu L(G)

nh l 4.3.2. Cho vn phm phi ng cnh G = (N, T, P, k) khi tn ti thut ton xc

nh G l rng hay khc rng.

Chng ta s khng chng minh nh l ny, ch tha nhn kt qu ca nh l phc v

cho vic xem xt phn sau.

nh l 4.3.1 cho php loi b k hiu khng t n -c, tuy nhin trong vn phm

vn cn cc k hiu tha. nh l d-i y cho php chng ta tip tc loi b i cc bin khng
tham gia trong qu trnh sinh ra ngn ng.

nh l 4.3.3. Cho vn phm phi ng cnh G = (N, T, P, k), L(G) . Khi tn ti vn

phm phi ng cnh G' sao cho:

a/ Vi mi nonterminal A trong G u tn ti mt t w T+ A * w
b/ L(G) = L(G).
Chng minh: Vi mi A N ta xt vn phm GA = (N, T, P, A), nh nh l 3 ta c th

xc nh -c L(GA) = hay L(GA) . Nu L(GA) = ta s loi A khi N cng vi tt c cc

dn xut trong P m A c mt v tri hoc v phi. Sau khi b i nhng nonterminal A m


L(GA) = ta nhn -c G' = (N', T, P', k).

Ta cn kim tra s t-ng -ng ca hai vn phm G v G', ngha l L(G)=L(G')?


Tr-c ht ta thy ngay rng L(G') L(G), v N' N, P'P.
By gi ta chng t L(G) L(G')
Ly w L(G). Gi s w khng thuc L(G') khi suy ra dn xut ca w trong G ch c

th l dn xut s dng cc qui tc c trong P m khng c trong P', iu ny cng c ngha l

s dng t nht mt qui tc lin quan n nonterminal A m L(GA) = . Gi s dn xut ca


w trong G c dng:

k 1A2 * w vi A N \ N'.
T y suy ra w c th vit -c w=w1w2w3 sao cho:
1 * w1; A* w2; 2 * w3
iu ny suy ra L(GA) hay A N' mu thun vi L(GA) = v A N \ N'. T y

suy ra L(G)=L(G') .

Rt gn cc qui tc dn xut
nh ngha 4.3.3. Cho vn phm phi ng cnh G ta gi dn xut dng: k 1 2 ...

n l dn xut tri nht trong G nu vi i , i c th vit -c d-i dng i = xiAi i, trong


xi T* , Ai N, i (N T)*.

nh ngha trn cho chng ta khi nim dn xut tri nht. l dn xut m trong mi

b-c p dng cc qui tc trong G, k hiu nonterminal tri nht trong xu v tri -c thay th
bng mt xu nh p dng mt quy tc no ca P.

B 4.3.1. Cho vn phm phi ng cnh G = (N, T, P, k) nu k * w l mt dn xut

trong G khi tn ti mt dn xut tri nht ca w trong G.

nh l 4.3.4. Cho vn phm phi ng cnh G = (N, T, p, k) lun lun tn ti vn phm G'

t-ng -ng vi G sao cho trong G' khng c cc quy tc dn xut dng:
AB

A,B G'

Chng minh:
Gi s cho G = (N, T, P, k). Ta chia cc quy tc dn xut ca P thnh 2 loi:
Loi 1: Gm cc qui tc dn xut dng A B A,B N
Loi 2: Gm cc quy tc cn li ca P
Ta xy dng tp P' cc quy tc dn xut ca G' nh- sau:
Tr-c ht P' gm tt c cc quy tc kiu 2 ca P
Nu A B

A,B N l qui tc loi 1 trong P th ta b xung cc quy tc dng

A vo P' nu c qui tc B l quy tc kiu 2 trong P.

Gi G' = (N, T, P', k). Ta cn chng t L(G') = L(G).


Tr-c ht ta nhn ra rng L(G') L(G) v P'P. Ng-c li gi s w L(G) ngha l k *

w; ta xt dn xut tri nht ca w trong G dng: k12 ... n = w. Nu k 12 ...

n = w m vi i, 1 i n u l quy tc loi 2 th -ng nhin w L(G'). Gi s b-c th


i no bt u s dng qui tc loi 1 v mi b-c th ji u l qui tc loi 2, ngha l dn

xut c th tch dng:

k *i-1 i * k*n
Trong on dn xut ii+1i+2 ... k , p dng quy tc loi 1.
Do on dn xut cng l ii+1i+2 ...k l dn xut tri nht v p dng quy tc

loi 1, cho nn cc xu i, i+1, ... k c di bng nhau. Mt k hiu thay bng mt k hiu

trong G cng v tr, nh-ng khi c th thay th mi b-c dn xut trong dy dn xut i

*k bng mt s quy tc trong G' . Do vy suy ra w L(G') hay L(G) L(G'), t y suy ra

L(G) = L(G').

Ch nh l trn khng nh c th rt gn tp quy tc P bng cch b i cc quy tc

dng AB, c bit nu A=B chng ta loi b -c nhng quy tc chu trnh trong G. tin

cho vic trnh by tip theo chng ta c mt vi nh ngha sau:

nh ngha 4.3.4. Vn phm phi ng cnh G = (N, T, p, k) -c gi l khng cha -quy

tc nu:

P khng cha cc quy tc dng A ( l xu rng)


Hoc qui tc k P v k khng xut hin bt k v phi trong cc quy tc ca p.
nh ngha 4.3.5. Cho vn phm phi ng cnh G = (N, T, p, k), quy tc dng AB, vi

A,B N -c gi l quy tc n gin trong G.

nh ngha 4.3.6.Vn phm phi ng cnh G -c gi l khng c chu trnh nu trong P

khng c quy tc dng:


AA

A N.

nh ngha 4.3.7. Vn phm G -c gi l ti gin nu n khng c chu trnh, khng c

-quy tc, khng c k hiu tha, khng c k hiu khng t -c, khng cha qui tc n gin.
nh l 4.3.5. Cho vn phm phi ng cnh G = (N, T, p, k) bt k khi tn ti mt vn

phm ti gin G sao cho L(G') = L(G).

Vic chng minh nh l trn l khng cn thit, v thc cht n l tng hp cc kt qu

nu trong cc nh l -c chng minh trn, nh nh l ny, t y chng ta lun lun c


th gi thit ch xt cc vn phm ti gin.

4.4. Cc dng chun ca vn phm phi ng cnh


Phn trn chng xem xt vn rt gn vn phm phi ng cnh nhn -c vn

phm ti gin. Tuy nhin mi ch nh- vy thi ch-a . Khi vit cc ch-ng trnh dch, ch-ng
trnh s phc tp ln rt nhiu nu mi qui tc dn xut c dng khc nhau. Vn t ra y
l liu c th -a cc qui tc dn xut ca P v cng mt dng no -c khng. Vic i tm

dng chung cho cc qui tc dn xut ca P -c gi tm dng chun cho vn phm. Ta s xt


y hai dng chun c tn gi dng chun Chomsky (Chomsky normal form) v dng chun
Greibach.

nh l 4.4.1. Dng chun Chomsky. Cho G = (N, T, P, k) l vn phm phi ng cnh, khi

s tn ti vn phm phi ng cnh G' sao cho:


1) L(G) = L(G')

2) Mi quy tc dn xut ca G' c dng:


ABC
hoc Aa

A,B,C N
a T.

Vn phm G' -c gi l dng chun Chomsky t-ng -ng vi G.


V d 4.4.1. Xt vn phm G = (N, T, p, k) :
N = A,B,k
P:

T = a,b

1) kbA
2) kaB
3) Aa
4) Bb
5) Aak
6) Bbk
7) AbAA
8) BaBB

Chng ta cn tm vn phm dng chun Chomsky t-ng -ng vi G, vn phm ny

khng c quy tc n gin dng AB.


Tr-c ht ta xy dng P':
1) Aa P'

(quy tc 3 ca P)

2) Bb P'

(quy tc 4 ca P)

Vi quy tc 1 ca P: kbA, ta b xung C1 vo N v thay kC1A, C1b. Vi quy tc 2

ca P: kaB, ta b xung C2 vo N v thay kC2 B, C2a. Vi quy tc 5 ca P: Aak, ta b


xung C3 vo N v thay BC3k C3a. Vi quy tc 6 ca P: Bbk, ta b xung C4 vo N v thay

BC4k C4b. Vi quy tc 7 ca P: AbAA, ta b xung C5 vo N v thay AC5 AA v C5b,

t AC5AA li b xung D1 vo N v thay AC5D1; D1AA. Cui cng vi quy tc 8 ca P:

BaBB, ta thm C6 vo N v thay BC6BB


BC6D2

D2BB.

C6a ; vi BC6BB ta thm D2 vo N :

Kt qu ta -c G' = (N', T, p', k) nh- sau:


N' = A,B,k,C1,C2 ,C3,C4,C5,C6,D1,D2
P':

1) Aa

8) C3 a

2) Bb

9) BC4k

3) kC1A

10) C4b

4) C1b

11) AC5D1

5) kC2B

12) D1AA

6) C2a

13) BC6D2

7) BC3 k

14) D2BB

R rng l G' dng chun m L(G) = L(G').


nh l 4.4.2. Dng chun Greibach. Cho vn phm phi ng cnh G = (N, T, P, k). C th

tm -c vn phm phi ng cnh G' =(N', T, P', k) t-ng -ng sao cho mi quy tc dn xut
ca G' c dng:
Aa

AN', a T, N'* v L(G) = L(G')

Vn phm G' nh- trn -c gi l vn phm dng chun GreiBach t-ng -ng vi G.
4.5. Lc l-ng ca vn phm phi ng cnh
Chng ta cp n tnh rng ca vn phm phi ng cnh. Chng ta khng nh tn

ti thut ton on nhn ngn ng phi ng cnh l rng hay khng. Do yu cu din t thut

ton rt phong ph a dng, ng-i ta cn mt vn phm sinh ra ngn ng l tp v hn. Vn


t ra l liu c cch no kim tra xem vn phm cho sinh ra tp ngn ng hu hn hay v
hn, phn ny s cp n lc l-ng ca tp ngn ng do mt vn phm phi ng cnh sinh ra.

nh l 4.5.1. Cho L l ngn ng phi ng cnh bt k. Khi tn ti hai s p v q ch ph

thuc L, sao cho zL, z p th z c th vit -c dng z = uvwxy tho mn iu kin:


vwx q
Cc xu x,v khng ng thi l xu rng e
Vi i 0

vuviwxiy L

(nh l trn cn c tn gi l nh l uvwxy)


nh l 4.5.2. Tn ti thut ton xc nh vn phm phi ng cnh cho l hu hn

hay v hn.

Chng minh: Cho G = (N, T, p, k). Chn p = 2N-1 q = 2N .


Gi s c t z L(G) z p. Khi theo nh l 3 xu z c th vit d-i dng z =

uvwxy m xu x v xu v khng ng thi l xu rng (v+x 0). Khi ta c uviwxiy


L(G) vi mi i tu . T y suy ra L(G) v hn khi tn ti xu c di ln hn p.

Ng-c li nu L(G) v hn n phi c t di ty (v nu khng phi nh- vy L(G) l

hu hn). Do vy trong L(G) phi c t c di hn p+q. Gi s z l xu nh- vy, khi xu z


c th vit d-i dng:

z = uviwxiy sao cho vwx q v v+x 0 v iN


By gi ta gim i cho xu z c di ngn li , mi ln gim i t z ngn li thc s v

v+x 0. Gi s sau mt ln gim i i 1 n v ta nhn -c xu z m z p+q th chng


ta dng, ng-c li ta coi z ng vai tr ca z lp li qu trnh rt gn trn.

Lp lun trn y cho php chng ta suy ra nu L v hn th ta lun lun tm -c t

di l sao cho p l q+p.

Ni mt cch khc L(G) v hn khi v ch khi tn ti t w c di tho mn:


p w p+q
T kt lun trn suy ra kim tra L(G) l hu hn hay v hn ta ch cn kim tra xem

cc xu c di ln hn p v khng nh hn p+q c thuc L(G) hay khng? Thut ton duyt

cc xu nh- vy lun lun kt thc sau mt s hu hn b-c v tp cc xu c di tho mn


iu kin trn l hu hn.
5. My Turing
5.1. M u
Khi nim trc gic v thut ton m chng ta s dng t tr-c n nay l: Thut ton l

mt dy cc qui tc hay ch th xc nh mt qu trnh tnh ton no . Chng hn thut ton


Euclide tm -c s chung ln nht ca hai s nguyn cho tr-c, thut ton xc nh mt s

nguyn cho tr-c c l s nguyn t hay khng? Tnh gn ng gi tr ca sinx, cosx, v.v... Nu
khng c cc bi ton dn n vic nghi ng c hay khng c thut ton gii quyt mt bi ton
no th ng-i ta khng cn bn ci n khi nim mang tnh trc gic nu trn. Bi v v mt

logic, mun ni c hay khng c thut ton th phi xc nh -c chnh xc thut ton l g?
Xut pht t cc bi ton khng bit c thut ton gii n hay khng? buc ng-i ta phi t

cu hi thut ton l g? Liu c th m t ton hc chnh xc v n hay khng? Lin quan n


vn ny chng ta lm quen vi my Turing.

Vo khong nm 1930-1936 cc nh ton hc Godel, Kleene, Church, Turing c gng

-a ra m t ton hc cho khi nim thut ton. Cc khi nim v thut ton ca cc tc gi nu

trn ni chung u t-ng -ng vi nhau. Trong s cc khi nim ton hc chnh xc v v
thut ton, khi nim do nh ton hcTuring -a ra nm 1937 -c tha nhn rng ri bi v n
thun tin cho vic nghin cu cc vn trong l thuyt v ng dng. My Turing m nh ton

hc Turing dng m t khi nim thut ton c gi tr c trong nghin cu l thuyt tru

t-ng ln trong cc qu trnh tnh ton c th. Sau khi xut hin my tnh in t, my Turing
tr thnh mt m hnh ton hc cho cc my tnh thc t.

Nh- chng ta bit, mi qu trnh thc hin theo thut ton, xt cho cng u l qu

trnh thc hin mt s php bin i s cp no . Trong m hnh my Turing, ng-i ta ch


dng mt loi bin i s cp n gin l thay th mt k hiu ny bng mt k hiu khc. V
vy my Turing l m hnh m t ton hc khi nim thut ton rt ph hp. Sau khi c khi

nim chnh xc v thut ton, ng-i ta c c s chuyn bi ton khng c thut ton gii n

v bi ton khng c my Turing tnh n. Trn c s khi nim thut ton, cch y gn na th
k Turing v Church nu ra lun mang tn Turing Church. Mc d lun khng c
chng minh nh-ng cho n nay ng-i ta vn khng tm -c phn v d ph nh gi tr chn l

ca n. Lun Turing Church c th tm tt nh- sau: Mi hm s tnh -c theo ngha trc

gic u l hm tnh -c bi mt my Turing no .

My Turing -c s dng nghin cu trong nhiu lnh vc khc nhau, tin cho vic

nghin cu, trong mi lnh vc ng-i ta -a ra cc nh ngha khc nhau v my Turing, trong
phm vi ca gio trnh ny, chng ta s nu nh ngha my Turing v kho st n theo gc
ngn ng.

5.2. Khi nim v nh ngha


V mt trc quan c th coi my Turing gm b iu khin c hu hn trng thi (finite

control), mt bng vo (Input tape), bng ny -c chia thnh tng ngn hay tng (cell). Bng

vo c ngn tri nht -c coi l ngn u tin ca bng v n -c coi l di v hn v pha


bn phi. My c mt u c bng (tape head), u c ny c th d c tng ngn trn bng
vo. Mi ngn ca bng vo c th cha mt k hiu thuc tp cc k hiu vo (Input symbol).

Mt dng vo l mt dy cc k hiu ca bng ch ci vo t trn bng, cc ngn trn bng

khng cha k hiu vo -c coi cha k hiu trng (blank), ta k hiu l B. Hnh 1 m t cu to
ca my Turing.

# a1 a2 a3 ai an # # #

B iu khin
Hnh 1.

Hot ng ca my Turing -c m t nh- sau:


Gi s my trng thi q, u c ang ch vo k hiu ai, n s chuyn u c sang

pha tri hoc pha phi ca ai v thay ai bng bj no Ni mt cch khc, ph thuc vo k

hiu m u c ang c trn bng vo v trng thi ca b iu khin my c th thc hin


cc thao tc sau:

Thay i trng thi


Ghi mt k hiu khc k hiu trng ln k hiu c trn bng vo.
Dch chuyn u c i mt ngn sang tri hoc sang phi.
nh ngha 5.2.1. V mt hnh thc ta c th coi my Turing l b 5 : T=<S,,,q0,F>.

Trong :

- S l tp hu hn cc trng thi.
- l tp hu hn k hiu vo.
- l nh x, -c gi l hm chuyn trng thi, : S x x {L,R } S x .
Ch rng c th khng xc nh vi mt vi b gi tr ca cc i s no ; L,R l k

hiu ch h-ng dch chuyn sang tri hoc sang phi t-ng ng ca u c. Khi k hiu

(s,a,M)=(q,b) c ngha rng automat trng thi s, u c ang tr vo k t a my i sang


trng thi q, thay k t a bng k t b v di chuyn u c theo h-ng M.
- q0 l phn t thuc S gi l trng thi bt u ca my Turing
- F S l tp cc trng thi kt thc ca automat.
a)
b)

s
s

a/b
a.

L
s

L
s

c) R
s

d)

R
s

a/b
a/b

s
s

Hnh 2.
Hnh 2 m t cch lm vic v chuyn i trng thi ca my Turingng vi cc tr-ng hp:
a/(s,a,L)=(s,b)
b/(s,a,L)=(s,a)
c/(s,a,R)=(s,b)
d/(s,a,R)=(s,a)

nh ngha 5.2.2. Ta gi b bn (s,x,,y) l hnh trng ca my Turing, vi sS, s l trng

thi hin thi ca T; x, y, *,, y l k hiu nm trn bng vo m u c ang tr

vo n, x l t bn tri u c, y l t bn phi u c.

Gi s my Turing T hnh trng (s,x,,y). N -c gi l chuyn sang hnh trng

(s,x,,y), nu (s,,M)=(s,). Khi ta vit:


(s,x,,y) (s,x,,y)

Gi s my Turing T hnh trng (s,x,,y) v c mt dy hnh trng

(s1,x1,1,y1),(s2,x2,2,y2),...,(sn ,xn ,n,yn) m:

(s,x,,y)(s1,x1,1,y1)(s2,x2,2,y2)...(sn,xn,n,yn).
Khi ta k hiu ngn gn l: (s,x,,y)*(sn,xn,n,yn).
Nu my Turing T ang mt hnh trng no m khng th chuyn sang hnh trng

no khc. Khi c th xy ra mt trong hai tnh hung sau:

My ang hnh trng c cha trng thi qfF S, khi ta ni my Turing hnh
trng kt thc.

My ang hnh trng m u c ngn u tin bn tri m n vn phi tip tc


di chuyn v bn tri, khi ta ni my Turing hnh trng tt.

My Turing mt trong hai hnh trng kt thc hoc tt gi l hnh trng dng hoc cht.
Hnh trng (q0,e,#,x) -c gi l hnh trng ban u ca my Turing T, y e l k hiu

xu rng,# l k hiu gii hn tri ca xu x , u c tr vo k t u tin ca x.


5.3. Hm Turing thc hin -c

nh ngha 5.3.1. Gi s l mt bng ch ci, l nh x : * *. -c gi l

hm Turing thc hin -c, nu tn ti my Turing T = <S, , , q0, F> sao cho

(q0,e,#,x)*(q,z,,y) v hnh trng (q,z, ,y) l hnh trng dng khi v ch khi (x)=y. Nu i

vi xu vo #x my T hnh trng dng nh-ng khng t n hnh trng kt thc, th (x) -c


coi l khng xc nh vi x.

nh ngha 5.3.2. Cho my Turing T = <S,,,q0,F> ta gi tp


L(T)={x*(q0 ,e,#,x)*(s,z,,y), sF}
l ngn ng on nhn bi my Turing T.

dng.

Hnh 1 m t tnh trng trn bng vo khi my Turing trng thi bt u v trng thi
t vo trn bng
#

t kt thc trn bng

.. .. .. ..

Hnh 1.
Ch : My Turing T = <S,,,q0,F> -c gi l tt nh, nu 0(s,a,M) 1; ng-c li

T -c gi l khng tt nh. Mt ngn ng c th on nhn bi my Turing tt nh hoc


khng tt nh.

C th nh ngha ngn ng on nhn bi my Turing T = <S,,,q0,F> bng cch sau:

Gi s l hm no : * * ta gi min xc nh ca hm Turing thc hin -c l

ngn ng on nhn bi my Turing T.

V d 5.3.1. My Turing cho bi s hnh 2 on nhn ngn ng l tp


L = {an bncn ; nN}.
a, b, c, x, y, z
L
#
x

(yes)

s3

c/z

s4

s6

s2

R
b/y

s1

a/x

R
R

b/z

R
#, c

a, b, c

s5

L
R

a, b, c, x, y, z #, a
#

(no)
s7

Hnh 2.
Khi nim vn phm, ngn ng v phn loi ngn ng s -c cp phn sau, tuy

nhin ngay t y, chng ta c th hiu vn phm loi 0 l vn phm khng c bt k mt iu

kin rng buc no v n sinh ra lp ngn ng rt phc tp nh- lp cc ngn ng t nhin m


loi ng-i ang s dng. Lin quan n lp ngn ng loi 0 ta c nh l sau.

nh l 5.3.1. Ngn ng L sinh bi vn phm loi 0 khi v ch khi n l ngn ng on

nhn bi my Turing .

Chng ta s khng chng minh nh l ny, tuy nhin kt qu nh l trn cho chng ta

nhn nhn mt cch h thng vn ang xem xt: C th nghin cu ngn ng theo quan im
sinh v quan im on nhn, hai quan im ny ng vi hai cng c l vn phm v Automat.
Kt qu thu -c t hai cng c ny l t-ng -ng nhau.

Nh my Turing chng ta -a ra khi nim hm Turing thc hin -c. Thc cht ca

hm Turing thc hin -c l hm bin i mt xu x sang xu x vi x,x*. Da vo hm


Turing thc hin -c chng ta -a ra khi nim hm Turing tnh -c.

nh ngha 5.3.3. Chng ta ni rng hm :NnN l Turing tnh -c, nu tn ti my

Turing thc hin hm :1,&* 1* tho mn iu kin :

a/ (1x1+1&1x2+1&......,&1xk+1)=1y+1, nu (x1,x2,....xk)=y ;
b/ (1x1+1&1x2+1&.....,&1xk+1)= khng xc nh, nu (x1,x2 ,.....xk) khng xc nh.
V d 5.3.2. Hm f(x,y)=x+y l hm Turing tnh -c.
Ta xy dng my Turing T=<S,,,q0,F >
'=1,&,B ,S==q0, q1, q2,q3
Hm chuyn trng thi -c cho trong hnh 3c. Hnh 3a, 3b t-ng ng l trng thi ca bng vo
lc bt u v khi kt thc.

# 111&111..1
a)

1/B

q0 R

q0

#B111
b)

R q1
&/1

Stop
q3

B/B
c)

R
q2

1/1

Hnh 3.
5.4. phc tp ca thut ton
Nh my Turing, chng ta c -c khi nim chnh xc v thut ton. Tuy nhin vi

mt hm Turing tnh -c ng-i ta c th xy dng cc my Turing khc nhau, ngha l c cc

thut ton khc nhau thc hin mt bi ton. Mt cch t nhin cu hi -c t ra l: trong
cc thut ton tnh hm f, ton thut no tt hn, tt hn theo ngha no ? y l vn rt quan
trng trong khoa hc tnh ton v ng dng thc tin. Lin quan n vn ny chng ta lm
quen vi mt s khi nim sau:

nh ngha 5.4.1. Gi s w* , ta k hiu T(w) l s cc b-c c bn ca qu trnh hot

ng ca my Turing T on nhn t w. Gi s 0,1,.,k l dy cc hnh trng m my

Turing T on nhn t w, vi k l hnh trng kt thc, ngha l 01 k khi T(w) xc


nh nh- sau: T(w)=k, nu T on nhn t w, ng-c li T(w) khng xc nh. Hm T(w) -c
gi l hm xc nh thi gian tnh ton. Vic xc nh, nh gi hm T -c gi l nh gi
phc tp thut ton theo thi gian

nh ngha 5.4.2. Gi s w* v 0 ,1,.....,k l dy hnh trng m my Turing T on

nhn t w, hnh trng i my T s dng i nh trn bng khi ta xc nh L(w) nh- sau:

L(w)= max i i=1,2..k nu T on nhn xu w, khng xc nh nu T khng on nhn xu w.

Hm L(w) -c gi l hm xc nh dung tch b nh.

Vic xc nh, nh gi hm L(w) -c gi l nh gi phc tp thut ton theo dung

tch b nh.

nh ngha 5.4.3. Gi s c hai hm f: N N v g:N N , chng ta ni rng hm f c

cng bc vi hm g nu tn ti s k v s m sao cho sao cho f(n) kg(n) vi mi n m k hiu


f=O(g).

Gi s f l hm xc nh thi gian tnh hoc hoc xc nh dung tch b nh khi thc hin

mt thut ton no v gi s f=O(g), khi ta ni rng thut ton c phc tp cng bc


vi g. Nu g(n) l a thc bc n th ta ni rng bi ton c phc tp a thc. Nu g(n) l hm
s m theo n th ta ni rng bi ton c phc tp hm s m...

V d 5.4.4. Gi s cho dy s gm k s n1,n2,...,nk , hy sp xp dy s theo th t khng

gim theo thut ton ni bt. Theo thut ton ny chng ta thc hin so snh v i ch hai

phn t ng cnh nhau nu s ng tr-c ln hn s ng sau. Nu coi thao tc c bn y l


php so snh th ln th nht thc hin (k-1) php so snh v phn t ln nht -c t cui

dy,ln th 2 cn k-2.....S php so snh sp xp dy s gm k phn t l (k-1)+(k1)+....+1=k(k-1)/2. T y suy ra phc tp ca thut trn l O(k2).

Bi tp
1. Hy xc nh cc thnh phn ca cc my hu hn trng thi M=(S, X, Y, , , s0), khi bit
biu din th ca chng:
a)
a/1

b/1
q0

q1

a/0

b/1
b)

a/0

A
b/0

b/1
D

a/1

B
a/0

b/0

b/0

a/0

c)
b/0

a/2
a/0

c/2
b/1

a/1

b/2

b/0

B
c/0

c/0
c/2

a/2

2. Mch tun t theo s d-i y l mch flip-flop


Gi s thot u x1=1, x2=1 v y=0 hy xc nh gi tr ca a,b v c.
x1
x2

a
c

b
y

3. Hy ch ra rng mi my hu hn trng thi cho d-i dng cc s d-i y l mt automat


hu hn trng thi v hy v li s ca cc automat t-ng ng

a)

q0

b)

a/0

b/0

q1

b/1

a/1
a/1

A
B

b/0

a/0
C

b/0
4. Cho L l tp hu hn cc xu sinh ra t tp a,b ch ra rng tn ti automat hu hn on
nhn tp L.

5. Hy thit k automat hu hn trng thi ch on nhn cc xu sinh ra t tp a,b nh-ng


khng ch-a k t a.

6. Hy ch ra rng mt xu bt k sinh t tp a,b chp nhn bi automat cho hnh d-i y

khi v ch khi c hai k t cui l bb


a

q0

q1

q2

7. Hy xc nh bng hm chuyn trng thi cho cc automat khng tt nh dng s hnh


d-i y:

a
a)

b)

a
b
b
a

b
b

A
a

b
C

D
b

B
a

8. Hy ch ra cc vn phm sau y thuc loi no ?


a/ T=a,b;N=*,A;* l k t bt u. Tp P gm cc dn xut :

*b*, *aA , Aa* , AbA , A a , *b


b/ T=a,b,c; N=*,A,B ;* l k t bt u. Tp P gm cc dn xut :
* AB , ABBA, A aA , BBb, A a, B b
c/ T=a,b; N=*,A,B ;* l k t bt u. Tp P gm cc dn xut:
* A, *AAB, Aa ABa ,Aaa , Bb ABb, ABABB, B b.
9. Cho vn phm sau :
T=a,b,c; N=*,A,B,C,D,E ;* l k t bt u. Tp P gm cc dn xut:
* aAB , * aB, AaAC, AaC ,BDc, Db, CDCE.
CEDE ,DE DC, CcDcc
a/Cho bit G l vn phm loi no ?
b/ Hy chng t L(G)= anbncn n=1,2....
c/ Hy vit vn phm trn dng BNF (Backur Naur Form)
10. Hy ch ra rng vn phm sinh G ra tp ngn ng L(G)= anbnck n,k=1,2.... l ngn ng phi
ng cnh (context-free language).

11. Vit vn phm G c tp T=a,b sao cho xu L(G) c mt trong cc tnh cht sau:
a/ Bt u bng k t a
b/ Kt thc bng hai k t ab
c/ Kt thc bng hai k t ba
d/ Khng kt thc bng ab
12. Gi s G l vn phm v l xu rng. Hy ch ra rng nu G c cc qui tc dn xut sau:

A , A B hoc A vi A,BN, T* \, th tn ti vn phm G chnh qui sao cho


L(G)=L(G).

13. Hy xy dng my hu hn trng thi tho mn cc iu kin sau:


a/ Nhn dng vo l dy cc bit
b/ Dng ra bng 1 nu trong dng vo c s ch s 1 l s chn, dng ra bng 0 trong

tr-ng hp ng-c li.

14. Hy xy dng my hu hn trng thi tho mn iu kin sau:

a/ Nhn dng vo l dy cc bit


b/ Dng ra bng 1 nu trong dng vo c k ch s 1 , k l s chia ht cho 3 . dng ra

bng 0 trong tr-ng hp ng-c li.

15. Chng minh rng khng c my hu hn trng thi c tnh cht sau :
a/ Nhn dng vo l dy cc bt
b/ Dng ra l 1 khi dng vo c s ch s 1 bng s ch s 0, dng ra bng 0 trong

tr-ng hp ng-c li

16. Hy xy dng vn phm G sao cho L(G)= a2n-1 n N .


17. Hy xy dng vn phm G sao cho L(G)= an b2m n, m N .
18. Cho vn phm G=< N,T,P,k > , N=A,B,k; T= a,b ;
P:

k bA

B b

A bAA

k aB

A ak

B aBB

A a

B bk

p dng nh l 4.5.2 hy ch ra s t nhiu nht cn phi duyt khng nh L(G) l hu hn


hay v hn ?

19. Hy xy dng vn phm G m L(G) l tp cc s thc.


20. Hy xy dng vn phm G m L(G) l tp cc biu thc s hc.
21. Hy xy dng vn phm G m L(G) l tp cc biu thc logic.
22. Hy xy dng vn phm G m L(G) l tp cu lnh trong ngn ng lp trnh Pascal.
23 . Cho vn phm phi ng cnh G=<N, T, P, K>, trong N gm n bin. Hy ch ra s ln
nht cc cy dn xut c di -ng i khng ln hn n+1.Bit rng mi v phi ca cc quy
tc dn xut trong P l xu c di khng ln hn m,

24 . Cho vn phm phi ng cnh G dng chun Chomsky, gi s wL(G) v k * w l dn


xut gm p b-c, khi w l xu c di bao nhiu?
25 . Cho G = <N,T, P, k>
N={k}, T={p, [,],~, }
p: k p
S S

S [S S]
Hy ch ra dng tng qut ca L(G)?
26. Chng minh rng mi vn phm phi ng cnh u c th sinh ra bi vn phm m mi qui
tc dn xut ca n c th mt trong cc dng sau:
A a, A aB, A aBC.
27. Gi s L1, L2 l ngn ng chnh qui trn bng ch ci , S l tp tt c cc xu trn
chng minh rng :

a/ S \ L1, L1 L2 l ngn ng chnh qui.


b/ L1 L2,

L1+ , L1L2 l ngn ng chnh qui

28. Hy xy dng automat hu hn khng tin nh on nhn tt c cc xu trn 0,1, bt


u bng 01 v c cha 110.

Ti liu tham kho


1. Kenneth H. Rosen. Ton ri rc ng dng trong Tin hc.- NXBKHKT, 2000
2. Nguyn c Ngha, Nguyn T Thnh. Ton ri rc.- NXBGD, 1997.
3. R. Johnsonbaugh. Discrete Mathematics.- Macmillan Pub., 1992.
4. E. Goodaire, M. Parment. Discrete Mathematics with Graph Theory.- 1993

You might also like